187
214 Sistemes d’equacions lineals A la cinquena: x x x x x 2 1 4 1 2 8 1 4 16 1 8 : : 2 1 15 16 15 8 2 1 32 x x x 15 16 1 32 1 16 x A la sisena: x x x x x x 2 1 4 1 2 8 1 4 16 1 8 32 1 16 :2 1 31 32 31 16 x x :2 1 64 31 32 1 64 1 32 x x Així, doncs: x x x x x x x 2 1 4 1 2 8 1 4 16 1 8 32 1 16 64 1 32 63 64 63 32 63 x x x 126 64 126 x x 107 La Mònica i l’Eva parlen per telèfon per comprovar si els sistemes que han resolt els donen els mateixos resultats. Només hi ha un resultat diferent. La Mònica diu que les solucions del sistema són x y z 8 7 11 18 7 , , , mentre que l’Eva diu que són x y z 10 11 7 18 11 , , . Després d’assegurar-se que totes dues han escrit l’enunciat del problema de la mateixa manera, comencen a pensar que potser es tracta de dues maneres de resoldre el mateix sistema d’equacions. Decideix-ho tu. x y z x z 8 7 11 18 7 7 8 7 y z x z y z 11 18 7 8 7 11 18 x y z x y 10 11 7 18 11 11 10 11 7 18 11 10 7 11 18 z y x y y z Si formem un sistema amb les tres equacions, tenim que: 7 8 7 11 18 11 10 x z y z x y Finalment, comprovem que les dues solucions són correctes.

Sistemes d’equacions lineals - RedDavid...216 Sistemes d’equacions lineals 3 Considera les matrius A xyx yy zz ¤ ¥¥ ¥¥ ¥¥ ´ µµ µµ µµ µµ 0 1, B (a 2 3) i C (4 0

  • Upload
    others

  • View
    2

  • Download
    0

Embed Size (px)

Citation preview

214

Sistemes d’equacions lineals

A la cinquena:

xx x x x

21

4

1

2 8

1

4 16

1

8: :2 1

15

16

15

82 1

32

xx

x 115

161

32

1

16

x

A la sisena:

xx x x x x

21

4

1

2 8

1

4 16

1

8 32

1

16: 2 1

31

32

31

16x

x:: 2 1

64

31

321

64

1

32

x x

Així, doncs:

x x x x x xx

21

4

1

2 8

1

4 16

1

8 32

1

16 64

1

32

63

64

63

3263

xx x 1126 64 126x x

107 La Mònica i l’Eva parlen per telèfon per comprovar si els sistemes que han resolt els donen els mateixos resultats. Només hi ha un resultat diferent. La Mònica diu

que les solucions del sistema són x y z8

711 18

7, , ,

mentre que l’Eva diu que són x y z10

117 18

11, , . Després

d’assegurar-se que totes dues han escrit l’enunciat del problema de la mateixa manera, comencen a pensar que potser es tracta de dues maneres de resoldre el mateix sistema d’equacions. Decideix-ho tu.

x

y

z

x z

8

7

11 18

7

7 8

7yy z

x z

y z11 18

7 8

7 11 18

x

y

z

x y

10

11

7 18

11

11 100

11 7 18

11 10

7 11 18z y

x y

y z

Si formem un sistema amb les tres equacions, tenim que:

7 8

7 11 18

11 10

x z

y z

x y

Finalment, comprovem que les dues solucions són correctes.

215

3SOLUCIONARI

PREPARA LA SELECTIVITAT(Activitats de Selectivitat)

1 Donada la matriu A 1 20 1

troba totes les matrius P a bc d

de manera que AP PA.

APa b

c d

a c b d1 2

0 1

2 2·

cc d

PAa b

c d

a a b

c c·

1 2

0 1

2

2 d

AP PA

a c a

b d a b

c c

d c d

2

2 2

2

c

d a

c c

c

0

0

Les matrius són de la forma ambPa b

aa

0,bb R.

2 Resol: 2 0 51 1 21 1 1

·xyz

223

502

2 0 5

1 1 2

1 1 1

·

x

y

z

2

2

3

5

0

2

2 0 5

1 1 2

1 1 1

·

x

y

z

7

2

1

x

y

z

2 0 5

1 1 2

11 1 1

7

2

1

1

·

2 0 5

1 1 2

1 1 1

16 0

3

16

5

16

5

161

16

7

16

9

161

8

1A

1

8

1

8

x

y

z

3

16

5

16

5

161

16

7

16

9

1611

8

1

8

1

8

·

7

2

1

1

1

1

216

Sistemes d’equacions lineals

3 Considera les matrius Ax y xy y

z z0

1, B (a 2 3) i C (4 0 2).

a) Troba els valors de x, y i z per als quals A no té inversa.

b) Determina els valors de a per als quals el sistema B A C té solució.

c) Resol el sistema anterior quan sigui possible.

a) A no té inversa si A 0.

x y x

y y

z z

y

x y x

z z

y y yz y z0

1

1 0 1

1

12( ) ( )

La inversa no existeix si y 0 o z 1.

b) BA C a

x y x

y y

z z

( ) · (2 3 0

1

44 0 2

2 3 4

3 0

2 3 2

)

ax y

ay z

ax y z

ax y

ay z

ax y z

A

a

a

a

2 1

3 0

2 3 2

2 0

0 3

22 3

2 0 1

0 3 0

2 3 2

A

a

a

a

*

2 0

36 0

a

a

a

a

a a a

2 0

0 3

2 3

3

2 0 1

3 0

2 3 2

3 62

Si a 0 → rang (A) rang (A*) 3 nre. d’incògnites

Sistema compatible determinat

Si a 0 → rang (A) 2 rang (A*) 3 → Sistema incompatible

c) Si a 0:

A a a xA

A

a

a

a

ax

x

1 2 0

0 3

2 2 3

3 63 6

3

22 2

A

a

a

a yA

A

a

a ay

y

1 0

0 0 3

2 3

33

3

12

A

a

a

a

a zA

A

a

az

z

2 1

0 0

2 2 3

1

3

22

2

217

3SOLUCIONARI

4 Considera el sistema x y z

y z ax z a

12

2 2 en què a és un paràmetre real.

a) Discuteix el sistema d’acord amb el valor de a.b) Resol el sistema per a a 0.c) Resol el sistema per a a 1.

a) A A

1 1 1

0 1 1

1 0 2

1 1 1 1

0* 11 1 2

1 0 2

1 1 1

0 1 1

1 0 2

2

a

a

A 0

1 1 1

0 1 2

1 0

2 1 12

2 2a

a

a a a( )

1 1

0 11 0

rang (A) 2

Si a 1 → rang (A*) 3 rang (A) 2 → Sistema incompatible

Si a 1 → rang (A) rang (A*) 2 nre. d’incògnites

Sistema compatible indeterminat

b) Per a a 0 → El sistema és incompatible, no té solució.

c) Per a a 1 considerem el sistema:

x y z

y z

z

y

z

1

2

1 2

2 amb R

5 Donat el sistema dependent del paràmetre : x y z

x y zx y z

111

a) Determina, de manera raonada, els valors de per als quals el sistema és compatible determinat, compatible indeterminat i incompatible.

b) Resol el sistema quan és compatible determinat.

c) Troba, de manera raonada, la solució del sistema quan 0.

a) A

A

1 1

1 1

1 1

1 1 1

1 1 1*

11 1 1

218

Sistemes d’equacions lineals

A

1 1

1 1

1 1

2 1 1

2 1

2 1

2

1 1 1

1 1

1 1

( )

( ) ( )( )2

1 1 1

0 1 0

0 0 1

2 1 2

1 1

1 1

1 1 1

1 0 1

0 1 1

0 0 1

1 2( )

Si R { 2, 1} → rang (A) rang (A*) 3 nre. d’incògnites

Sistema compatible determinat

Si 2 → rang (A) 2 rang (A*) 3 → Sistema incompatible

Si 1 → rang (A) rang (A*) 1 nre. d’incògnites

Sistema compatible indeterminat

b) Si R { , } ( )( )2 1 2 1 2A

A xA

Ax

x

1 1 1

1 1

1 1

11

2

2( )

A yA

Ay

y

1 1

1 1 1

1 1

11

2

2( )

A zA

Az

z

1 1

1 1

1 1 1

11

2

2( )

c) Si 01

2x y z

6 Un país importa 21.000 vehicles de tres marques, A, B i C al preu de 10.000, 15.000 i 20.000 respectivament. El total de la importació puja 332 milions d’euros. Cal dir també que hi ha 21.000 vehicles comptant només els de la marca B i vegades els de la A.

a) Planteja un sistema d’equacions amb les condicions del problema en funció del nombre de vehicles de cada marca.

b) Estableix el nombre de vehicles de cada marca si suposem que 3.

c) Estudia si existeix algun valor de per al qual la situació no es pugui donar en el camp dels nombres reals.

a) Considerem x, y, z els vehicles de cada marca. Així, doncs:

x y z

x y z

21 000

10 000 15 000 20 000 332 000

.

. . . . ..

.

.

000

21 000

21 000

10 1

x y

x y z

x 55 20 332 000

21 000

y z

x y

.

.

219

3SOLUCIONARI

b) Si 3

21 000

10 15 20 332 000

3

x y z

x y z

x y

.

.

21 000

21 000

10 5 88 000

.

.

.

x y z

x y

33 21 000x y .

x y z

x y

x

21 000

10 5 88 000

5 17 000

.

.

.

x

y

z

3 400

10 800

6 800

.

.

.

c) A A

1 1 1

10 15 20

1 0

1 1 1 21

*

..

.

.

000

10 15 20 332 000

1 0 21 000

1 1 1

10 15 20

1 0

5 101

10

1

155 0

1 1 21 000

10 15 332 000

1 21 000

17 000 17 000

.

.

.

. .

Si 2 → A 0 → rang (A) 2 rang (A*) 3 → Sistema incompatible

7 Determina raonadament els valors del paràmetre m que fan que el sistema tingui més d’una solució.

Com que es tracta d’un sistema homogeni, perquè el sistema tingui més

d’una solució, la matriu dels coeficients ha de tenir rang 2, és a dir,

el determinant ha de ser nul. Així, doncs:

m 2 1 1

1 m 2 1

1 1 m 4

0 m3 8m2 17m 1 0

m 5

m 2

m 1

L I T E R AT U R A I M AT E M ÀT I Q U E S

Nocilla ExperienceEn Marc estudia minuciosament el llibre que té davant, Guia agrícola Philips 1968; va trobar-la entre els trastos vells del seu pare i va que-dar-se-la. Observa de reüll el terrat a través de la porta de la caseta. Hi viu. Un cobert, situat al capdamunt d’un edifici de 8 plantes, que ha anat construint amb diferents fulls de llaunes, bidons, trossos de car-tons banyats de petroli i fragments d’uralites. Tot encaixat de manera que les 4 parets configuren un mosaic de paraules i icones escrosto-nades d’oli La Giralda, lubrificants Repsol, Beba Pepsi o sanitaris Roca. De vegades les mira, i entre tot aquest agermanament de mar-ques comercials intenta descobrir mapes, recorreguts, senyals latents d’altres territoris artificials. Al terrat, que ja no freqüenta cap veí, hi ha una sèrie de filferros que van de costat a costat on, en comptes de roba estesa, hi ha fulls escrits, a mà i per una sola cara, amb fórmules matemàtiques; cada una subjectada per una agulla. Quan bufa el vent [sempre bufa] i els mirem de front, el conjunt de fulls formen una mena de mar de tinta teòric i convuls. Si els mirem des de darrere, les cares en blanc dels DIN-A4 semblen la simbologia més exacta d’un desert. Els veu aletejar i pensa, La meva teoria és fascinant. Tanca la Guia agrícola Philips 1968, la deixa sobre la taula, surt i despenja uns quants fulls dels cables número 1, 4 i 7. Abans de tornar a entrar es recolza a la barana i pensa en el Mundial que mai no hem guanyat, en què allò més pla que existeix sobre la Terra són les vies dels trens, en què la música d’El cuirassat Potemkin, si t’hi fixes, és el «Purple Haze» de Jimi Hendrix versionat. Després entra a la caseta, que tre-mola quan tanca la porta amb un cop. […]

Diumenge, són més de les 4 de la tarda, la gent és a la platja; ell enca-ra no ha dinat. Entre les uralites de la caseta entra un pizell de llum que incideix sobre la tecla 0 del PC. Hi sona el CD de Sufjan Stevens, The Avalanche, […] mentre acaba de fer els últims retocs a una demos-tració de la qual se sent molt satisfet. Surt al terrat amb el foli a la mà, i als estenedors que conformen la retícula l’estén en la posició, x 10, y 15. No hi ha res millor per comprovar la fermesa d’una teoria que airejar-la abans de propagar-la, pensa.

AGUSTÍN FERNÁNDEZ MALLO

Números realesGeometria en l’espai4

221

Nocilla ExperienceAgustín Fernández Mallo

El protagonista d’aquesta novel·la, l’autor de la qual és llicenciat en Ciències físiques,

és una mena d’ermità modern que viu en una espècie de cabana que ha construït al terrat

d’una casa.

L’assumpte que el distreu és una teoria que fa anys que té en marxa, emmarcada en una cosa més àmplia que ell anomena sociofísica teòrica. El radi d’acció, el banc de proves per constatar-la, no passa de dues o tres illes de cases al voltant del seu terrat. Al barri troba tot allò que necessita: queviures, converses banals i roba de temporada en tergal. La pretensió de la seva teoria consisteix a demostrar amb termes matemàtics que la soledat és una propietat, un estat, connatural als éssers humans superiors, i per fer-ho es fonamenta en una evidència física que els científics coneixen bé: a la natura només existeixen dues classes de partícules, els fermions [electrons i protons, per exemple] i els bosons [fotons, gluons, gravitons, etcètera]. Els fermions es caracteritzen pel fet, àmpliament demostrat, que no n’hi pot haver dos o més en un mateix estat, o el que és el mateix, que no poden estar junts. La virtut dels bosons és justament la contrària: no tan sols n’hi pot haver uns quants en un mateix estat i junts, sinó que busquen aquest amuntegament, el necessiten. Així, en Marc agafa com a reflex i patró aquesta classificació per postular l’existència de persones solitàries que, com els fermions, no suporten la presència de ningú. Són aquestes les úniques que li mereixen un respecte. A banda, hi ha les altres, les que, com els bosons, s’apleguen sempre que poden en associacions, grups i altres apinyaments amb la finalitat d’emmascarar en la massa la seva mediocritat genètica. Aquests últims, en Marc els menysprea, per això no és estrany que a ell no li importi com va el món, ni si hi ha pobresa o riquesa, ni si puja o baixa el preu de la fruita o el peix, ni les manifestacions, col·lectivitats, partits polítics, religions o ONG. Per descomptat, té per autèntics models de vida elevada, de vida essencialment fermiònica, Nietzsche, Wittgenstein, Unabomber, Cioran i, sobretot, Henry J. Darger, aquell home que no va sortir mai de la seva habitació de Chicago. A més, en Marc, com tots els fermions, fa temps que va deixar de freqüentar dones i amics. La seva única connexió estable amb el món és la xarxa internauta.

Malgrat el seu aïllament, a en Marc li passen algunes coses interessants, com ara la visita

del gran escriptor Julio Cortázar, a qui l’autor de la novel·la converteix en un personatge

de ficció.

4SOLUCIONARI

Com ha aconseguit en Marc convertir el conjunt d’estenedors del terrat en un sistema de coordenades cartesianes?

Imagina, ara, que en Marc ja els ha omplert tots i que, per tenir més espai, decideix posar, a mitja alçada, un altre cable a sota de cada un dels que ja hi havia. Explica com assignaries les coordenades per determinar la posició dels folis que hi pengi.

Ha numerat els cables per ordre i ha assignat una numeració, també ordenada, a les pinces

que fa servir per penjar els fulls.

Es podrien mantenir les dues coordenades que ja tenia cada foli i afegir-hi una tercera

coordenada que fos 1, si el foli estigués penjat als cables que ja hi havia, o bé 2,

si es col·loqués a la nova sèrie de filferros.

222

Geometria en l’espai

ABANS DE COMENÇAR… RECORDA

001 Donats els punts A(0, 3), B(2, 1), C( 2, 2) i D( 3, 4), troba els vectors:

a) ABW CDW b) ACW DCW c) BDW CAW

a) AB CD ( , ) ( , ) ( , )2 2 1 2 3 4W W

b) AC DC ( , ) ( , ) ( , )2 1 1 2 1 3WW

c) BD CA ( , ) ( , ) ( , )5 3 2 1 3 4W W

002 Donats Wu (2, 1) i Wv (0, 3), efectua aquestes operacions amb vectors:

a) Wu 3Wv b) 5Wu Wv c) Wu 2Wv

a) u v3 2 1 3 0 3 2 10( , ) ( , ) ( , )W W

b) 5 5 2 1 0 3 10 2u v ( , ) ( , ) ( , )W W

c) u v2 2 1 2 0 3 2 7( , ) ( , ) ( , )W W

003 Calcula aquests determinants.

a) 0 31 2

b) 1 42 8

c)

2 2 02 6 15 5 2

d)

1 1 02 1 32 4 6

a) 0 3

1 23 c)

2 2 0

2 6 1

5 5 2

4

b) 1 4

2 80 d)

1 1 0

2 1 3

2 4 6

0

004 Troba el rang d’aquestes matrius.

a) b) c)0 31 2

1 42 8

2 2 002 6 15 5 2

1 1 02 1 32 4 6

d)

a) rang 0 3

1 22 c) rang

2 2 0

2 6 1

5 5 2

3

b) rang 1 4

2 81 d) rang

1 1 0

2 1 3

2 4 6

2

223

4SOLUCIONARI

ACTIVITATS

001 Donats els vectors següents, calcula:

a) Wu Wv

b) Wv Ww

c) Wu Ww

d) Wu Wv Ww

Wv

WuWw

e) Wu Wv Ww

f ) Wu 2Wv Ww

g) (Wu Wv ) (Wv Ww )

a)

Wv Wu Wv

Ww

Wu

b)

WvWv Ww

Ww

Wu

c)

Wv

Wu Ww

Ww

Wu

d)

Wv Wu Wv

Ww

Ww

Wu

e)

Wv

Wu Wv

Ww

WwWu

224

Geometria en l’espai

Wv

Wu

2Wv Ww

Ww

Wu

f )

Wv( Wu

Wv )

(Wv Ww

)

Ww

Wu

g)

002 Demostra que l’oposat de qualsevol vector coincideix amb el vector multiplicat per 1.

Op (Wv ) 1 Wv

Dedueix que l’oposat de l’oposat d’un vector és el mateix vector: Op Op (Wu ) Wu

1 v v vOp( )W W W Op Op Op( (( ( )) ) )u u u uW W W W

003 Comprova les igualtats següents:

a) Op (Wu Wv ) Wv Wu

b) Op Wu Op Wu Op Wu Op (WWu Wv ) Wv

a) Op( ) ( )u v u v u v v uW W WWW W W W

b)

u u v u u v( ( ( ))) ( ( ))

(

Op Op Op Op Op

Op uu v u v v) ( )Op

Op Op Op Op Op Op Op( ( ( ( )))) ( ( (u u u u v u u u v uu )))W W W W

W

W

W

W W W

W

WWW

W

W

W W W

W

W

004 És cert que si tres vectors són linealment independents, dos a dos, aleshores són linealment independents si els considerem els tres alhora?

No necessàriament, perquè si són linealment independents dos a dos tenen

direccions diferents, però la combinació lineal d’un parell d’aquests vectors pot

donar com a resultat el tercer, de manera que siguin linealment dependents.

005 Comprova si els vectors l’expressió dels quals respecte de la base {Wu1, Wu2, Wu3} és (1, 0, 2), (2, 0, 1) i (1, 0, 5), són base de l’espai.

Els vectors formen una base si existeixen tres valors 1, 2 i 3 de manera que:

1 2 31 0 2 2 0 1 1 0 5 0 0 0( , , ) ( , , ) ( , , ) ( , , )

1 2 3

1 2 3

2 0

0 0

2 5 0

→ Sistema compatible indeterminat

Els vectors no són linealment independents i no formen una base.

225

4SOLUCIONARI

006 Calcula les coordenades i el mòdul d’aquests vectors:

Y

X

2

2

2

Z

WaWc

Wb

a ( , , ) ( , , ) ( , , )3 1 0 1 0 2 2 1 2W

a 2 1 2 32 2 2( ) ( )W

b ( , , ) ( , , ) ( , , )0 2 4 0 1 3 0 1 1W

b 0 1 1 22 2 2W

c ( , , ) ( , , ) ( , , )1 3 1 1 1 0 2 2 1W

c ( )2 2 1 32 2 2W

007 Donada la igualtat de vectors següent:(1, 2, 3) (3, 2, 1) (4, 4, 4)

comprova aquesta desigualtat dels seus mòduls:

( , , ) ( , , ) ( , , )1 2 3 3 2 0 4 4 4

( , , )1 2 3 1 2 3 142 2 2

( , , )3 2 0 3 2 0 132 2 2

( , , )4 4 4 4 4 4 4 32 2 2

( , , ) ( , , ) ( , , )1 2 3 3 2 0 14 13 4 3 4 4 4

008 Demostra que per a tot nombre real :

( , , ) ( , , )a b c a b c

( , , ) ( ) ( ) ( )

(

a b c a b c

a

2 2 2

2 2 b c a b c

a b c

2 2 2 2 2)

( , , )

009 Efectua les operacions amb vectors següents:Wu1 (0, 1, 1) Wu2 (1, 0, 1) Wu3 (1, 1, 0)

a) Wu1 2Wu2 3Wu3 b) Wu1 Wu2 Wu3

a) u u u1 2 32 3 0 1 1 2 1 0 1 3 1 1 0 5 4 3( , , ) ( , , ) ( , , ) ( , , ))W W W

b) u u u1 2 3 0 1 1 1 0 1 1 1 0 0 2 0( , , ) ( , , ) ( , , ) ( , , )W W W

226

Geometria en l’espai

010 Troba dos vectors Wu i Wv que verifiquin que:Wu Wv (0, 0, 1) Wu Wv (1, 0, 0)

Quants vectors en l’espai verifiquen aquestes dues condicions?

2 0 0 1 1 0 0 1 0 1

2 1

u u v u v

u

( ) ( ) ( , , ) ( , , ) ( , , )

( ,, , ) , ,0 11

20

1

2u

W W W W W

W W

2 1 0 0 0 0 1 1 0 1

2

v u v u v

v

( ) ( ) ( , , ) ( , , ) ( , , )

(11 0 11

20

1

2, , ) , ,v

W WW W W

WW

Els vectors Wu i Wv són únics.

011 Determina el nombre màxim de vectors independents i escull vectors que ho siguin.

Wv1 (1, 1, 0) WWv2 (0, 1, 1) Wv3 (3, 0, 3) Wv4 (1, 1, 1)

1 1 0

0 1 1

1 1 1

3 0

1 1 0

0 1 1

3 0 3

1 1

rang

11

3

Hi ha tres vectors linealment independents.

v v v1 2 41 1 0 0 1 1 1 1 1( , , ), ( , , ) ( , , )iW W W són vectors linealment

independents.

012 Comprova si aquestes col·lecions de vectors són base de l’espai o no.

a) Wv1 (2, 1, 0), Wv2 (2, 1, 0) i Wv3 (2, 0, 1)b) Wv1 (3, 7, 1), Wv2 ( 1, 4, 0) i Wv3 (5, 10, 2)

a)

2 1 0

2 1 0

2 0 1

4 0

2 1 0

2 1 0

2 0 1

rang 3

Els vectors formen una base.

b)

3 7 1

1 4 0

5 10 2

0

1

1

3 7

1 45 0

3 7 1

1 4 0

5 10 2

1

1rang 2

Els vectors no formen una base.

227

4SOLUCIONARI

013 Troba un punt C en el segment AB, determinat pels punts A( 3, 0, 1) i B(0, 6, 5), de manera que AC# sigui la meitat que CB#.

Si C c c c( , , )1 2 3 llavors: AC c c c( , , )1 2 33 1W i CB c c c( , , )1 2 36 5W

AC CB c c c c c c1

23 1

1

26 51 2 3 1 2 3( , , ) ( , , )Si W W

c11 1

2 2

3 3

31

2

1

26

11

25

c

c c

c c

( )

( )

c

c

c

1

2

3

2

2

7

3

C 2 27

3, ,

014 Troba un punt D, perquè el polígon ABCD sigui un paral·lelogram.A(0, 0, 0) B(2, 1, 3) C( 1, 2, 1)

Resposta oberta.

Per exemple:

Si considerem els vectors ABW i ACW, el punt D que busquem és:

D B ACW C ABW

ABW (2, 1, 3) ACW ( 1, 2, 1)

D B AC ( , , ) (2 1 3 1, 2, 1) (1, 1, 4)W

015 Calcula l’equació vectorial de la recta que passa pel punt i té el vector director indicat:a) A(2, 1, 1) i Wv ( 2, 4, 4) b) A(1, 1, 1) i Wv ( 2, 2, 2)

a) OP OA t v x y z t( , , ) ( , , ) ( , , )2 1 1 2 4 4W W W

b) OP OA t v x y z t( , , ) ( , , ) ( , , )1 1 1 2 2 2W W W

016 Troba les equacions paramètriques de la recta, si saps que un punt i un vector director són:a) A(3, 0, 7) i Wv ( 10, 2, 6) b) A(0, 0, 0) i Wv (1, 0, 0)

a)

x t

y t

z t

3 10

2

7 6

b)

x t

y

z

0

0

017 Calcula l’equació contínua de la recta que passa per cada parella de punts.a) A(2, 1, 1) i B(0, 5, 3) b) A(1, 1, 1) i B( 1, 1, 1)

a) ABx y z

( , , )2 4 42

2

1

4

1

4

W

b) ABx y z

( , , )2 2 21

2

1

2

1

2

W

228

Geometria en l’espai

018 Troba les equacions cartesianes de la recta que passa per cada parella de punts.a) A(3, 0, 7) i B( 7, 2, 1) b) A(0, 0, 0) i B(1, 0, 0)

a) ABx y z

x y

x

( , , )10 2 63

10 2

7

6

2 6 10

6 18 10 70

5 3 0

6 10 52 0z

x y

x z

W

b) ABy

z( , , )1 0 0

0

0W

019 Troba l’equació vectorial del pla en cada cas.a) A(2, 1, 1), B(0, 5, 3) i C(1, 1, 1) b) A(1, 1, 1), B( 1, 1, 1) i C(1, 2, 2)

a) ABW ( 2, 4, 4) ACW ( 1, 2, 2)

OP OA AB AC x y z( , , ) ( , , ) ( , , )2 1 1 2 4 4 ( , , )1 2 2W W W W

b) ABW ( 2, 2, 2) ACW (0, 1, 1)

OP OA AB AC x y z( , , ) ( , , ) ( , , )1 1 1 2 2 2 (( , , )0 1 1W W W W

020 Troba les equacions paramètriques del pla corresponent.a) A(3, 0, 7), Wu ( 10, 2, 6) i Wv (0, 3, 10)b) A(0, 0, 0), Wu (1, 0, 0) i Wv (4, 4, 4)

a) :

x

y

z

3 10

2 3

7 6 10

b) :

x

y

z

4

4

4

021 Troba l’equació general del pla que passa pel punt P( 1, 0, 2) i conté la recta d’equació:

rx y z

:1

1

3

14

3

El pla està definit per P( 1, 0, 2), el vector director de la recta Wv2 (1, 1, 3)

i el vector APW, amb A (1, 3, 4) r.

APW ( 2, 3, 6)

:

x y z

x y z

1 0 2

1 1 3

2 3 6

0 3 12 5 13 0:

022 Troba l’equació general del pla que passa pels punts A(1, 1, 7), B(5, 2, 9) i C(5, 4, 0).

ABW (4, 3, 16) ACW (4, 5, 7)

:

x y z

x y z

1 1 7

4 3 16

4 5 7

0 59 36 8 151 0:

229

4SOLUCIONARI

023 Calcula l’equació general dels plans que contenen dos dels eixos de coordenades.

Eix X i eix Y:

x y z

z1 0 0

0 1 0

0 0

Eix X i eix Z:

x y z

y1 0 0

0 0 1

0 0

Eix Y i eix Z:

x y z

x0 1 0

0 0 1

0 0

024 Determina la posició d’aquestes rectes:

r x y z t

sx y z

: ( , , ) ( , , ) ( , , )

:

0 5 3 1 1 1

32 2

22

rP

u:

( , , )

( , , )

0 5 3

1 1 1W

sQ

v:

( , , )

( , , )

3 0 2

2 2 2W

PQW (3, 5, 5)

rang 1 1 1

2 2 21 rang

1 1 1

2 2 2

3 5 5

2

Les rectes són paral·leles.

025 Determina les posicions relatives de les rectes següents:

r x y z t: ( , , ) ( , , ) ( , , )2 2 2 1 1 1

s x y z t: ( , , ) ( , , ) ( , , )0 0 0 1 0 0

rP

u:

( , , )

( , , )

2 2 2

1 1 1W

sQ

v:

( , , )

( , , )

0 0 0

1 0 0W

PQW ( 2, 2, 2)

rang 1 1 1

1 0 02 rang

1 1 1

1 0 0

2 2 2

2

Les rectes són secants.

230

Geometria en l’espai

026 Estudia la posició relativa d’aquestes rectes:

rx y z

x y zs

x y zx y z

: :2 0

2 02 0

2 0

1 2 1

2 1 1

1 1 1

1 0 rang

1 2 1

2 1 1

1 1 1

1 2 1

3

1 2 1 0

2 1 1 0

1 1 1 2

1 2 1 0

0 rang

1 2 1 0

2 1 1 0

1 1 1 2

1 2 1 0

3

Les rectes són secants.

027 Estudia la posició relativa d’aquestes rectes:

r

y zx z

:3 0

2 1 0

s

y zx y z

:0

3 1 0

0 1 1

2 0 1

1 3 1

3 0

0 1 1

2 0 1

0 1 1

1 3

rang

11

3

0 1 1 3

2 0 1 1

0 1 1 0

1 3 1 1

9 0 rang

0 1 1 3

2 0 1 1

0 1 1 0

1 3 1 1

4

Les rectes es creuen.

028 Calcula la posició relativa de la recta i el pla:

rx y z

x y zx z: :

2 03 1 0

1 0

1 1 1

1 3 1

1 0 1

6 0

1 1 1

1 3 1

1 0 1

rang rang

1 1 1 2

1 3 1 1

1 0 1 1

3

La recta i el pla es tallen en un punt.

231

4SOLUCIONARI

029 Troba la posició relativa de la recta i el pla:

rx y z

x y z: :1

2

21

12 5 3 3 0

rx y z

rx y

x zr

x: : :

1

2

2

1

1

2 2

1

2 yy

x z

2 0

1 0

2 1 0

1 0 1

2 5 3

11 0

2 1 0

1 0 1

2 5 3

rang rang

2 1 0 2

1 0 1 1

2 5 3 3

3

La recta i el pla es tallen en un punt.

030 Troba la posició relativa d’aquestes parelles de plans.

a) 1: x 2y z 0 2: x 2y z 1 0

b) 1: x z 11 0 2: 2y z 11 0

a) rang1 2 1

1 2 11

1 0

1 11 0

1 2 1 0

1 2 1 1rang 22

Els plans són paral·lels.

b) 1 0

0 22 0

1 0 1

0 2 1rang rang

11 0 1 11

0 2 1 112

Els plans són secants.

031 Estudia la posició relativa dels plans.

a) 1: 6x 5y 3 z 2 0 2: x y z 0

b) 1: x 2y z 1 0 2: 2x 4y 2 z 3 0

a) 6 5

1 11 0

6 5 3

1 1 1rang

66 5 3 2

1 1 1 02

Els plans són secants.

b) 1 1

2 24 0

1 2 1

2 4 2rang rrang

1 2 1 1

2 4 2 32

Els plans són secants.

232

Geometria en l’espai

032 Escriu l’equació vectorial d’un pla que sigui paral·lel al pla que passa pels punts A(0, 1, 2), B( 1, 2, 3) i C(2, 1, 4).

Quants plans hi ha que verifiquin aquesta condició?

El pla que busquem té com a vectors directors ABW ( 1, 1, 1)

i ACW (2, 2, 2), i pot passar per qualsevol punt que no pertanyi al pla

que passa per A, B i C.

El punt D (0, 0, 0) no pertany a aquest pla.

(x, y, z) (0, 0, 0) ( 1, 1, 1) (2, 2, 2)

Hi ha infinits plans paral·lels al pla que passa per A, B i C.

033 Troba l’equació del pla que passa per l’origen de coordenades i és paral·lel a les rectes:

rx y z

sx y z

: :1

1

3

14

3 1

1

1 3

rA

us

B

v:

( , , )

( , , ):

( , , )

(

1 3 4

1 1 3

0 1 0

1,, , )1 3W W ABW ( 1, 4, 4)

: :

x y z

x y z1 1 3

1 4 4

0 8 7 5 0

034 Escriu l’equació de tres plans en l’espai que es tallin a l’origen de coordenades. Comprova que verifiquen la condició de plans que es tallen en un punt.

Resposta oberta. Per exemple: 1

2

3

0

0

0

:

:

:

x y z

x y z

x y z

rang ra

1 1 1

1 1 1

1 1 1

nng

1 1 1 0

1 1 1 0

1 1 1 0

33 Secants en un punt.

035 Dóna l’equació vectorial de tres plans diferents que continguin l’eix X, i comprova que verifiquen la condició de plans concorrents en una recta.

Resposta oberta. Per exemple:

1

2

0 0 0 1 0 0 1 1 1: ( , , ) ( , , ) ( , , ) ( , , )

: ( ,

x y z

x y,, ) ( , , ) ( , , ) ( , , )

: ( , , ) (

z

x y z

0 0 0 1 0 0 2 1 1

3 00 0 0 1 0 0 1 1 2, , ) ( , , ) ( , , )

1

2

3

0

0

2 0

:

:

:

y z

y z

y z

1 1

1 12 0

0 1 1

0 1 1

0 2 1

rang rang

0 1 1 0

0 1 1 0

0 2 1 0

2

Els plans verifiquen la condició de ser secants en una recta.

233

4SOLUCIONARI

036 Determina la posició relativa d’aquests plans:

1: 6x 3y z 0

2: 3x 2y z 3 0

3: 2y z 1 0

6 3 1

3 2 1

0 2 1

3 0

6 3 1

3 2 1

0 2 1

rang rang

6 3 1 0

3 2 1 3

0 2 1 11

3

Els plans es tallen en un punt.

037 Estudia la posició relativa dels plans:

1: x z 2 0

2: 2x 2y 3z 3 0

3: 3x 8y 7z 1 0

1 0 1

2 2 3

3 8 7

20 0

1 0 1

2 2 3

3 8 7

rang rang

1 0 1 2

2 2 3 3

3 8 7 1

3

Els plans es tallen en un punt.

038 Donats els vectors Wu (3, 5, 1) i Wv (6, 4, 2), efectua les operacions següents.

a) 2Wu 3Wv

b) 5Wu 4Wv

c) Wu 2Wv

d) 2Wu Wv

a) 2 3 6 10 2 18 12 6 24 2 4u v ( , , ) ( , , ) ( , , )W W

b) 5 4 15 25 5 24 16 8 9 41 13u v ( , , ) ( , , ) ( , , )W W

c) u v2 3 5 1 12 8 4 9 13 5( , , ) ( , , ) ( , , )W W

d) 2 6 10 2 6 4 2 12 6 0u v ( , , ) ( , , ) ( , , )W W

039 Troba dos vectors Wu i Wv que verifiquin simultàniament les condicions següents:2Wu Wv ( 3, 4, 1)

3Wu 2Wv ( 8, 13, 19)

2 3 4 1

3 2 8 13 19

2 1 1u v

u v

( , , )

( , , )

W

W

W

W

2⎯→ 4 uu v

u v

u

2 6 8 2

3 2 8 13 19

7

1 1( , , )

( , , )

( , , ) ( , , )14 21 21 2 3 3u

W

W

W W

W

W

2Wu Wv ( 3, 4, 1) → Wv = ( 3, 4, 1) 2Wu

v ( , , ) ( , , ) ( , , )3 4 1 2 2 3 3 1 2 5W

234

Geometria en l’espai

040 Calcula m i n perquè es verifiqui que mWa nWb Wc , en què Wa (2, 10, 4), Wb ( 1, 3, 2) i Wc (11, 15, 2).

m n

m n

m( , , ) ( , , ) ( , , )2 10 4 1 3 2 11 15 2

2 11

10 3 15

4 2 2

n

m n

Resolem les dues primeres equacions:

16 3 33

10 3 1516 48 3 5

m n

m nm m n

Comprovem que es verifica l’última equació:

4 3 2 ( 5) 2

La solució és vàlida: m 3 i n 5.

041 Troba un vector Wt que verifiqui que 2Wu 3Wv 2Wt Ww , en què Wu (8, 1, 3), Wv (2, 0, 6) i Ww ( 6, 2, 4).

2 2 3 16 2 6 6 0 18 6 2 4 4t u v w ( , , ) ( , , ) ( , , ) ( , 00 28

2 0 14

, )

( , , )t

W

W

W W W

042 Els components de Wu , Wv i Ww en una base determinada de V3 són:

Wu (2, 0, 1) Wv ( 3, 1, 2) Ww (4, 2, 7)

Troba, en aquesta mateixa base, els components del vector:

2Wu Wv 13

Ww

(Activitat de Selectivitat)

21

34 0 2 3 1 2

1

34 2 7

25

3u v w ( , , ) ( , , ) ( , , ) ,

55

3

5

3,WWW

043 Expressa, en cada cas, el vector Ww com a combinació lineal de Wu ( 2, 3, 3) i Wv (1, 2, 5).

a) Ww ( 9, 14, 17)b) Ww (10, 16, 22)c) Ww ( 1, 1, 1)

a) ( , , ) ( , , ) ( , , )9 14 17 2 3 3 1 2 5a b

2 9

3 2 14

3 5 17

4 2a b

a b

a b

a bb

a ba a b

18

3 2 144 4 1

Comprovem a l’última equació: 3 4 5 ( 1) 17

Així doncs: Ww 4Wu Wv

235

4SOLUCIONARI

b) ( , , ) ( , , ) ( , , )10 16 22 2 3 3 1 2 5a b

2 10

3 2 16

3 5 22

4 2a b

a b

a b

a bb

a ba a b

20

3 2 164 4 2

Comprovem a l’última equació: 3 ( 4) 5 2 22

Així doncs: Ww 4Wu 2Wv

c) ( , , ) ( , , ) ( , , )1 1 1 2 3 3 1 2 5a b

2 1

3 2 1

3 5 1

4 2a b

a b

a b

a b 2

3 2 11 1 1

a ba a b

Comprovem a l’última equació: 3 1 5 1 2 1

Així doncs, Ww no es pot expressar com a combinació lineal de Wu i Wv.

044 Determina k perquè els tres vectors siguin linealment dependents.Wu (3, 2, 0) Wv (1, 3, 11) Ww (2, 4, k)

Si els tres vectors són linealment dependents:

rang

3 2 0

1 3 11

2 4

3

3

k

22 0

1 3 11

2 4

0 11 176 0 16

k

k k

045 Demostra que no es poden trobar a, b i c, diferents de zero, de manera que aWu bWv cWw 0, en què Wu ( 2, 0, 4), Wv ( 1, 3, 9) i Ww (3, 1, 7).

a b c( , , ) ( , , ) ( , , ) ( , , )2 0 4 1 3 9 3 1 7 0 0 0

2 3 0

3 0

4 9 7 0

2 1 3a b c

b c

a b c

00 3 1

4 9 7

92 0

El sistema és compatible determinat. Com que és un sistema homogeni, l’única

solució és: a b c 0

046 Demostra que els tres vectors són coplanaris.Wu (9, 4, 1) Wv ( 3, 3, 7) Ww (2, 2, 6)

9 4 1

3 3 7

2 2 6

52 0

Els vectors són coplanaris perquè són linealment dependents.

236

Geometria en l’espai

047 Decideix si els trios de vectors següents són bases de l’espai.a) Wu (5, 2, 1) Wv (4, 0, 2) Ww (3, 4, 2)b) Wu (5, 2, 3) Wv ( 2, 4, 2) Ww ( 1, 10, 9)c) Wu (2, 3, 0) Wv (0, 1, 3) Ww ( 1, 1, 1)d) Wu ( 2, 1, 0) Wv (2, 2, 3) Ww (4, 3, 3)

a)

5 2 1

4 0 2

3 4 2

4 0

Formen una base perquè són linealment independents.

b)

5 2 3

2 4 2

1 10 9

0

No formen una base perquè són linealment dependents.

c)

2 3 0

0 1 3

1 1 1

1 0

Formen una base perquè són linealment independents.

d)

2 1 0

2 2 3

4 3 3

12 0

Formen una base perquè són linealment independents.

048 Quin valor ha de tenir p perquè el vector Wu (4, p, 6) estigui al mateix pla que Wv (2, 1, 5) i Ww ( 5, 3, 13)?

Perquè el vector Wu estigui al mateix pla que Wv i Ww , cal que els tres vectors siguin

coplanaris; per tant, han de ser linealment dependents.

4 6

2 1 5

5 3 13

0 2 0 2

1

1

p

p p

049 Quina relació han de complir a i b perquè els tres vectors siguin coplanaris?Wu (a, 2, b) Wv (3, a, 5) Ww (1, b, 1)

Els vectors són coplanaris si són linealment dependents.

a b

a

b

a ab b

2

3 5

1 1

0 4 3 16 02 2

237

4SOLUCIONARI

050 Digues si l’afirmació següent és certa o falsa, i justifica la resposta amb un exemple il·lustratiu:

Si tres vectors Wu , Wv i Ww són linealment independents, aleshores també ho són els vectors: Wu Wv , Wu Wv i Wu Wv Ww.

(Activitat de Selectivitat)

L’afirmació és certa. Si tres vectors són linealment independents, el determinant

que formen les seves coordenades és diferent de 0.

Per les propietats dels determinants, si fem combinacions lineals de les files,

el valor del determinant no varia, és a dir, també és diferent de 0.

Resposta oberta. Per exemple:

u

v

w

( , , )

( , , )

( , , )

2 1 3

1 2 1

1 0 1

W

W

W

2 1 3

1 2 1

1 0 1

2 0 Són linealment independents.

u v

u v

u v w

( , , )

( , , )

( , , )

3 3 4

1 1 2

2 1 3

W

W

W

W

W

W W

3 3 4

1 1 2

2 1 3

4 0 Són linealment

independents.

051 Considera els vectors: Wu (1, 1, m) Wv (0, m, 1) Ww (1, 2m, 0)a) Determina el valor de m perquè els vectors Wu , Wv i Ww siguin linealment

dependents.b) Per al valor de m obtingut en l’apartat anterior, expressa el vector Ww

com a combinació lineal dels vectors Wu i Wv .

(Activitat de Selectivitat)

a)

1 1

0 1

1 2 0

0 2 1 0 12

m

m

m

m m m

b) ( , , ) ( , , ) ( , , )1 2 0 1 1 1 0 1 1

1

2

0

a b

a

a b

a b

b 1 → Ww Wu Wv

052 Troba un punt C en el segment AB, si saps que A( 1, 4, 3) i B(2, 10, 6), de manera que AC# sigui la meitat que CB#.

Si C c c c( , , )1 2 3 , aleshores:

AC CB c c c c c1

21 4 3

1

22 10 61 2 3 1 2( , , ) ( , , c

c c

c c

c c

3

1 1

2 2

3 3

1 11

2

4 51

2

3 31

2

)

c

c

c

1

2

3

0

6

0

C (0, 6, 0)

238

Geometria en l’espai

053 Determina els quatre punts que divideixen el segment d’extrems A(2, 6, 3) i B(12, 1, 8) en cinc parts iguals.

O

A

D

E

F

B

C

OC OA AC OA AB1

52 6 3

1

510 5 5 4 5( , , ) ( , , ) ( , , 44 4 5 4) ( , , )CW W W W W

OD OA AD OA AB2

52 6 3

2

510 5 5 6 4( , , ) ( , , ) ( , , 55 6 4 5) ( , , )DW W W W W

OE OA AE OA AB3

52 6 3

3

510 5 5 8 3( , , ) ( , , ) ( , , 66 8 3 6) ( , , )EW W W W W

OF OA AF OA AB4

52 6 3

4

510 5 5 10 2( , , ) ( , , ) ( , ,, ) ( , , )7 10 2 7FW W W W W

054 Comprova si els següents punts en l’espai estan alineats.a) A(3, 3, 5), B(4, 6, 1) i C(2, 4, 5) b) A( 4, 1, 2), B(0, 6, 1) i C( 12, 9, 4)

a) ABW (1, 9, 6) ACW ( 1, 7, 10)

Els vectors no són proporcionals; per tant, els punts no estan alineats.

b) ABW (4, 5, 1) ACW ( 8, 10, 2)

Els vectors són proporcionals; per tant, els punts estan alineats.

055 Troba els valors de a i b que fan que els tres punts estiguin alineats: P(2, 1, a), Q(5, 1, 6) i R(b, 5, 9).

Els punts estan alineats si els vectors són proporcionals.

PQ a

PR b a b

( , , )

( , , )

3 2 6

2 4 9

3

2

2

4

6

9

7

4

a

a

a

b

W

W

056 Tres vèrtexs consecutius d’un paral·lelogram són A(3, 1, 0), B(4, 5, 2) i C(4, 7, 2).a) Troba el quart vèrtex del paral·lelogram. b) Calcula’n el perímetre.

a) Considerem D (d1, d2, d3) el vèrtex que busquem.

ABW (1, 4, 2)

Aleshores: CD d d d D( , , ) ( , , )1 2 34 7 2 1 4 2 (5, 11, 0)W

b) AB 1 4 2 212 2 2W

ACW (1, 6, 2)

BC 1 6 2 412 2 2( )W

El perímetre del paral·lelogram és: 2 21 2 41

239

4SOLUCIONARI

057 Decideix si els següents grups de punts estan en el mateix pla o no. a) A(0, 1, 2), B(2, 2, 1), C(1, 0, 5) i D(1, 3, 2)b) A(0, 0, 1), B(1, 0, 2), C(0, 1, 2) i D(4, 1, 5)

a) ABW (2, 1, 1) ACW (1, 1, 3) ADW (1, 2, 4)

1 0 1

0 1 1

4 1 6

11 Els punts són coplanaris.

b) ABW (1, 0, 1) ACW (0, 1, 1) ADW (4, 1, 6)

1 0 1

0 1 1

4 1 6

11 Els punts no són coplanaris.

058 Quines condicions han de complir a, b i c perquè els punts P(3, 5, 7), Q(4, a, 3) i R(b, 7, c) estiguin alineats?

Si P, Q i R estan alineats, aleshores PQW i PRW són linealment dependents, és a dir,

són proporcionals.

PQ a

PR b c b

a( , , )

( , , )

1 5 4

3 12 7

1

3

55

12

4

7

5 3 12

5 7 48c

a b

a c

( )( )

( )( )

W

W

059 Troba l’equació de la recta que passa pels punts A(3, 1, 4) i B( 4, 3, 2).

ABW ( 7, 4, 2) r

x t

y t

z t

:

3 7

1 4

4 2

060 Determina una recta paral·lela a l’eix Y que passi pel punt (4, 2, 3).

Com que és paral·lel a l’eix Y, un vector director de la recta és (0, 1, 0).

r

x

y t

z

:

4

2

3

061 El punt ( 1, 2, 7) pertany a la recta rxyz

:2

32 3

? En cas negatiu, troba l’equació

en forma paramètrica de la recta paral·lela a r que passa per aquest punt.

1 2 1

2 3 1

7 2 3 1 El punt no pertany a la recta.

r

x

y

z

:

1

2

7 3

240

Geometria en l’espai

062 Expressa en forma contínua l’equació de la recta que passa pel punt ( 5, 4, 0) i és paral·lela a la recta r, les equacions paramètriques de la qual són:

rxyz

:1 23

2

a) El punt (1, 13, 3) és en aquesta recta? b) I el punt ( 3, 7, 2)?

rx y z

:5

2

4

3 1

a) 1 5

2

13 4

3

3

1 El punt pertany a la recta.

b) 3 5

2

7 4

3

2

1 El punt no pertany a la recta.

063 Considera els punts A( 3, 2, 9), B(1, 0, 7) i C(0, 4, 3):a) Troba l’equació de la recta que passa per A i B.b) Els tres punts estan alineats?

a) ABW (4, 2, 16)

r

x t

y t

z t

:

1 4

2

7 16

b)

0 1 41

4

4 21

4

t t

t t

C no pertany a la recta que passa per A i per B.

Els punts no estan alineats.

064 Calcula el valor que ha de tenir m perquè les rectes següents es tallin en un punt.

rx m y z

:1

10

43

0s

xyz

:16 4

1 2

rP m

v:

( , , )

( , , )

10 3

1 4 0W s

Q

u:

( , , )

( , , )

1 6 1

0 4 2W

1 4

0 44 0

1 4 0

0 4 22rang

PQW (1 m, 16, 2)

Les rectes es tallen si rang

1 4 0

0 4 2

1 16 2

2

1

1

m

..

1 4 0

0 4 2

1 16 2

0 8 32 0 4

1

1

m

m m

241

4SOLUCIONARI

065 Expressa en la forma indicada l’equació de les rectes les equacions implícites de les quals són:

a) rx y z

x y z:

3 4 23 2 4

en forma paramètrrica

b) sx y z

x y z:

2 13 2 0

en forma contínua

a) x y zx y z

y z

x t

y2 3 4

3 4 2

8 10 2

5

4

1

45

4

t

z t

b) y x zx y z

x z

x t

y1 2

2 1

5 5 3

3

511

5

3

5

1

1

5

1 1t

z t

x yz

066 Passa a forma implícita l’equació de la recta:

rx y z

:3

2

2

11

3

rx y z

rx y

x z: :

1

3

1

1

2

2

1 3 3

2 2 3 6r

x y

x z:

3 4 0

2 3 8 0

067 Calcula l’equació cartesiana de la recta l’expressió paramètrica de la qual

és rxyz

:1 312 2

Existeix algun valor de v tal que el punt (v, v, v ) pertanyi a la recta? Raona la resposta.

(Activitat de Selectivitat)

rx y z

rx y

x z: :

1

3

1

1

2

2

1 3 3

2 2 3 6r

x y

x z:

3 4 0

2 3 8 0

rx y

x z:

3 4 0

2 3 8 0 ⎯⎯→ P (v, v, v) v v v3 4 0 1

2 3 8 0

Cap punt (v, v, v) verifica les dues equacions; per tant, no existeix un valor de v

tal que P(v, v, v) r.

242

Geometria en l’espai

068 Calcula l’equació de la recta paral·lela a la recta: rx y zx y z

:0

2 1 que passsa

pel punt (0, 1, 0).(Activitat de Selectivitat)

rz x y

xr

x

y t

z t

: :3 1

1

31

3

La recta paral·lela és s:

x

y t

z t

0

1

069 Sigui r la recta definida per x y k z21 2 3

i s la recta definida per

x y z23

1

23

1 . Troba k si saps que les rectes r i s es tallen en un punt.

rP k

v:

( , , )

( , , )

2 0

1 2 3W s

Q

u:

( , , )

( , , )

2 1 3

3 2 1W PQW ( 4, 1 k, 3)

1 2

3 24 0

1 2 3

3 2 12rang

1 2 3

3 2 1

4 1 3

0 8 12 03

2k

k k

070 Determina l’equació de la recta que passa pel punt P( 2, 1, 4) i recolza en les rectes següents:

rx ty tz t

:412

sx y z

x y z:

2 4 03 2 3 0

Calculem l’equació del pla que conté el punt P i la recta r:

Q (4, 1, 2) r PQW (6, 2, 6) Wvr (1, 1, 1)

: :

x y z

x z x z

2 1 4

6 2 6

1 1 1

4 4 8 0 2 0

Calculem l’equació del pla que conté el punt P i la recta s:

sx y z

y zs

x t

y t: :2 4 0

7 5 10 0

1

5

zz t2 7

R( 1, 0, 2) s PRW (1, 1, 2)

Wvs ( 1, 5, 7)

: :

x y z

x y z x y z

2 1 4

1 1 2

1 5 7

3 9 6 9 0 3 2 33 0

La recta que busquem és m: x z

x y z

2 0

3 2 3 0

243

4SOLUCIONARI

071 Considera A i B els punts de l’espai de coordenades A(0, 1, 2), B(1, 2, 3). Troba l’equació paramètrica de la recta que passa per aquests punts.Existeixen valors de r i s per als quals el punt C de coordenades C(3, r s, r s) pertanyi a la recta que has calculat abans? En cas afirmatiu, calcula els valors de r i s. En cas negatiu, raona la resposta.

(Activitat de Selectivitat)

ABW (1, 1, 1)

m

x t

y t

z t

: 1

2

3

1

2

4

5

t

r s t

r s t

r s

r s

r

s

9

2

1

2

072 Troba l’equació contínua de la recta que passa pel punt P(1, 0, 0) i talla les rectes:

rx y z

rx y zx y z

1 22

1

1

1 2

2 1 02 3 0

: :

(Activitat de Selectivitat)

Calculem l’equació del pla que conté el punt P i la recta r1:

Q (2, 1, 0) r1 PQW (1, 1, 0) Wu (1, 1, 2)

1 1

1

1 1 0

1 1 2

2 2 2 2 0 1 0: :

x y z

x y z x y z

Calculem l’equació del pla que conté el punt P i la recta r2:

2 2

2 1 0

3 4 04 3: :

x y z

x y

x t

y t

z 7 5t

R(0, 4, 7) r2 PRW ( 1, 4, 7)

Wvr (1, 3, 5)

2 2: :

x y z

x y z x y z

1

1 4 7

1 3 5

2 1 0 2 1 0

La recta és s: x y z

x y zs

x t

y t

z

1 0

2 1 02 2

3

:

331

2

2

3

3t

sx y z

:

073 Prova que les equacions x y zx z

4 03 5 9 0

i x y z35

1

2 3

representen la mateixa recta.

(Activitat de Selectivitat)

rx y z

x zr

x t

y t: :4 0

3 5 9 0

3 5

1 2

zz t

rx y z

3

3

5

1

2 3:

Les dues rectes tenen la mateixa equació, representen la mateixa recta.

244

Geometria en l’espai

074 Considera P i Q els punts de l’espai P(1, 2, 2) i Q(2, a, a). Troba el valor de a perquè la recta que uneix P i Q passi per l’origen de coordenades.

Troba l’equació de la recta com a intersecció de dos plans i en forma paramètrica.

(Activitat de Selectivitat)

PQW (1, a 2, a 2) r

x t

y a t

z a t

:

1

2 2

2 2

( )

( )

Si la recta passa per l’origen de coordenades:

0 1

0 2 2

0 2 2

1t

a t

a t

t

a( )

( )44

Així, doncs, l’equació de la recta és:

r

x t

y t

z t

rx y

: :

1

2 2

2 2

1

1

2

2

zzr

x y

x z

2

2

2 0

2 0:

075 De totes les rectes que passen pel punt P(0, 2, 1), troba la que talla les rectes d’equacions:

(x, y, z) (1, 1, 2) t (2, 1, 0) (x, y, z) (0, 1, 1) s ( 3, 1, 2)

(Activitat de Selectivitat)

Considerem A (1 2t, 1 t, 2) un punt de la primera recta, i B ( 3s, 1 s, 1 2s)

un punt de la segona.

Si la recta que busquem passa per A, B i P, aleshores els punts estan alineats,

és a dir, APW i BPW són proporcionals.

AP t t

BP s s s

( , , )

( , , )

1 2 1 3

3 1 2 2

11 2

3

1

1

3

2 2

t

s

t

s s

WW

( )( )

( )( ) ( )

1 2 2 2 9

1 2 2 3 1

t s s

t s s

44 11 4 2

2 5 2 1

t s st

t s st

4 1 2 112 11

4 1t s s t

s

s( )

( )

2 11

2 15

2 11

2 11 5 4 02s

ss s

s

ss s

( ) ( )

ss

s t

1

47

2

( )no vàlida

Així, doncs, P(0, 2, 1) r i un vector director és BPW (12, 5, 6).

r

x t

y t

z t

:

12

2 5

1 6

245

4SOLUCIONARI

076 Troba l’equació de la recta continguda al pla : x 2y 6z 2 0 que talla els eixos Y i Z.

(Activitat de Selectivitat)

x y z

x

z

A

2 6 2 0

0

0

0 1 0( , , ) és el punt d’intersecció del pla amb l’eix Y.

x y z

x

y

B

2 6 2 0

0

0

0 01

3, , és el punt d’intersecció del pla amb l’eix Z.

Així, doncs, un vector director de la recta continguda en que talla els eixos Y i Z

és AB 0 11

3, ,W .

r

x

y t

z t

:

0

1

1

3

077 Donats els punts A( 2, 4, 3) i B(2, 6, 5), i la recta rx y z

x y z:

12 3 2

,

esbrina si existeix alguna recta tal que contingui els punts A i B i talli la recta r. Raona la resposta.

(Activitat de Selectivitat)

La recta que conté A i B és:

ABW (4, 10, 8) s

x t

y t

z t

:

2 4

6 10

5 8

Calculem un vector director de r:

rx y z

x zr

x t

y t

z

: :1

3 2 2

2

2 5

1 3tt

Wvr (2, 5, 3)

P(0, 2, 1) r APW (2, 2, 2)

Estudiem les posicions relatives de r i s.

rang4 10 8

2 5 32

4 10 8

2 5 3

2 2 2

12 0

4 10 8

2 5 3

2 2 2

1

1

1

1

rang 3

Les rectes no es tallen, es creuen.

246

Geometria en l’espai

078 Troba les equacions paramètriques del pla que passa pels punts P(3, 1, 5), Q(1, 2, 3) i R(9, 2, 2).

PQW ( 2, 3, 2) PRW (6, 1, 7)

:

x

y

z

3 2 6

1 3

5 2 7

079 Troba l’equació implícita del pla que passa pels punts P(4, 3, 1), Q(6, 2, 3) i R(2, 4, 2).

PQW (2, 1, 4) PRW ( 2, 1, 3)

: :

x y z

x y x y

4 3 1

2 1 4

2 1 3

7 14 70 0 2 10 0

080 Troba les equacions paramètriques del pla paral·lel a les rectes

rx y z

:2

3

11

2 i s

xyz

:23

1 3 que passa pel punt P (8, 9, 1).

Els vectors directors de les rectes són els vectors directors del pla.

:

x

y

z

8 2

9 3

1 2 3

081 Escriu en forma paramètrica les equacions del pla 2 x y 4z 7.

:

x

y

z

7 2 4

082 Escriu en forma implícita l’equació del pla:

xyz

1 33 25 2

: :

x y z

x y z x y z

1 3 5

1 2 0

3 1 2

4 2 7 45 0 4 2 7 45 0

247

4SOLUCIONARI

083 Escriu les equacions paramètriques dels plans cartesians (els plans que contenen dos dels eixos cartesians).

Equacions paramètriques del pla OXY :

x

y

z 0

Equacions paramètriques del pla OXZ :

x

y

z

0

Equacions paramètriques del pla OYZ :

x

y

z

0

084 Determina l’equació del pla que conté la recta rx y z

:1

1

1

35

2

i passa pel punt A(4, 0, 1).

P(1, 1, 5) r APW ( 3, 1, 6) Wvr ( 1, 3, 2)

Busquem un pla que passi per A(4, 0, 1) i que té com a vectors directors Wvr

i APW .

: :

x y z

x z x z

4 1

1 3 2

3 1 6

20 10 70 0 2 7 0

085 Troba els punts en què el pla : 2x 3y 5z 30 0, talla els eixos coordenats.

2 3 5 30 0

0

0

15

x y z

y

z

x

Talla l’eix X en el punt P(15, 0, 0).

2 3 5 30 0

0

0

10

x y z

x

z

y

Talla l’eix Y en el punt Q(0, 10, 0).

2 3 5 30 0

0

0

6

x y z

x

y

z

Talla l’eix Z en el punt R(0, 0, 6).

248

Geometria en l’espai

086 Troba les equacions del pla paral·lel al pla 2 x 2y 3z 12 0 que passa pel punt ( 2, 3, 1).

: 2 2 3 12 0

6 3

2

x y z

x

y

z

:

Els vectors Wu = (1, 1, 0) i Wv = ( 3, 0, 2) també són vectors directors del pla

que busquem.

' ': :

x y z

x y z x y

2 3 1

1 1 0

3 0 2

2 2 3 13 0 2 2 33 13 0z

087 Calcula l’equació del pla que conté la recta r : x y z12

1

23

1 i és paral·lel

a la recta sx y z

x y z:

2 4 02 3 2 4 0

.

sx y z

x y zs

x t

y: :2 4 0

2 3 2 4 0

3

4 tt

z t1 7

Wvs ( 1, 4, 7)

El punt que busquem passa pel punt P( 1, 1, 3) r, i té per vectors directors

Wvr ( 2, 2, 1) i Wvs ( 1, 4, 7).

: :

x y z

x y z x

1 1 3

2 2 1

1 4 7

18 15 6 15 0 6 5yy z2 5 0

088 Considera els punts de l’espai:

A(0, 0, 1) B(1, 1, 2) C(0, 1, 1)

a) Troba l’equació del pla ABC.

b) Si D és el punt de coordenades (k, 0, 0), quin valor ha de tenir k perquè els quatre punts A, B, C i D siguin coplanaris?

(Activitat de Selectivitat)

a) ABW (1, 1, 1) ACW (0, 1, 2)

: :

x y z

x y z x y z

1

1 1 1

0 1 2

2 1 0 2 1 0

b) Si D pertany al mateix pla, aleshores:

k 2 0 0 1 0 → k 1

249

4SOLUCIONARI

089 Calcula l’equació d’una recta que passa pel punt d’intersecció

del pla : x y z 6 0 amb la recta sx

y z:3

2 1 i que és paral·lela

a la recta rx y

x y z:

3 4 04 3 1 0

.

(Activitat de Selectivitat)

Calculem la intersecció entre i s.

x t

y t

z t

x y z

t

3

2

1

6 0

3 2 tt t t1 6 0 3

P( 9, 1, 4) és el punt d’intersecció del pla i la recta s.

Calculem un vector director de r :

rx y

x y zr

x t

y t

z

: :3 4 0

4 3 1 04 3

133 13t

Wvr (1, 3, 13)

La recta paral·lela a r que passa per P és m

x t

y t

z t

:

9

1 3

4 13

090 Troba el pla que és paral·lel a la recta: rx y z

:2

2

1

33

4 i que passa

pels punts: A( 3, 0, 1) B(2, 2, 3).

El pla passa per A i té per vectors ABW (5, 2, 4) i Wvz (2, 3, 4).

: :

x y z

x y z x y

3 1

5 2 4

2 3 4

4 28 19 7 0 4 28 119 7 0z

091 Considera els punts A(1, 1, 1), B(a, 2, b) i C(1, 0, 0).a) Amb a 2, calcula b perquè els tres punts determinin un pla que passi pel punt

P(2, 0, 1). Quina és l’equació d’aquest pla? b) Calcula els valors de a i b perquè els punts A, B i C estiguin alineats.

(Activitat de Selectivitat)

a) ABW (1, 1, b 1) ACW (0, 1, 1)

: :

x y z

b b x y z b

1

1 1 1

0 1 1

0 2 2 0( )

Si el pla conté el punt P, aleshores: ( )b b b2 2 1 2 0 3

L’equació del pla és: x y z 1 0

250

Geometria en l’espai

b) A, B i C estan alineats si els vectors ABW i ACW són proporcionals.

AB a b

AC

a

b

( , , )

( , , )

1 1 1

0 1 1

1 0

1 11

1

2

a

b

W

W

092 Calcula l’equació paramètrica i l’equació cartesiana del pla que conté els punts A, B i C de coordenades A(1, 0, 0), B(0, 1, 1) i C(1, 1, 1). Existeix algun valor de u tal que el punt (3, 2u, u 3) pertanyi al pla?Raona la resposta, i calcula el valor de u si és la resposta és afirmativa.

(Activitat de Selectivitat)

ABW ( 1, 1, 1) ACW (0, 1, 1)

: :

x

y

z

x y z1 1

1 1 1

0 1 1

0 0: y z

Si el pla conté el punt (3, 2u, u 3), aleshores: 2 3 0 3u u u

093 Donades les rectes:

rx y z

:2

1

1

2 1 s

x y z:

11

7

25

3

i el punt P(1, 1, 1), volem trobar l’equació de la recta que passa per P i que talla r i s. Per aconseguir-ho: a) Troba l’equació general o cartesiana (és a dir, l’equació de la forma

Ax By Cz D 0) del pla que conté la recta r i el punt P.b) Troba el punt M calculant el punt d’intersecció del pla amb la recta s.c) Troba l’equació de la recta que passa pels punts P i M.d) Comprova que la recta que has trobat a l’apartat anterior és

la que busquem.

(Activitat de Selectivitat)

a) Q (2, 1, 0) r PQW (1, 2, 1)

: :

x y z

y z y z

1 1 1

1 2 1

1 2 1

2 4 2 0 2 1 0

b) s

x t

y t

z t

:

1

7 2

5 3

: y z t t t M2 1 0 7 2 10 6 1 0 2 3 3 1( , , )

c) PMW (2, 4, 2) m

x t

y t

z t

:

1 2

1 4

1 2

d) La recta m passa per P, per a t 0, i pel punt M, punt d’intersecció de r i s,

per a t 1.

251

4SOLUCIONARI

094 Determina una recta que sigui paral·lela al pla que passa pels punts de coordenades (1, 1, 0), (1, 0, 1) i (0, 1, 1), que també sigui paral·lela al pla x 2y 3z 0, i que no estigui continguda en cap d’aquests plans.

(Activitat de Selectivitat)

El pla que passa per A(1, 1, 0), B(1, 0, 1) i C(0, 1, 1) és:

ABW (0, 1, 1) ACW ( 1, 0, 1)

: :

x y z

x y z x y z

1 1

0 1 1

1 0 1

2 0 2 0

La recta d’intersecció entre els dos plans és:

rx y z

x y zr

x y z

y z: :

2 0

2 3 0

2 0

2 2 0

4

2 2r

x t

y t

z t

:

La solució del problema és una recta paral·lela a r. Per exemple:

s

x t

y t

z t

:

1

1 2

1

095 Determina els extrems d’un segment AB si saps que el punt A pertany

al pla 2x y z 0, el punt B pertany a la recta x y z1

2

2

1 3

i que el punt mitjà del segment és (0, 0, 0).

(Activitat de Selectivitat)

El punt B és de la forma (1 2t, 2 t, 3t).

Si el punt mitjà del segment és (0, 0, 0), aleshores el punt A ha de ser

de la forma ( 1 2t, 2 t, 3t).

Com que aquest punt pertany al pla 2x y z 0, tenim que:

2 1 2 2 3 0 6 42

3( )t t t t t

Així, doncs, els punts són A1

3

8

32, , i B

1

3

8

32, , .

096 Troba les condicions que han de satisfer a i b perquè el punt Q(2, a, b) estigui en el mateix pla que els punts A(1, 3, 1), B(1, 0, 1) i C(0, 0, 2).

(Activitat de Selectivitat)

ABW (0, 3, 2) ACW ( 1, 3, 1)

: :

x y z

x y z x y

1 3 1

0 3 2

1 3 1

9 2 3 6 0 9 2 3 6 0z

Si Q pertany al pla, aleshores: 18 2 3 6 0 2 3 12a b a b

252

Geometria en l’espai

097 Considera la recta r x y z t t t: ( , , ) ( , , )1 2 3 , el pla : x y z2 0 i el punt P(1, 1, 1). Es demana:

a) Determina l’equació del pla 1 que passa pel punt P i és paral·lel al pla .

b) Determina l’equació del pla 2 que conté la recta r i passa pel punt P.

c) Calcula l’equació paramètrica de la recta intersecció dels plans anteriors, 1 i 2.

(Activitat de Selectivitat)

a) : :

x

y

z

x

y

z

2 1 2

11

1

1 1 1

2 1 0

1 0 1

0 21 1: :

x y z

x yy z 2 0

b) Q (1, 0, 0) r PQW (0, 1, 1) Wvr (1, 2, 3)

2 2

1 1 1

1 2 3

0 1 1

0 1 0:

x y z

x y z:

c) sx y z

x y zs

x y z

y: :

2 2 0

1 0

2 2 0

3 3 01s

x t

y

z t

:

098 Considera el punt A(3, 5, 1) i la recta: rx

yz

:1

22

14

Troba el punt B pertanyent a r tal que el vector d’extrems A i B és paral·lel al pla d’equació 3x 2y z 5 0.

(Activitat de Selectivitat)

El punt B és de la forma ( , , )1 2 2 1 4t t t .

ABW ( 2 2t, 7 t, 4t)

: :3 2 5 0

5 3 2

x y z

x

y

z

Si el vector d’extrems A i B és paral·lel a , aleshores els vectors ABW,

Wu (1, 0, 3) i Wv (0, 1, 2) són linealment dependents.

2 2 7 4

1 0 3

0 1 2

0 8 8 0 1

t t t

t t

Si substituïm t 1 a l’expressió de B obtenim B ( 1, 3, 5).

253

4SOLUCIONARI

099 Estudia les posicions relatives de les parelles de rectes següents.

a) rx y

z s xy z: :

21

23

3 24

14

2

b) rx y

z sx y z

: :3

2

2

14

14

5

21

2

c) rx y z

sx z

x y z: :

31

2

21

3

3 105 16

d) rx z

x y zs

x y zx y z

: :2 43 4

83 3 188

e) rx y z

x y zs

x y zx

: :2 1

2 2 02 2

3 yy z3 11

f ) rx y zx y z

sx y

: :2 4 7

2 21

3

1

1zz 1

2

a) rP

u:

( , , )

( , , )

2 2 3

1 3 1W s

Q

v:

( , , )

( , , )

2 4 4

1 1 2W

PQW (0, 2, 1)

1 3

1 12 0

1 3 1

1 1 2rang 22

1 3 1

1 1 2

0 2 1

0

1 3 1

1 1 2

0 2 1

rang 2

Les rectes són secants.

b) rP

u:

( , , )

( , , )

3 2 4

2 1 1Ws

Q

v:

( , , )

( , , )

1 5 1

4 2 2W

PQW ( 4, 3, 3)

rang2 1 1

4 2 21

4 2

4 34 0

2 1 1

4 2 2

4 3 3

rang 2

Les rectes són paral·leles.

c) rP

u:

( , , )

( , , )

3 2 1

1 2 3W s

x z

x y zs

x t

y t

z

: :3 10

5 1626 8

10 3t

sQ

v:

( , , )

( , , )

0 26 10

1 8 3W

PQW ( 3, 24, 9)

254

Geometria en l’espai

1 2

1 810 0

1 2 3

1 8 32rang

1 2 3

1 8 3

3 24 9

0

1 2 3

1 8 3

2

2

2

2rang

33 24 9

2

Les rectes són secants.

d)

2 0 1

1 3 1

1 1 1

10 0

2 0 1

1 3 1

1 1 1

3 1 3

rang 3

2 0 1 4

1 3 1 4

1 1 1 8

3 1 3 18

180 0

2 0 11

1

1

1

rang

44

1 3 1 4

1 1 1 8

3 1 3 18

1

14

Les rectes es creuen.

e)

2 1 1

1 2 2

1 1 2

3 0

2 1 1

1 2 2

1 1 2rang

3 1 3

3

2 1 1 1

1 2 2 0

1 1 2 2

3 1 3 11

0

2 1

rang

1 1

1 2 2 0

1 1 2 2

3 1 3 11

3

Les rectes són secants.

f ) rx y z

y zr

x t

y t

z t

: :2 4 7

6 3 3

4 3

1 2

rP

u:

( , , )

( , , )

4 0 1

3 1 2W s

Q

v:

( , , )

( , , )

1 1 1

3 1 2W

PQW ( 3, 1, 2)

rang3 1 2

3 1 21

rang

3 1 2

3 1 2

3 1 2

1

Les rectes són coincidents.

255

4SOLUCIONARI

100 Decideix si les dues rectes es tallen, i en el cas que es tallin, calcula el pla que les conté.

rx y z s

xyz

: :32

1

21

1

3 21

rP

u:

( , , )

( , , )

3 1 1

2 2 1W s

Q

v:

( , , )

( , , )

3 1 0

2 1 1W

PQW (6, 0, 1)

2 2

2 16 0

2 2 1

2 1 12rang

2 2 1

2 1 1

6 0 1

0

2 2 1

2 1 1

6 0 1

rang 2

Les rectes són secants.

El pla que les conté és:

: :

x y z

x y z x y

3 1 1

2 2 1

2 1 1

4 6 1 0 4 6 zz 1 0

101 Digues si les dues rectes són paral·leles o no.

rx y zx y z

sxyz

: :2 13

2 7 16

1 32

En cas afirmatiu, determina l’equació del pla que les conté.

rx y z

y zr

x t

y t

z t

: :2 13

2

9 3

2

rP

u:

( , , )

( , , )

9 2 0

3 1 1W s

Q

v:

( , , )

( , , )

1 2 0

3 1 1W

PQW ( 8, 0, 0)

rang rang3 1 1

3 1 11

3 1 1

3 11 1

8 0 0

2

Les rectes són paral·leles.

El pla que les conté és:

: :

x y z

y z y z

1 2

3 1 1

8 0 0

8 8 16 0 2 0

256

Geometria en l’espai

102 Decideix si aquestes dues rectes es tallen i, en cas afirmatiu, determina’n el punt de tall.

rxyz

sxyz

: :4 7

21

110 43 2

rP

u:

( , , )

( , , )

4 0 1

7 2 0W s

Q

v:

( , , )

( , , )

1 10 3

1 4 2W PQW (5, 10, 4)

7 2

1 430 0

7 2 0

1 4 22rang

7 2 0

1 4 2

5 10 4

0

7 2 0

1 4 2

5 10 4

1

1

1

1rang 2

Les rectes són secants.

4 7 1

2 10 4

1 3 2

1

2

t

t

tt

P ( , , )3 2 1 és el punt de tall.

103 Decideix les posicions relatives de les següents parelles formades per un pla i una recta.

a) rxyz

x y z: :32

1 32 3 2 0

b) rx y z

x y zx y z: :

2 3 3 05 5 7 1 0

3 55 0

c) rx y zx y z

x: :

4 2 3 02 4 4 0

1yz

2 23

a) r

x z

y

rx z

y: :

3

1

1

3

2

3 9 1

2r

x z

y:3 8 0

2 0

3 0 1

0 1 0

2 3 1

1 0

3 0 1

0 1 0

2 3 1

rang 3

rang

3 0 1 8

0 1 0 2

2 3 1 2

3

La recta i el pla es tallen en un punt.

257

4SOLUCIONARI

b)

2 1 3

5 5 7

1 3 1

0 2 1

5 55 0

2 1 3

5 5 7

1 3 1

rang 2

2 1 3

5 5 1

1 3 5

0

2 1 3 3

5 5 7 1

1 3 1 5

rang 2

La recta està continguda en el pla.

c) : :

x y z

x y z x y z

1 2 3

1 2 0

1 1 1

2 3 0 2 3 0

2 1 1

4 1 2

2 4 1

0 2 1

4 16 0

2 1 1

4 1 2

2 4 1

rang 2

2 1 3

4 1 3

2 4 4

60 0

2 1 1 3

4 1 2 3

2 4

rang

11 4

3

La recta i el pla són paral·lels.

104 Estudia les posicions relatives de les parelles de plans:

a) : :xyz

2 21 2 3

1 5' 55 5 4 12 0x y z

b) : :x y z x y z2 4 1 0 2 5 3 2 0'

c) : :xyz

x3 2 31 3 4

' y z 2 0

a) : :

x y z

x y z x y

2 1 1

2 2 0

1 3 5

10 10 8 22 0 5 5 44 11 0z

rang5 5 4

5 5 41

5 11

5 12115 0

5 5 4 11

5 5 4 12rang 2

Els plans són paral·lels.

258

Geometria en l’espai

b) 1 2

2 59 0

1 2 4

2 5 32rang

rang11 2 4 1

2 5 3 22

Els plans són secants.

c) : :

x y z

x y z x y z

3 1

1 2 3

1 3 4

0 2 0

Les equacions de i ’ són iguals, és a dir, els plans són coincidents.

Si ho fem per rangs:

rang

rang

1 1 1

1 1 11

1 1 1 2

1 1 1 21

Els plans són coincidents.

105 Estudia les posicions relatives dels grups de plans següents:

a) :::

4 3 2 03 5 4 7 0

3 6 0

x y zx y z

y z'''

b) :::

x y zx y zx y z

2 3 1 02 5 07 4 7 7

''' 00

c) :::

6 3 9 1 02 1 0

4 2 6

x y zx y zx y z

''' 55 0

d) :::

2 3 02 4 5 06 5 9 1 0

x y zx y zx y z

'''

a)

4 1 3

3 5 4

0 1 3

44 0

4 1 3

3 5 4

0 1 3

rang 3

4 1 3 2

3 5 4 7

0 1 3 6

rang 3

Els plans es tallen en un punt.

259

4SOLUCIONARI

b)

1 2 3

2 1 1

7 4 7

0

1 2

2 15 0

1 2 3

2 1 1

7 4 7

rang 2

1 2 1

2 1 5

7 4 7

0

1 2 3 1

2 1 1 5

7 4 7 7

rang 2

Com que no hi ha dos plans coincidents, els tres plans es tallen en una recta.

c)

6 3 9

1 2 1

4 2 6

0

6 3

1 29 0

6 3 9

1 2 1

4 2 6

rang 2

6 3 1

1 2 1

4 2 5

51 0

6 3 9 1

1 2 1rang 11

4 2 6 5

3

Com que i ’ són paral·lels:

6

4

3

2

9

6

1

7

Tenim dos plans paral·lels que tallen el tercer.

d)

2 3 1

2 1 4

6 5 9

0

2 3

2 14 0

2 3 1

2 1 4

6 5 9

rang 2

2 3 0

2 1 5

6 5 1

44 0

2 3 1 0

2 1 4 5

6 5 9

rang

11

3

Com que no hi ha plans paral·lels, els plans es tallen dos a dos.

260

Geometria en l’espai

106 Determina la posició relativa de les rectes següents:

rx y z

x zr

x y z1 2

7 5 7 12 02 3 11 0

5 5: :

16 03 2 7 0x y

(Activitat de Selectivitat)

7 5 7

2 0 3

5 5 1

260 0

7 5 7

2 0 3

5 5 1

3

rang

22 0

3

7 5 7 12

2 0 3 11

5 5 1 16

3 2 0 7

552 0

1

rangg

7 5 7 12

2 0 3 11

5 5 1 16

3 2 0 7

4

Les rectes es creuen.

107 Determina si el pla : 2 3 4 0x y talla o no el segment d’extrems A(2,1, 3) i B(3, 2, 1).

(Activitat de Selectivitat)

Calculem l’equació de la recta que passa per A i per B.

ABW (1, 1, 2)

r

x t

y t

z t

rx y z

: :

2

1

3 2

2

1

1

1

3

2

2 1

2 4 3

1 0

2r

x y

x zr

x y

x: :

z 7 0

1 1 0

2 0 1

2 3 0

5 0

1 1 0

2 0 1

2 3 0

rang 3

1 1 0 1

2 0 1 7

2 3 0 4

rang 3

La recta i el pla es tallen en un punt; en calculem la intersecció.

2 2 3 1 4 03

5

7

5

2

5

21

5( ) ( ) , ,t t t P

Aquest punt no està situat entre A i B; per tant, el pla no talla el segment.

261

4SOLUCIONARI

108 Considera r la recta d’equació x y z52

2

1 4 i la recta s donada

per 3 2 2

2 3 2x y z

x y z.

a) Determina la posició relativa de totes dues rectes.b) Troba l’equació del pla que conté la recta r i és paral·lel a la recta s.

(Activitat de Selectivitat)

a) sx y z

y zs

x t

y t

z t

:3 2 2

2 2

2

2 2: sQ

v:

( , , )

( , , )

2 2 0

1 2 1W

rP

u:

( , , )

( , , )

5 2 0

2 1 4W

PQW ( 3, 4, 0)

2 1

1 25 0

2 1 4

1 2 12rang

2 1 4

1 2 1

3 4 0

35 0

2 1 4

1 2 1

3 4 0

rang 3

Les rectes es creuen.

b) : :

x y z

x y z

5 2

2 1 4

1 2 1

0 9 2 5 49 0

109 Considera r i s les rectes donades per:

rx y m

z ys

x yx z

: :2

2 32

2 3

a) Troba m perquè les dues rectes es tallin.b) Per a m 1, troba l’equació del pla que conté les rectes r i s.

(Activitat de Selectivitat)

a) El rang de la matriu de coeficients i de l’ampliada ha de ser 3.

2 1 0

0 2 1

1 1 0

3 0

2 1 0

0 2 1

1 1 0

1 0 2

rang 3

2 1 0

0 2 1 3

1 1 0 2

1 0 2 3

5 5

2 1 0

0 2 1 3

1 1

m

m

m

rang00 2

1 0 2 3

3 1m

262

Geometria en l’espai

b) rx y

z yr

x t

y t

z t

: :2 1

2 31 2

5 4

sx y

x zs

x t

y t

z t

: :2

2 3

3 2

1 2

El pla que busquem passa pel punt A (0, 1, 5) r i té com a vectors directors

els vectors directors de les rectes r i s.

: :

x y z

x y z

1 5

1 2 4

2 2 1

0 10 7 6 23 0

110 Estudia, segons els valors del paràmetre k, la posició relativa de les rectes següents:

x ky

kz x

k

yz

1

2 1 2 1

2

12

(Activitat de Selectivitat)

rP k

u k:

( , , )

( , , )

1 0

1 2 1 2W s

Q

v k:

( , , )

( , , )

0 2 2

1 1 1W

PQW ( k, 3, 2)

1 2 1

1 12 1

k

kk k( )

1 2

1 12 1

kk

2 1 2

1 12 1

kk

1 2 1 2

1 1 1

3 2

2 7 32

k

k

k

k k 1 1

3 21

2 7 3 0

3

1

2

2k k

k

k

Si k R 31

2, : el rang de la matriu formada per Wu , Wv i PQW és 3

i el de la matriu determinada per Wu i Wv és 2; per tant, les rectes es creuen.

Si k 3: els rangs de les dues matrius valen 2; per tant, les rectes són secants.

Si k1

2: el rang de la matriu formada per Wu , Wv i PQW és 2, i el de la matriu

determinada per Wu i Wv és 1; així, doncs, les rectes són paral·leles.

263

4SOLUCIONARI

111 Considera les rectes: rx y z

sxyz

: :1

1

12

3

13 21

a) Estudia’n la posició relativa.b) Calcula l’equació del pla que conté la recta r i és paral·lel a la recta s.

(Activitat de Selectivitat)

a) rP

u:

( , , )

( , , )

0 1 2

1 1 3W s

Q

v:

( , , )

( , , )

1 3 1

1 2 1W

PQW (1, 2, 1)

1 1

1 23 0

1 1 3

1 2 12rang

1 1 3

1 2 1

1 2 1

6 0

1 1 3

1 2 1

1 2 1

rang 3

Les rectes es creuen.

b) : :

x y z

x y z

1 2

1 1 3

1 2 1

0 5 4 3 2 0

112 Donades les rectes següents: x a y z2

1

11

2

x y zx z

02 1

calcula el valor de a de manera que les dues rectes es tallin. Determina’n el punt de tall.

(Activitat de Selectivitat)

sx y z

x zs

x t

y t

z t

: :0

2 11 3

1 2

sQ

v:

( , , )

( , , )

0 1 1

1 3 2W

rP a

u:

( , , )

( , , )

1 1

2 1 2W PQW ( a, 2, 2)

2 1

1 37 0

2 1 2

1 3 22rang

Les rectes es tallen si la matriu formada per Wu , Wv i PQW té rang 2.

2 1 2

1 3 2

2 2

0 10 8 05

4a

a a

5

42

1 1 3

1 2 1 2

t

t

t

1

4

3

4

3

4

5

4

1

2t

P , , és el punt de tall.

264

Geometria en l’espai

113 Considera r1 i r2 les rectes d’equacions: rx z

y zr

x y zy z

1 22 1 0

2 01 0

2 2: :

a 0Determina el valor de a perquè r1 i r2 siguin coplanàries.

(Activitat de Selectivitat)

Les rectes són coplanàries si no es creuen. Així, doncs, la matriu formada

per les dues rectes no pot tenir rang 4.

1 0 2 1

0 1 1 2

1 1 1 1

0 2 2

4 16 0 4

a

a a

114 Considera la recta rx ty tz t

:22 23 3

, el pla : 2 4 2 0x y z i el punt P(1, 1, 1).

a) Determina l’equació del pla 1 que passa pel punt P i és paral·lel al pla .

b) Determina l’equació general del pla 2 que conté la recta r i passa pel punt P.

(Activitat de Selectivitat)

a) :

x

y

z 2

1 1

1 1 1

1 0 1

0 1 2

0 2 2 0: :

x y z

x y z

b) rQ

u:

( , , )

( , , )

2 2 3

1 2 3W

PQW (1, 1, 2)

2 2

1 1 1

1 2 3

1 1 2

0 1 0: :

x y z

x y z

115 Calcula a perquè la recta: rx y zx y z

:5 0

4sigui paral·lela al pla

: ax y z6 4 5.

(Activitat de Selectivitat)

1 1 0

1 1 4

6 4 5

18 0

5 1 1 0

1 1 1 4rang

a 6 4 5

3

La recta és paral·lela al pla si la matriu de coeficients té rang 2.

5 1 1

1 1 1

6 4

2 52 0 26

a

a a

265

4SOLUCIONARI

116 Considera el pla d’equació 2x 3y 4z a i r la recta que conté el punt P(1, 1, 1) el vector de direcció de la qual és Wv (1, 2, 2). Existeix algun valor de a per al qual la recta estigui continguda en el pla? Raona la resposta en cas negatiu. En cas afirmatiu, troba el valor de a.

(Activitat de Selectivitat)

rx y z

rx y

x z: :

1

1

1

2

1

2

2 1

2 1

2 1 0

2 0 1

2 3 4

0 2 1

2 02 0

2 1 0

2 0 1

2 3 4

rang 2

La recta està continguda en el pla si la matriu ampliada té rang 2.

2 1 1

2 0 1

2 3

2 2 0 1

a

a a

117 Per a quins valors del paràmetre m la recta: x ymz

111

3 és paral·lela

al pla 2x y z 9? Determina el punt d’intersecció de la recta i el pla per a m 2.

(Activitat de Selectivitat)

rx y

x mzr

x y

x mz: :

1

3 11

1

3 11

1 1 1

3 0 11

2 1 9

3 0

1 1 0 1

3 0 11

2 1 1 9

1

1

1

1

rang m 3

La recta és paral·lela al pla si la matriu de coeficients té rang 2.

1 1 0

3 0

2 1 1

3 3 0 1m m m

1 1

3 03 0

1 1 0

3 0

2 1 1

rang m 2

Calculem la intersecció per a m 2.

Si m r

x t

y t

z t

2 1

11

2

3

2

: I, substituint en el pla:

2 111

2

3

29 3 3 2 1t t t t P ( , , ) és el punt d’intersecció.

266

Geometria en l’espai

118 Estudia la posició relativa del pla : 5 2 1 0x y z

i la recta rx y

x y z:

2 12 1

segons els valors del paràmetre .

(Activitat de Selectivitat)

5 2 1

2 0 1

1 2 1

20 0

5 2 1

2 1 0 1

1 1 2

rang

1

3

2 1

1 11 0

5 2

2 1 0

1 1 2

4 8

Si 2

El rang de la matriu de coeficients és 3 i coincideix amb el de la matriu ampliada;

per tant, la recta i el pla es tallen.

Si 2

El rang de la matriu de coeficients és 2, diferent del de la matriu ampliada;

així, doncs, la recta i el pla són paral·lels.

119 Considera els plans:

1 2 3 2: x y z

2 1: x y z

a) Determina la posició relativa dels plans.

b) Calcula una recta que estigui continguda en el pla 2 1: x y z , que sigui paral·lela a la intersecció d’aquests dos plans i que passi pel punt (5, 3, 1).

(Activitat de Selectivitat)

a) 2 3

1 11 0

2 3 1

1 1 12

2 3

rang

rang11 2

1 1 1 12

Els plans són secants.

b) rx y z

x y zr

x y z

x y: :2 3 2

1

2 3 2

3 4 33

1 4

3r

x t

y t

z t

:

El punt (5, 3, 1) pertany a aquesta recta (t 1); per tant, aquesta recta

és la que compleix les condicions de l’exercici.

267

4SOLUCIONARI

120 Considera les rectes:

rx yy z

ry

x y zr1 2 3

52

16

: : :xx yy z

13

Et demanem: a) Demostra que les rectes r1 i r2 es tallen en un únic punt.b) Troba les equacions en forma contínua de la recta que passa pel punt

d’intersecció de r1 i r2 i que és paral·lela a r3.

(Activitat de Selectivitat)

a)

1 1 0

0 1 1

0 1 0

1 0

1 1 0

0 1 1

0 1 0

rang 3

1 1 0 5

0 1 1 2

0 1 0 1

1 1 1 6

0

1 1 0 5

0 1 1 2

0 1 0 1

1 1 1 6

rang 3

Les rectes r1 i r2 es tallen en un punt.

b)

x y

y z

y

x y z

P

5

2

1

6

4 1 1( , , )) és el punt d’intersecció de r1 i r2.

r

x t

y t

z t

3

4

3: La recta és sx y z

:4

1

1

1

1

1

121 Estudia si existeix algun punt que pertanyi alhora als tres plans següents. Calcula’n els punts en comú (si existeixen).

1 2 30 2111 2

: : :x y z z yxyz

(Activitat de Selectivitat)

3 3

1 1 1

1 1 2

0 1 1

3 1 0 3 1: :

x y z

x y z x y z 0

1 1 1

0 2 1

3 1 1

6 0

1 1 1

0 2 1

3 1 1

rang rang

1 1 1 0

0 2 1 0

3 1 1 1

3

Els tres plans es tallen en un punt.

268

Geometria en l’espai

x y z

y z

x y z

x y z

y

0

2 0

3 1

0

2 zz

y z

x y z

y z

z

0

2 4 1

0

2 0

3 1

x

y

z

1

6

1

6

1

3

El punt d’intersecció és: P1

6

1

6

1

3, ,

122 Discuteix, segons els valors de a, la posició relativa dels plans següents i indica les figures que determinen (no és necessari resoldre-ho).

1 2 31 3 2 4 2 2: ( ) : :a x y z x y az x ay z a

(Activitat de Selectivitat)

a

a

a

a a a

1 1 1

1 2

1 2

63 2

a a a a a a

a

a

a

3 2 26 0 6 0

0

2

3

( )

Si a R { , , }3 0 2 : el sistema format per les equacions dels tres plans

és compatible determinat, és a dir, els plans es tallen en un punt.

Si a

x y z

x y z

x y z

3

2 3

2 3 4

3 2 6

:

2 1

1 25 0 El rang de la matriu de coeficients és 2.

2 1 3

1 2 4

1 3 6

5 0 El rang de la matriu ampliada és 3.

Com que no hi ha plans paral·lels, els plans es tallen dos a dos.

Si a

x y z

x y

x z

0

3

2 4

2 0

:

1 1

1 21 0 El rang de la matriu de coeficients és 2.

1 1 3

1 2 4

1 0 0

2 0 El rang de la matriu ampliada és 3.

Com que no hi ha plans paral·lels, els plans es tallen dos a dos.

269

4SOLUCIONARI

Si a

x y z

x y z

x y z

2

3 3

2 2 4

2 2 4

:

3 1

1 25 0 El rang de les dues matrius és 2.

Dos plans coincideixen i tallen el primer en una recta.

123 Determina a i b perquè els plans:x y z 2 2x 3y z 3 ax 10y 4z b

es tallin en una recta r. Dóna algun tipus d’equacions per a r (les que vulguis).

(Activitat de Selectivitat)

1 1

2 31 0

Els plans es tallen en una recta si el rang de la matriu de coeficients

i de l’ampliada és 2. Així, doncs:

1 1 1

2 3 1

10 4

14 2 0 7

1

1

a

a a

1 1 2

2 3 3

7 10

11 0 11

1

1

b

b b

rx y z

x y zr

x t

y t

z t

: :2

2 3 3

1 2

1

rx y z

:1

2 1

1

1

124 En l’espai, considerem els tres plans d’equacions:

1 2 32 1 1 2 1: : :x y z px y pz px y zen què p és un paràmetre real. a) Esbrina per a quins valors de p els tres plans es tallen en un únic punt.

Troba aquest punt quan p 1.

b) Hi ha algun valor de p que fa que la intersecció comuna sigui una recta? Si és així, escriu l’equació vectorial d’aquesta recta.

c) Troba quina és la posició relativa dels tres plans quan p12

.

(Activitat de Selectivitat)

a) Els plans es tallen en un punt si el rang de la matriu de coeficients

i de l’ampliada és 3.

1 2 1

1

1 2

2 5 2 0

1

2

2

2p p

p

p pp

p

270

Geometria en l’espai

Si p R1

22, : el sistema és compatible determinat, és a dir,

els tres plans es tallen en un únic punt.

Si p 1:

x y z

x y z

x y z

x y z2 1

1

2 1

2 1

yy

y z

x

y

z

0

0

1

0

0

El punt d’intersecció és P(1, 0, 0).

b) Si p 2:

x y z

x y z

x y z

2 1

2 2 1

2 2 1

1 2

2 13 0

Com que la segona i la tercera equació són iguals, el rang

de les dues matrius és 2.

Un pla talla dos plans coincidents en una recta.

rx y z

x y zr

x t

y

z t

: :2 1

2 2 1

1

3

1

3

r x y z: ( , , ) , ,1

3

1

30 t ( , , )1 0 1

c) Si p

x y z

x y z

x y z

1

2

2 1

1

2

1

21

1

22 1

:

11

2

1 2

3

20 El rang de la matriu de coeficients és 2.

2 1 1

11

21

1 2 1

3

20 El rang de la matriu ampliada és 3.

Com que els dos primers plans són paral·lels, el tercer els talla en dues rectes

paral·leles.

271

4SOLUCIONARI

125 Troba l’equació del pla que passa pel punt P(3, 1, 4) i és paral·lel a les rectes:

rx y z

x y zr

x y1 2

5 3 4 02 1 0

11

2

2: :

z 23

(Activitat de Selectivitat)

rx y z

x zr

x t

y1 1

5 3 4 0

3 2 3 0

1 2

1: : t

z t3

El pla que busquem passa per (3, 1, 4) i té com a vectors directors els vectors

directors de r1 i r2.

: :

x y z

x y z x y

3 1 4

2 1 3

1 2 3

3 3 3 18 0 zz 6 0

126 Troba les equacions paramètriques d’una recta si saps que talla la recta r d’equacions x y z, és paral·lela al pla d’equació 3x 2y z 4 i passa pel punt A(1, 2, 1).

(Activitat de Selectivitat)

Determinem els vectors directors de .

: 3 2 4

4 3 2

x y z

x

y

z

Calculem l’equació del pla paral·lel a que passa per A.

' ': :

x y z

x y z

1 2 1

1 0 3

0 1 2

0 3 2 8 0

Trobem el punt d’intersecció de la recta r amb aquest pla.

r x y z

x y z x x x x

:

: 3 2 8 0 3 2 8 0 2

El punt d’intersecció del pla amb la recta r: x y z és P (2, 2, 2).

La recta que es demana passa per A i per P.

APW (1, 0, 3)

r

x t

y

z t

':1

2

1 3

272

Geometria en l’espai

127 Estudia i resol, segons els valors de , el sistema:

x y zx y

y zx y z

22

32 0

Si les dues primeres equacions representen una recta r i les dues últimes una altra recta s, interpreta geomètricament els resultats que has obtingut. (Activitat de Selectivitat)

1 1 1

2 1 0

0 1 3

7 0

1

2

0

1

1

1

1

1

1

0

3

2

rang 3

1 1 1 2

2 1 0

0 1 3

1 1 2 0

14 14 0 1

Si 1: el sistema és incompatible i les rectes es creuen.

Si 1: el sistema és compatible determinat; és a dir, les rectes es tallen

en un punt.

128 Estudia la posició relativa dels quatre plans següents:

7 8 04

2 3 5 10

x y zx y

x y zx y

(Activitat de Selectivitat)

7 8 1

1 1 0

2 3 5

70 0

7 8 1

1 1 0

2 3 5

1 1

rang

00

3

7 8 1 0

1 1 0 4

2 3 5 1

1 1 0 0

14 0

7 8 1

rang

0

1 1 0 4

2 3 5 1

1 1 0 0

4

Els quatre plans no tenen una intersecció comuna. Estudiem les posicions relatives

dels tres primers plans.

7 8 1

1 1 0

2 3 5

70

7 8 1

1 1 0

2 3 5

rang rang

7 8 1 0

1 1 0 4

2 3 5 1

3

Els tres primers plans es tallen en un punt. Com que el quart pla no és

paral·lel a cap dels altres tres, aquest pla talla en una recta cadascun

dels altres plans.

273

4SOLUCIONARI

129 Considera les rectes:

r x yz

sx y z m

: :3 45

25

2

4

1 2en què m R.a) Estudia, segons els valors del paràmetre m, les posicions relatives

de les dues rectes. En cas que les rectes r i s es tallin, calcula’n el punt de tall.b) Quan siguin coplanàries, determina l’equació general del pla que les conté.c) Estudia la posició relativa del pla de l’apartat anterior amb el pla que passa

pels tres punts: A(3, 4, 5), B (5, 4, 3) i C (1, 2, 1).Indicació: No cal construir el pla que passa per aquests tres punts.

(Activitat de Selectivitat)

a) rP

u:

( , , )

( , , )

3 4 5

1 1 2W s

Q m

v:

( , , )

( , , )

5 4

2 1 2W

PQW (2, 0, m 5)

1 1

2 11 0

1 1 2

2 1 22rang

1 1 2

2 1 2

2 0 5

3

m

m

Si m 3 0 → m 3:

El rang de la matriu de coeficients és 2 i el de l’ampliada és 3; les rectes

es creuen.

Si m 3 0 → m 3:

El rang de les dues matrius és 2; les rectes són secants.

Calculem en aquest cas la intersecció entre les dues rectes.

Per fer-ho, igualem les equacions paramètriques de les dues rectes.

3 5 2

4 4

5 2 3 2

2t

t

tt 22

1 2 1C ( , , ) és el punt de tall.

b) Les rectes són coplanàries si són secants, és a dir, si m 3.

El pla que busquem passa per P (3, 4, 5) r i té com a vectors directors

els vectors directors de r i s.

: :

x y z

x y z

3 4 5

1 1 2

2 1 2

0 4 6 7 0

c) A (3, 4, 5) r

B (5, 4, 3) r

C (1, 2, 1) 1 3 2 4 1 5 → C r

Els punts A, B i C pertanyen a les rectes r i s, per tant, també són al pla.

El pla que passa per aquests tres punts és coincident amb l’anterior.

274

Geometria en l’espai

PREPARA LA SELECTIVITAT(Activitats de Selectivitat)

1 Respon aquestes qüestions.

a) Estan alineats els punts A(1, 0, 1), B( 1, 1, 2) i C(3, 0, 1)? Justifica la resposta.

b) Si estan alineats, determina l’equació de la recta que els conté. Si no n’estan, determina l’equació del pla que passa pels tres punts.

a) ABW ( 2, 1, 3)

ACW (2, 0, 2)

Els vectors ABW i ACW no són proporcionals; per tant, els punts A, B i C

no estan alineats.

b) : :

x y z

x y z x y z

1 1

2 1 3

2 0 2

2 10 2 4 0 5 2 0

2 Troba les equacions paramètriques de la recta donada per r : 3 0

2 0x y z

x y z .

Existeix algun valor de s tal que el punt ( 3, s, s) pertanyi a la recta? Raona la resposta, tant en cas afirmatiu com negatiu.

rx y z

x zr

x t

y t

z t

: :3 0

4 3 0

3

5

4

3 3

5

4

1

5

5 4

t

s t

s t

t

s

Noo existeix cap valor de tal que ( 3, ,s s ss r) .

3 Calcula l’equació d’una recta que passa pel punt d’intersecció

del pla : x y z 6 0 amb la recta s: x3

y 2 z 1 i és paral·lela

a la recta r : 3 4 0

4 3 1 0x y

x y z.

s

x t

y t

z t

:

3

2

1

Calculem la intersecció entre s i .

: x y z t t t t6 0 3 2 1 6 0 3( )

P( 9, 1, 4) és el punt d’intersecció del pla i de la recta s.

275

4SOLUCIONARI

rx y

x y zr

x t

y t

z

: :3 4 0

4 3 1 04 3

133 13t

La recta que busquem és: r

x t

y t

z t

' :

9

1 3

4 13

4 Considera un pla : x y mz 3 i la recta r x yz

: 12

2.

a) Troba m perquè r i siguin paral·lels.

b) Troba m perquè r i siguin perpendiculars.

c) Existeix algun valor de m per al qual la recta r estigui continguda en el pla ?

rx y

x zr

x y

x z: :

1

2 2

1

2 2

1 1

2 02 0

1 1 0

2 0 1

1 1

2 2

m

m

1 1 1

2 0 2

1 1 3

8 0

a) Si 2 2m = 0 → m 1:

El rang de la matriu de coeficients és 2, i el de la matriu ampliada és 3. La recta r

i el pla són paral·lels.

b) Perquè siguin perpendiculars, el vector director de la recta Wd (1, 1, 2)

ha de ser paral·lel al vector normal del pla Wn (1, 1, m);

per tant:

1

1

1

1

2

mm 2

b) El rang de la matriu ampliada és 3 per a qualsevol valor de m; per tant,

no hi ha cap valor per al qual la recta r estigui continguda en el pla .

5 Estudia la posició relativa dels plans:

: x y 2z 2 : 2x my 2mz 2 m : mx 2y (2 m)z 0

segons els valors de m.

1 1

22

mm

1 1 2

2 2

2 2

4 42m m

m m

m m

1 1 2

2 2

2 0

4 2m m

m

m

Si m 2: el rang de la matriu de coeficients i el de la matriu ampliada és 3;

els plans es tallen en un únic punt.

Si m 2: el rang de la matriu de coeficients i el de l’ampliada és 1;

els plans són coincidents.

276

Geometria en l’espai

6 Considera les rectes rx y z k

sx y z

x z: :

11

1

1 1

33 1

i :

a) Troba el valor de k perquè les dues rectes estiguin contingudes en el mateix pla.

b) Per al valor de k obtingut en l’apartat anterior, determina l’equació general del pla que les conté.

a) rx y

x z kr

x y

x z k: :

1 1

1

0

1

Les rectes són coplanàries si no es creuen, és a dir, si la matriu ampliada

no té rang 4.

1 1 0 0

1 0 1 1

1 1 1 3

3 0 1 1

4 0 4k

k k

Si k 4, les rectes estan contingudes en el mateix pla.

b)

1 1 0

1 0 1

1 1 1

1 0

1 1 0

1 0 1

1 1 1

3 0 1

rang 3

1 1 0 0

1 0 1 5

1 1 1rang

33

3 0 1 1

3

Si k 4, les rectes es tallen en un punt.

Calculem el punt d’intersecció entre r i s.

x y

x z

x y z

x z

x

y

0

5

3

3 1

2

2

7z

El punt d’intersecció entre r i s és P( 2, 2, 7).

Determinem un vector director de s.

sx y z

x zs

x t

y t

z t

: :3

3 12 2

1 3

Wv (1, 2, 3)

El pla que busquem passa per P( 2, 2, 7) r ∩ s i té com a vectors directors

els vectors directors de r i s.

: :

x y z

x y z x y z

2 2 7

1 1 1

1 2 3

2 5 0 2 5 0

277

4SOLUCIONARI

7 Sigui r la recta definida per x y k z2

3 4 5 i s la recta definida

per x y z2

1

1

23

3.

a) Troba k si saps que les rectes r i s es tallen en un punt.

b) Determina l’equació del pla que conté les rectes r i s.

a) rP k

u:

( , , )

( , , )

2 0

3 4 5W s

Q

u:

( , , )

( , , )

2 1 3

1 2 3W PQW ( 4, 1 k, 3)

3 4

1 210 0

3 4 5

1 2 32rang

Les rectes es tallen si rang

3 4 5

1 2 3

4 1 3

2

k

.

3 4 5

1 2 3

4 1 3

8 14 04

7k

k k

b) El pla que busquem passa per Q( 2, 1, 3) s i té com a vectors directors

els vectors directors de r i s.

:

x y z

x y z x y

2 1 3

3 4 5

1 2 3

2 14 10 12 0 7 5: zz 6 0

8 Calcula el valor de a perquè la recta r: 5 0

4x y zx y z

sigui paral·lela

al pla : ax 6y 4z 5.

Perquè la recta i el pla siguin paral·lels, el rang de la matriu de coeficients

ha de ser 2, i el de la matriu ampliada, 3.

5 1

1 14 0

5 1 1

1 1 1

6 4

2 52

a

a

1 1 0

1 1 4

6 4 5

18 0

El rang de la matriu ampliada és 3 per a qualsevol valor de a. El rang de la matriu

de coeficients és 2 si es compleix que:

2a 52 0 → a 26

L I T E R AT U R A I M AT E M ÀT I Q U E S

L’estel daurat[A l’última classe de matemàtiques, abans de l’examen final, el profes-sor Novák intenta ajudar el seu alumne Vili i li demana que parli de qualsevol tema. En Vili, seguint l’explicació que inicia en Novák, tria les variacions.]

–Tracem una recta –diu en Vili assenyalant la pissarra amb la idea d’aprofitar el passeig fins a la tarima per moure els músculs.

–Com vulgui –va respondre en Novák, sorprès–. Traci la recta. Però no aquí, sinó mentalment.

–Que sigui AB.

–O CD –va proposar en Novák, contrariat, ja que veia que el noi no tenia ni idea del que estava parlant.

En Vili li seguia el corrent aferrant-se a les seves paraules com un sal-vavides.

–Doncs que sigui CD –va repetir.

–O XY –va suggerir el professor, complicant-ho encara més.

–O XY –va cedir en Vili.

–Però, què és el que pretén amb aquesta recta? –va exclamar finalment en Novák–. Per què farà servir la recta en les variacions? Definitiva-ment, no ho entenc. […]

–M’he fet un embolic –va balbucejar en Vili.

–Amb què, fill? Si fins ara no ha obert la boca!

El professor va proposar un altre exercici: traçar una perpendicular a un pla oblic.

En Vili ho repetia tot com un lloro, però quan en Novák l’interrogava, es quedava mut, desemparat, sense saber què dir.

Afligit per tanta incomprensió, en Novák es va prometre a si mateix que, per molt que li costés, aconseguiria que aquell noi ho entengués. […] En Vili mirava el professor i pensava: «Per a ell és tan fàcil.» Però en comptes de visualitzar la imatge del pla inclinat i la línia perpen-dicular, conceptes que li eren totalment aliens, només veia gestos bruscos del professor, que gesticulava com un saltimbanqui: els seus dits, els seus anells –fins i tot el de cornalina– ballaven en l’aire. Les abstraccions no eren el fort d’en Vili. Només li resultava intel·ligible la realitat més immediata, el que era visible i palpable.

DEZSÖ KOSZTOLÁNYI

Números realesProducte escalar5

279

5SOLUCIONARI

Determina l’equació de la recta perpendicular al pla horizontal z 1 que passa pel punt ( 1, 3, 2). Com pots veure, aquest problema és senzill. Però, com es faria si fos el pla inclinat 2 x y z 1?

Una recta perpendicular al pla horitzontal z 1 té per vector director Wn (0, 0, 1).

Per tant:

Punt:

Vector director:

P

n

( , , )

( , , )

1 3 2

0 0 1r

x

y

z

:

1

3

2W

Si el pla és 2x y z 1, un vector que hi és perpendicular és Wn (2, 1, 1).

Per tant:

Punt:

Vector director:

P

n

( , , )

( , , )

1 3 2

2 1 1s

x

y

z

:

1 2

3

2W

L’estel dauratDezsö Kosztolányi

Un dels protagonistes d’aquesta novel·la, que es desenvolupa en una ciutat d’Hongria, és un noi

anomenat Vili, a qui no li agraden ni les matemàtiques ni la física. Una tarda, a la seva habitació,

obre el llibre de Física i observa durant una estona aquesta fórmula:

g SrR

T g

R

r g1

42

42

2

2

2cos cos

Li van marxar les ganes de viure. «Quin sentit té això? –es va preguntar, confús–. Qui inventa aquestes coses per amargar-li la vida als alumnes?» Va sentir ràbia. Se li va fer un nus a la gola. Els estúpids nombres s’arrossegaven davant d’ell com cucs, i les lletres ho feien com si fossin larves.

Mentre esperava el professor particular, va mirar de resoldre alguns problemes

de matemàtiques:

Podia passar hores llegint-ne, però en va: «Un senyor ha comprat cinc metres de roba...», «Fa vuit anys, un pare era cent vegades més vell que el seu fill; vuit anys després, només li faltaven quatre anys per ser tres vegades més gran que el mateix fill...», «Un home ric que contracta dos jornalers...» Es fixava només en l’aspecte anecdòtic i, sense preocupar-se de les qüestions que havia de resoldre, imaginava situacions divertides amb els personatges dels problemes. Es deixava embolcallar com en un somni lent, i s’imaginava els detalls: el color de la roba, qui eren el pare i el fill, si aquell senyor tenia barba, si el noi sabia anar en bicicleta i on devia viure el ric... Però, quan arribava el moment inevitable d’heure-se-les amb els nombres, desbaratat el seu joc, es justificava argumentant: «Però, a veure, qui necessita aquesta roba? Jo, segur que no. És ben clar que el pare, el fill i el ric, tots plegats, són uns rucs i no serveixen per a res.»

A l’escena que es narra al text, el professor de Matemàtiques i Física, Antal Novák, preocupat

perquè en Vili no ha après res del que ha explicat durant el curs, intenta, sense èxit, d’ensenyar-li

alguna cosa abans de l’examen final.

280

Producte escalar

ABANS DE COMENÇAR… RECORDA

001 Si Wu (1, 1, 2), Wv (0, 2, 4) i Ww ( 7, 0, 7), troba els vectors:

a) Wu 3Wv Ww

b) Wv 13

Ww

c) 23

(Wu Wv ) 13

Ww

a) u v w3 1 1 2 3 0 2 4 7 0 7 6 7( , , ) ( , , ) ( , , ) ( , ,, )21W W W

b) v w1

30 2 4

7

30

7

3

7

3( , , ) , , , 2

19

3,W W

c)2

3

1

3

2

31 3 6

7

30

7

3( ) ( , , ) , ,u v w

5

32

19

3, ,W W W

002 Calcula el mòdul del vector ABW, si els punts són els següents:

a) A(1, 1, 1) i B( 1, 1, 1)b) A( 1, 1, 1) i B(1, 1, 1)

a) AB ( ) ( ) ( )2 2 2 12 2 32 2 2W

b) AB 2 2 2 12 2 32 2 2( )W

003 Troba un punt C en el segment AB, amb A(1, 1, 1) i B ( 1, 1, 1), de manera que AC sigui la meitat que CB.

Anomenem C(c1, c2, c3) el punt que ens demanen.

AC c c c CB c c c( , , ) ( , , )1 2 3 1 2 31 1 1 1 1 1W W

Com que AC CB c c cc c1

21 1 1

1

2

1

2

11 2 3

1 2( , , ) , ,

cc3

2

W W

Igualem coordenades:

cc

c11

111

2

1

3

cc

c22

211

2

1

3

cc

c33

311

2

1

3

El punt és C1

3

1

3

1

3, , .

281

SOLUCIONARI 5

ACTIVITATS

001 Calcula el producte escalar Wu Wv , si saps que Wu ( 1, 1, 0), Wv 2 i que l’angle que formen és 30°.

u u v u v( )1 1 0 2 2 2 30 62 2 2 cos cos °W W W W W

002 Si Wu i Wv són tals que Wu ( 1, 1, 0), Wv 2 i 30°, troba:a) ProjWu Wv b) ProjWv Wu

a) u v u v u vucos ProjW WWW W W

Com que iu u v( )1 1 2 62 2 WW W :

u v u v v vu u uProj Proj Proj6 2 3W W W W WW WWW

b) u v v u u uv v vProj Proj Proj6 26

2W W W W WWWWW

003 Si Wu Wv 2, Wu Ww 1 i Wv Ww 2, calcula aquestes operacions.

a) Wu ( Ww Wv ) c) 3 Ww ( 2Wu Wv )b) Ww (2Wu Wv ) d) (2Wu Wv ) ( Ww )

a) u v w u v u w( ) ( ) ( ) ( ) ( )1 2 1 1 3W W W WW WW

b) w u v w u w v( ) ( ) ( ) ( ) ( ) ( ) (2 2 1 2 1 1 2 0)W W W WW WW

c) 3 2 6 3 6 1 3w u v w u w v( ) ( ) ( ) ( ) ( )( ) 22 12W W W WW WW

d) ( ) ( ) ( ) ( ) ( ) ( ) ( )2 2 1 2 1u v w u w v w ( )1 2 0W WW WW W W

004 Donats els vectors Wu (0, 1, 0), Wv ( 1, 2, 0) i Ww ( 4, 1, 1), efectua les operacions:

a) Wu ( Ww Wv ) c) 3 Ww ( 2Wu Wv )b) Ww (2Wu Wv ) d) (2Wu Wv ) ( Ww )

a) u w v( ) ( , , ) ( , , )0 1 0 3 3 1 3W WW

b) w u v( ) ( , , ) ( , , )2 4 1 1 1 4 0 0W WW

c) 3 2 12 3 3 1 0 0 12w u v( ) ( , , ) ( , , )W WW

d) ( ) ( ) ( , , ) ( , , )2 1 4 0 4 1 1 0u v wW W W

005 Troba l’angle que formen aquests vectors.

a) Wu ( 2, 3, 0) Wv (0, 1, 1) b) Wu (3, 1, 2) Wv (0, 1, 1)

a) cosu v

u v

( ) ( )

( )

2 0 3 1 0 1

2 3 02 2 2 00 1 1

3

26

3

2653 96

2 2 2( )

,arccos °

W W

W W

W

282

Producte escalar

b) cosu v

u v

3 0 1 1 2 1

3 1 2 02 2 2 2

( ) ( )

(( ) ( )

,

1 1

3

28

3

28124 54

2 2

arccos °

W W

W W

006 Discuteix, amb l’ajuda d’un exemple, quan l’angle que formen dos vectors és igual a l’angle que formen uns altres dos vectors paral·lels a ells. Els angles poden ser diferents?

Considerem els vectors uW (1, 2, 1) i vW (2, 1, 0).

uW vW 0 0 90cos °

Dos vectors paral·lels als anteriors són, per exemple, uW1 (2, 4, 2)

i vW1 (4, 2, 0).

uW1 vW1 0 0 90cos °

L’angle que formen dos vectors sempre és igual que l’angle que formen

dos vectors diferents que hi són paral·lels.

007 Calcula els vectors perpendiculars a aquests:

a) Wu (1, 0, 0) b) Wv (1, 1, 0) c) Ww (1, 1, 1)

a) Prenem vW (v1, v2, v3).

uW (1, 0, 0) vW (v1, v2, v3) 1 0 0 01 2 3v v v

0 01 2 3v v v, ,

( , , ) ( , , ) ( , , ) ( , , ) ( , , )v v v1 2 3 0 0 0 0 0 0 1 0 ( , , )0 0 1

Els vectors (0, 1, 0) i (0, 0, 1) formen una base dels vectors

perpendiculars a uW.

Tot vector perpendicular a uW és combinació lineal d’aquests vectors.

b) Prenem uW (u1, u2, u3).

vW (1, 1, 0) uW (u1, u2, u3) 1 1 0 01 2 3u u u

1 2 1 2 3u u u u, , u

( , , ) ( , , ) ( , , ) ( , , )u u u1 2 3 1 1 0 0 0 1

Els vectors (1, 1, 0) i (0, 0, 1) formen una base dels vectors

perpendiculars a vW.

Tot vector perpendicular a vW és combinació lineal d’aquests vectors.

c) Prenem zW (z1, z2, z3).

wW (1, 1, 1) zW (z1, z2, z3) → 1 1 1 01 2 3z z z

1 2 3z z z 1 2 3z z z, ,

( , , ) ( , , ) ( , , ) ( , , )z z z1 2 3 1 1 0 1 0 1

Els vectors ( 1, 1, 0) i ( 1, 0, 1) formen una base dels vectors

perpendiculars a wW.

Tot vector perpendicular a wW és combinació lineal d’aquests vectors.

283

5SOLUCIONARI

008 Troba el vector normal al pla:

: 3 2 1x y z

Anomenem nW (n1, n2, n3) un vector genèric normal al pla .

Prenem P(0, 0, 1), Q(1, 1, 0) i R(2, 0, 5) punts pertanyents al pla.

Busquem el vector nW que compleixi que:

n PQ n PQ n n n n n n0 1 1 1 01 2 3 1 2 3( , , ) ( , , )W WW W

n PR n PR n n n n n0 2 0 6 2 6 01 2 3 1 3( , , ) ( , , )W WW W

Resolem el sistema: n n n

n n

n

n

n

1 2 3

1 3

1

2

3

0

2 6 0

3

2 n ( , , )3 2W

Un vector normal al pla serà, per exemple, quan 1 3 2 1n ( , , )W .

009 Troba els plans que passen pel punt P(2, 1, 2) i que són perpendiculars a les rectes.

a) r x y z: 22

32

13

b) sx y z

x z:

2 3 1 02 0

a) Un vector director de la recta és uW (2, 2, 3).

L’equació del pla és de la forma: 2 2 3 0x y z D

Com que passa per P D D( , , )2 1 2 2 2 2 1 3 2 0 12

El pla és : 2 2 3 12 0x y z .

b) Escrivim la recta en forma paramètrica:

x y z

x z

x

y

z

2 3 1 0

2 0

2

1

2

r

x

y

z

:

2

1

2

1

2 Un vector director és uW 2

1

21, , .

Prenem com a vector director de la recta un vector proporcional

a uW → vW ( 4, 1, 2).

L’equació del pla és de la forma: 4 2 0x y z D

Com que passa per P D D( , , )2 1 2 4 2 1 2 2 0 3

El pla és : 4 1 2 3 0x y z .

284

Producte escalar

010 Troba l’equació de la recta que passa per l’origen de coordenades i que és perpendicular al pla:

:xyz

3 3 2

2 Quin n’és el punt de tall?

Escrivim l’equació del pla en forma implícita:

: :

x y z

z z

3 2

3 1 0

2 1 0

2 0 2 0

Un vector normal al pla és nW (0, 0, 1).

La recta que busquem és l’eix Z.

r

x

y

z

:

0

0 i el punt de tall de la recta i el pla és P (0, 0, 2).

011 Troba el feix de plans secants i el feix de plans perpendiculars a la recta:

r x y z: 11

11 1

Trobem l’equació de la recta en forma implícita:

rx y

y zr

x y

y z: :

1 1

1

0

1 0

El feix de plans secants és:

( ) ( )x y y z 1 0

El feix de plans perpendiculars és:

x y z D 0

012 Calcula l’equació de la recta el feix de plans perpendiculars de la qual és x y z D 0, si saps que passa pel punt P (0, 0, 0). Quin n’és el feix de plans secants?

El vector director de la recta és vW (1, 1, 1), i com que passa pel punt

P rx y z

( , , ) :0 0 01 1 1

Calculem el feix de plans secants:

x y

y z

x y

y zx y y

0

0( ) ( z ) 0

013 Troba l’angle que formen la recta r, que passa pels punts A (1, 1, 1) i B (0, 0, 0), i la recta s : (x, y, z ) ( 1, 2, 3) (1, 2, 3).

El vector ABW ( 1, 1, 1) és un vector director de la recta r, i el vector director

de la recta s és uW (1, 2, 3).

285

5SOLUCIONARI

cos ( , )( ) ( ) ( )( )

(AB u

AB u

AB u

1 1 1 2 1 3

11 1 1 1 2 3

0

3 14

2 2 2 2 2 2) ( ) ( ) ( )

arccoss 0 90°

W

WWW

W

W

014 Calcula l’angle que formen la recta r : (x, y, z ) (0, 0, 1) (1, 0, 1) i el pla d’equació : x y 3z 2 0.

La recta té per vector director uW (1, 0, 1), i el vector normal del pla

és nW (1, 1, 3).

cos ( , )( )( )

u nu n

u n

1 1 0 1 1 3

2 11

4

22

9004

2290 31 48 58 52° ° ° °arccos , ,

WW

WW

W

W

015 Calcula l’angle que formen les parelles de plans següents.

a) : :3 2 3 0 25 0x y z x y'

b) : :x y z x y z0 2 3 2 0'

c) : :2 2 3 0 2 0y z y z'

a) El vector normal del pla és nW1 (3, 2, 1), i el vector normal del pla ' és nW2 (1, 1, 0).

cosn n

n n

1 2

1 22 2 2

3 1 2 1 1 0

3 2 1

( )( )

( ) 1 1

5

28

5

2819 07

2 2( )

,arccos °

W

W

W

W

b) El vector normal del pla és nW1 (1, 1, 1), i el vector normal del pla ' és nW2 (1, 2, 3).

cosn n

n n

1 2

1 22 2 2 2 2

1 1 1 2 1 3

1 1 1 1 2 322

6

42

6

4222 21arccos , °

W

W

W

W

c) El vector normal del pla és nW1 (0, 2, 2), i el vector normal al pla ' és nW2 (0, 1, 1).

cosn n

n n

1 2

1 22 2

0 0 2 1 2 1

2 2 1

( ) ( )

( )22 21

4

41

1 0

( )

arccos °

W

W

W

W

Els plans són paral·lels.

286

Producte escalar

016 Troba l’angle que formen els plans següents:

:

: ( , , ) (

xyz

x y z

1

' 11 1 2 1 1 1 1 0 1, , ) ( , , ) ( , , )

Calculem l’equació implícita del pla .

:

x y z

y z

1

1 1 1

1 1 1

2 2 0 Vector normal nW1 (0, 2, 2)

Calculem l’equació implícita del pla '.

':

x y z

x y z

1 1 2

1 1 1

1 0 1

2 3 0 Vector normal nW2 ( 1, 2, 1)

Calculem l’angle que formen els dos plans.

cos

a

n n

n n

1 2

1 2

0 1 2 2 2 1

8 6

2

48

( ) ( )

rrccos2

4873 22, °

W

W

W

W

017 Troba la projecció dels punts A(0, 0, 0), B( 1, 1, 1) i C (0, 2, 0) sobre la recta r.

a) r x y z: 33

22

11

b) rx z

y z:

2 30

a) Projecció del punt A.

0 3

3

0 2

2

0 1

1 A pertany a r.

La projecció ortogonal de A sobre r és A.

Projecció del punt B.

Calculem l’equació del pla perpendicular a la recta r que passa

pel punt B.

Vector normal:

Punt:

n

B

( , , )

( , , )

3 2 1

1 1 13 1 2 1 1 1 0

3 2 0

( ) ( ) ( )

:

x y z

x y z

W

Calculem el punt de tall de la recta amb el pla..

x y z

x y z

x

y

z

3

3

2

2

1

1

3 2 0

0

0

00

La projecció ortogonal de B sobre r és Q (0, 0, 0).

287

5SOLUCIONARI

Projecció del punt C.

Calculem el pla perpendicular a la recta r que passa

pel punt C.

Vector normal:

Punt:

n

C

( , , )

( , , )

3 2 1

0 2 03 0 2 2 1 0 0

3 2 4 0

( ) ( ) ( )

:

x y z

x y z

W

Calculem el punt de tall de la recta amb el pla.

x y z

x y z

x

y

3

3

2

2

1

1

3 2 4 0

6

7

44

7

2

7z

La projecció del punt C sobre r és Q6

7

4

7

2

7, , .

b) Projecció del punt A.

L’equació en forma contínua de la recta r és:

x y z3

2

0

1

0

1

Calculem l’equació del pla perpendicular a la recta r que passa

pel punt A.

Vector normal:

Punt:

n

A

( , , )

( , , )

2 1 1

0 0 02 0 1 0 1 0 0

2 0

( ) ( )( ) ( )( )

:

x y z

x y z

W

Calculem el punt de tall de la recta amb el pla.

x z

y z

x y z

x

y

z

2 3

2 0

2 0

9

7

12

7

66

7

La projecció de A sobre la recta r és Q9

7

12

7

6

7, , .

Projecció del punt B.

Calculem l’equació del pla perpendicular a la recta r que passa

pel punt B.

Vector normal:

Punt:

n

B

( , , )

( , , )

2 1 1

1 1 12 1 1 1 1 1 0

2

( ) ( )( ) ( )( )

:

x y z

x y z 44 0

W

288

Producte escalar

Calculem el punt de tall de la recta amb el pla.

x z

y z

x y z

x

y

2 3

2 0

2 4 0

1

7

20

7

zz10

7

La projecció de B sobre la recta r és Q1

7

20

7

10

7, , .

Projecció del punt C.

Calculem l’equació del pla perpendicular a la recta r que passa

pel punt C.

Vector normal:

Punt:

n

C

( , , )

( , , )

2 1 1

0 2 02 0 1 2 1 0 0

2 2

( ) ( )( ) ( )( )

:

x y z

x y z 0

W

Calculem el punt de tall de la recta amb el pla.

x z

y z

x y z

x

y

2 3

2 0

2 2 0

5

7

16

77

8

7z

La projecció de C sobre la recta r és el punt Q5

7

16

7

8

7, , .

018 Calcula la projecció dels punts A(0, 0, 0), B( 1, 1, 1) i C(0, 2, 0) sobre el pla determinat per les rectes:

r x y z: 33

22

11

sx zy z

:2 32 0

Trobem el pla determinat per les rectes r i s.

rx y z

sx z

y zr

x

: : :3

3

2

2

1

1

2 3

2 0

3 2

2y

z

: :

x y z

y z y z

3

3 2 1

2 2 1

5 10 0 2 0

El vector normal és nW (0, 1, 2).

Projecció del punt A.

El punt A pertany a la recta r; per tant, la projecció de A sobre

el pla és A.

289

5SOLUCIONARI

Projecció del punt B.

Determinem l’equació de la recta perpendicular al pla que passa

pel punt B.

Vector normal:

Punt:

n

B

( , , )

( , , )

0 1 2

1 1 1r

x y zr

x

y z: :

1

0

1

1

1

2

1

2 3

W

Calculem el punt de tall de la recta amb el pla.

x

y z

y z

x

y

z

1

2 3

2 0

1

6

5

3

5

La projecció del punt B sobre el pla és Q 16

5

3

5, , .

Projecció del punt C.

Calculem l’equació de la recta perpendicular al pla que passa

pel punt C.

Vector normal:

Punt:

n

C

( , , )

( , , )

0 1 2

0 2 0r

x y zr

x

y z: :

0

0

2

1

0

2

0

2 4

W

Calculem el punt de tall d’aquesta recta amb el pla .

x

y z

y z

x

y

z

0

2 4

2 0

0

8

5

4

5

La projecció de C sobre el pla és Q 08

5

4

5, , .

019 Calcula la projecció ortogonal de la recta:

r x y z: 21

21

21

sobre el pla:

:xyz

2 33 2 2

Trobem l’equació del pla en forma implícita.

: :

x y z

x z x z

0 2 3

1 1 2

1 3 2

4 2 6 0 2 3 0

Vector director de la recta r : uW (1, 1, 1)

Vector normal del pla : nW (2, 0, 1)

Punt de r : P (2, 2, 2)

290

Producte escalar

Calculem el pla ' que passa per P i que té per vectors directors uW i nW.

' ': :

x y z

x y z x y z

2 2 2

1 1 1

2 0 1

2 8 0 2 88 0

Calculem la recta s d’intersecció entre els plans i '.

sx z

x y zs

x

y: :2 3 0

2 8 0

3

2

1

2

19

22

5

2

z

La projecció de la recta r sobre el pla és s

x

y

z

:

3

2

1

2

19

2

5

2

020 Troba el punt on es tallen les projeccions de les rectes:

rx y

x ys

xyz

: :2 2 0

1 0

81

1

sobre el pla: : x y z 2 0

Calculem la recta r' projecció de la recta r sobre el pla .

rx y

x yr

x

y

z

: :2 2 0

1 0

0

1 v Pr r( , , ) , ,0 0 1 0 1 0( )

Vector director de :

Vector normal a :

r vr ( , , )0 0 1

nn

r Pr

( , , )

( , , )

1 1 1

0 1 0Punt de :

'::

x y z

x y

1

0 0 1

1 1 1

1 0

Per tant, la projecció de r sobre és:

rx y z

x y':

2 0

1 0

Trobem la recta s' projecció de la recta s sobre el pla .

Vector director de :

Vector normal a :

s vs ( , , )1 1 1

nn

s Ps

( , , )

( , , )

1 1 1

8 1 1Punt de :

'' '': :

x y z

x z x z

8 1 1

1 1 1

1 1 1

2 2 18 0 9 00

WW

W

W

W

291

5SOLUCIONARI

Per tant, la projecció de s sobre és:

sx y z

x z':

2 0

9 0

El punt de tall de r' i s' és:

x y z

x y

x y z

x z

2 0

1 0

2 0

9 0

x

y

z

10

3

13

3

17

3

Les projeccions de r i s sobre el pla es tallen a Q10

3

13

3

17

3, , .

021 Troba el simètric de P (0, 0, 1) respecte de Q(1, 2, 1).

El punt Q és el punt mitjà del segment PP', en què P'(a, b, c).

( , , ) , ,1 2 12 2

1

2

2

4

3

a b ca

b

c

El punt simètric és P'(2, 4, 3).

022 Troba el simètric del punt P ( 1, 2, 3) respecte de la recta rx y z: 3

31

2 2.

Calculem la projecció ortogonal Q del punt P sobre r.

Vector normal:

Punt:

n

P

( , , )

( , , )

3 2 2

1 2 3

: :3 1 2 2 2 3 0 3 2 2 1 0( ) ( ) ( )x y z x y z

W

x y z

x y z

x y3

3

1

2 2

3 2 2 1 0

2 3 33

2 3 6

3 2 2 1 0

15

17

7

1x z

x y z

x

y77

24

17z

La projecció ortogonal de P sobre r és 15

17

7

17

24

17Q , , .

Trobem el punt simètric P' respecte de la projecció Q.

15

17

7

17

24

17

1

2

2

2

3

2, , , ,

a b c

a

b

c

47

17

20

17

99

17

El punt simètric de P respecte de r és P'47

17

20

17

99

17, , .

292

Producte escalar

023 Calcula els vèrtexs del triangle simètric al que formen els punts A(0, 1, 2), B( 1, 2, 0) i C (2, 0, 1).

a) Respecte de l’origen de coordenades.b) Respecte de la recta r x y z: ( , , ) ( , , ) ( , , )1 1 1 1 1 1 .

a) Calculem el simètric del punt A respecte de O (0, 0, 0).

( , , ) , ,0 0 00

2

1

2

2

2

1 2 3

1

a a aa

' ' '' 0

1

2

0 1 22

3

a

a

A'

'

'( , , )

Determinem el simètric del punt B respecte de O (0, 0, 0).

( , , ) , ,0 0 01

2

2

2

0

2

1 2 3

1

b b bb

' ' '' 1

2

0

1 2 02

3

b

b

B'

'

'( , , )

Calculem el simètric del punt C respecte de O (0, 0, 0).

( , , ) , ,0 0 02

2

0

2

1

2

1 2 3

1

c c cc

' ' '' 2

0

1

2 0 12

3

c

c

C'

'

'( , , )

El triangle simètric respecte de l’origen tindrà per vèrtexs A', B' i C'.

b) Escrivim la recta en forma contínua.

rx y z

:1

1

1

1

1

1

Punt simètric de A respecte de r.

Calculem la projecció ortogonal QA del punt A sobre r .

Vector normal:

Punt:

n

A

( , , )

( , , )

1 1 1

0 1 2: x y z 1 0

W

x y z

x y z

x y

x z

x

1

1

1

1

1

1

1 0 yy z

x

y

z

1 0

1

3

1

3

1

3

QA

1

3

1

3

1

3, ,

Determinem el punt simètric A'(a'1, a'2, a'3) respecte de la projecció QA.

1

3

1

3

1

3

0

2

1

2

2

2

1 2 3, , , ,

a a a' ' '

a

a

a

'

'

'

1

2

3

2

3

5

3

4

3

El punt simètric de A respecte de r és A'2

3

5

3

4

3, , .

293

5SOLUCIONARI

Punt simètric de B respecte de r .

Calculem la projecció ortogonal QB del punt B sobre r.

Vector normal:

Punt:

n

B

( , , )

( , , )

1 1 1

1 2 0: x y z 1 0

W

x y z

x y z

x y

x z

x

1

1

1

1

1

1

1 0 yy z

x

y

z

1 0

1

3

1

3

1

3

QB

1

3

1

3

1

3, ,

Determinem el punt simètric B'(b'1, b'2, b'3) respecte de la projecció QB.

1

3

1

3

1

3

1

2

2

2

01 2 3, , , ,

b b b' ' '

22

2

3

8

3

2

3

1

2

3

b

b

b

'

'

'

El punt simètric de B respecte de r és B'1

3

8

3

2

3, , .

Punt simètric de C respecte de r .

Calculem la projecció ortogonal QC del punt C sobre r.

Vector normal:

Punt:

n

C

( , , )

( , , )

1 1 1

2 0 1: x y z 1 0

W

x y z

x y z

x y

x z

x

1

1

1

1

1

1

1 0 yy z

x

y

z

1 0

1

3

1

3

1

3

QC

1

3

1

3

1

3, ,

Determinem el punt simètric C'(c'1, c'2, c'3) respecte de la projecció QC.

1

3

1

3

1

3

2

2

0

2

1

2

1 2 3, , , ,

c c c' ' '

c

c

c

'

'

'

1

2

3

4

3

2

3

5

3

El punt simètric de C respecte de r és C'4

3

2

3

5

3, , .

El triangle simètric respecte de la recta r tindrà per vèrtexs A', B' i C'.

294

Producte escalar

024 Troba el simètric del punt P (0, 1, 3) respecte del pla l’equació del qual és:

: x y z 2 0

Calculem la projecció ortogonal Q del punt P sobre el pla .

Vector normal:

Punt:

n

P

( , , )

( , , )

1 1 1

0 1 3r

x y z:

0

1

1

1

3

1

W

x y z

x y z

x y

x z1

1

1

3

1

2 0

1

3

xx y z

x

y Q

2 0

4

3

7

3

4

3

7

3

5

3, ,

z5

3

Calculem les coordenades del punt P' simètric de P respecte

de la projecció Q.

4

3

7

3

5

3

0

2

1

2

3

2

1 2 3, , , ,

p p p' ' '

p

p

p

'

'

'

1

2

3

8

3

11

3

1

3

El punt simètric del punt P respecte del pla és P'8

3

11

3

1

3, , .

025 Si els punts P (2, 0, 2) i P ‘(3, 1, 3) són simètrics, troba el punt, una recta i el pla respecte dels quals aquests punts són simètrics. Són únics?

Els punts P i P' són simètrics respecte del punt mitjà

del segment PP'.

Q Q2 3

2

0 1

2

2 3

2

5

2

1

2

1

2, , , ,

La recta respecte de la qual P i P' són simètrics passa pel punt Q,

i el seu vector director és perpendicular a PPW' (1, 1, 5).

Un vector perpendicular a PPW' és uW (5, 5, 2); per tant:

r

x

y

z

:

5

25

1

25

1

22

295

5SOLUCIONARI

El pla respecte del qual P i P' són simètrics passa pel punt Q,

i té per vector normal el vector PPW' (1, 1, 5).

Vector normal:

Punt:

PP

Q

' ( , , )

, ,

1 1 5

5

2

1

2

1

2

: 15

21x y

1

25

1

20 5z x y: zz

11

20

W

El pla i el punt respecte del qual P i P' són simètrics, són únics.

Tot i així, com que existeixen infinits vectors perpendiculars a PPW', hi ha infinites

rectes respecte de les quals els dos punts són simètrics.

026 Troba la distància del punt P (1, 1, 1) a la recta r x y z: 23

11 2

.

Calculem el pla que passa per P i és perpendicular a r .

Vector normal:

Punt:

n

P

( , , )

( , , )

3 1 2

1 1 1: 3( )

: 3

x y z

x y z

1 1 2 1 0

2 6 0

( ) ( )W

Determinem el punt de tall de la recta r i el pla .

x y z

x y z

x y2

3

1

1 2

3 2 6 0

3 5

22 3 4

3 2 6 0

5

14

25

14x z

x y z

x

y

zz11

7

La projecció ortogonal de P sobre r és Q5

14

25

14

11

7, , .

La distància del punt P a la recta r és la distància de P al punt Q.

d P Q( , )5

141

25

141

2 2

11

71

266

14

2

027 Calcula el perímetre del triangle de vèrtexs:A(0, 0, 3) B(2, 2, 2) C(2, 0, 5)

Calculem la mida dels costats del triangle.

d A B AB

d A C AC

d B

( , )

( , )

( ,

2 2 5 33

2 8 68

2 2 2

2 2

CC BC) ( )2 3 132 2

W

W

W

El perímetre del triangle és:

P 33 68 13

296

Producte escalar

028 Calcula l’àrea del triangle que formen els punts:A(2, 0, 0) B( 1, 3, 2) C(1, 4, 1)

Pren, per exemple, com a base el costat AB i l’altura serà la distància del vèrtex C a la recta que determinen els punts A i B.

La base és: d A B AB( , ) ( )3 3 2 222 2 2W

La recta que determinen els punts A i B és rx y z

:2

3 3 2

Calculem el pla que passa per C i és perpendicular a r .

Vector normal:

Punt:

n

C

( , , )

( , , )

3 3 2

1 4 1:

:

3 1 3 4 2 1 0

3 3 2 1

( ) ( ) ( )x y z

x y z 77 0

W

Determinem el punt de tall de la recta r i el pla .

x y z

x y z

x y2

3 3 2

3 3 2 17 0

3 3 66

2 3 4

3 3 2 17 0

7

2

3x z

x y z

x

y22

1z

La projecció ortogonal del punt C sobre la recta r és 7

2

3

21Q , , .

L’altura del triangle és la distància del punt C a la recta r.

d C r d C Q( , ) ( , )7

21

3

24

2 2

21 150

4

5 2

2( )

Per tant, l’àrea del triangle és:

Àrea 22

5 2

2

2

5 11

2

029 Troba la distància del punt P (2, 1, 0) al pla .

:xyz

23 21 2

Trobem l’equació implícita del pla .

: :

x y z

x y z x y z

2 1

1 3 1

1 2 2

4 9 0 4 9 0

d P( , )4 2 1 1 1 0 9

4 1 10

2 2 2

El punt P (2, 1, 0) pertany al pla .

297

5SOLUCIONARI

030 Calcula l’altura traçada des del vèrtex D del tetraedre determinat pels punts:A(2, 0, 0) B( 1, 3, 2) C(1, 4, 1) D (0, 0, 0)

Troba el pla determinat pels punts A, B i C, i calcula la distància del punt D a aquest pla.

Calculem el pla que determinen els punts A, B i C.

ABW ( 3, 3, 2) ACW ( 1, 4, 1)

: :

x y z

x y z x y z

2

3 3 2

1 4 1

5 5 15 10 0 3 2 0

Altura d D( , )( )

1 0 1 0 3 0 2

1 1 3

2

11

2 11

112 2 2

031 Troba la distància que hi ha entre els plans: : 2x 2y 3z 0': 4x 4y 6z 12

Els vectors normals als plans, nW (2, 2, 3) i nW' (4, 4, 6),

són proporcionals. El punt P(3, 0, 2) i P(3, 0, 2) '; per tant, els plans

són paral·lels.

d d P( , ) ( , )( )

( )' '

4 3 4 0 6 2 12

4 4 62 2 22

12

68

6 17

17

032 Calcula la distància entre la recta r x y z:1

21 1

i el pla : x 2y 3z 1 0.

El vector director de r és uW (1, 1, 1), i el vector normal del pla

és nW (1, 2, 3).

Com que uW nW 0 i P r( , , )0 2 0 i P ( , , )0 2 0 , la recta i el pla són paral·lels.

d r d P( , ) ( , )( )

1 0 2 2 3 0 1

1 2 3

3

142 2 2

33 14

14

033 Calcula la distància entre els parells de rectes següents.

a) r x y z:2 3 1

s x y z:1

13 1

b) r x y z: 11

11

11

sx yy z

:2 3 02 3 0

a) Estudiem la posició relativa de r i s.

vWr (2, 3, 1)

vWs (1, 3, 1)

P

QP Q

r

s

r s

( , , )

( , , )( , , )

0 0 0

0 1 00 1 0#$

rang

0 1 0

2 3 1

1 3 1

3 Les rectes es creuen.

:x y z x y z5 5 15 10 0 3 2 0

298

Producte escalar

Per tant, d(r, s) d(s, r), en què r és el pla que conté r i és paral·lel a s.

r r

x y z

x y z x y z: :

0 0 0

2 3 1

1 3 1

6 9 0 6 9 0

d s d Qr s r( , ) ( , )( )

( )

6 0 1 1 9 0

6 1 92 2 2

11

118

118

118

b) La forma contínua de la recta s és: x y z0

9

0

6

0

4

Estudiem la posició relativa de r i s.

vWr ( 1, 1, 1)

vWs (9, 6, 4)

P

QP Q

r

s

r s

( , , )

( , , )( , , )

1 1 1

0 0 01 1 1#$

rang

1 1 1

1 1 1

9 6 4

2

Com que vWr i vWs no són proporcionals, les rectes són secants d r s( , ) .0

034 Troba la distància entre els parells de rectes següents.

a) rx y

z:

2 2 00

sx y z

x y:

00

b) r és la recta que passa per l’origen de coordenades i pel punt P ( 1, 2, 1), i s és la recta que passa pel punt Q(1, 1, 1) i és perpendicular al pla : x 0.

a) Estudiem la posició relativa de r i s analitzant el sistema format

pels quatre plans.

A

2 1 0

0 0 1

1 1 1

1 1 0

A**

2 1 0 2

0 0 1 0

1 1 1 0

1 1 0 0

rang ( ) rang ( )A A* 3

El sistema és compatible determinat. Són rectes secants d r s( , ) .0

b) Calculem les equacions de les rectes.

POW ( , , ) :1 2 10

1

0

2

0

1r

x y z

Vector normal a : vWs (1, 0, 0) s

x

y

z

:

1

1

1

Estudiem la seva posició relativa.

vWr (1, 2, 1)

vWs (1, 0, 0)

P

QPQ

( , , )

( , , )( , , )

1 2 1

1 1 02 1 0W

299

5SOLUCIONARI

rang

2 1 0

1 2 1

1 0 0

3 Les rectes es creuen.

Per tant, d r s d s r( , ) ( , ), en què r és el pla que conté r i és paral·lel a s.

r r

x y z

y z y z: :

1 2 1

1 2 1

1 0 0

2 0 2 0

d s d Qr r( , ) ( , )( )

0 1 1 1 2 1

0 1 2

1

5

5

2 2 2 55

035 Donats Wu (3, 4, 2) i Wv (5, 1, 6), calcula:a) Wu Wv b) Wu c) Wv

a) u v 3 5 4 1 2 6 1( ) ( )W W

b) 9 16 4 29u u uW W W

c) v v v 25 1 36 62W W W

036 Troba el mòdul del vector Wu (3, 5, 2). Calcula també dos vectors unitaris que hi siguin paral·lels.

u 3 5 2 382 2 2( )W

Vectors unitaris paral·lels a uW:

uW1 1

383 5 2

3

38

5

38

2

38( , , ) , ,

uW2 1

383 5 2

3

38

5

38

2

38( , , ) , ,

037 En general, no és cert que Wa Wb Wa Wb . Comprova-ho amb Wa ( 3, 2, 2) i Wb (1, 4, 3).

a 17W b 26W a b ( , , )2 2 5 33W W

a b a b 17 26 33WW WW

038 Com han de ser dos vectors perquè es verifiqui que Wa Wb Wa Wb ?

Els vectors aW i bW han de ser paral·lels i han de tenir el mateix sentit; en cas contrari,

el mòdul de la suma és menor.

039 Comprova amb Wu ( 5, 1, 2), Wv (3, 4, 1) i Ww (2, 1, 4) que es verifica la propietat distributiva: Wu (Wv Ww ) Wu Wv Wu Ww .

v w u v w( , , ) ( ) ( ) ( )5 5 5 5 5 1 5 5 2 30W WW W W

u v u w 13 17 30( )W W WW

300

Producte escalar

040 Donats els vectors Wu (3, 1, 2), Wv ( 2, 3, 1) i Ww (7, 2, 1), efectua les operacions que siguin possibles i explica per què no es poden fer la resta d’operacions.

a) Wu Wv Ww b) Wu (Wv Ww ) c) ( Wu Wv ) Ww

a) No es pot fer. El producte escalar uW vW és un nombre i no el podem sumar

a un vector.

b) v w u v w( , , ) ( )5 1 0 16W WW W W

c) ( ) ( , , )u v w w5 35 10 5W WWW

041 Demostra que les diagonals d’un paral·lelogram només són iguals quan els seus costats són perpendiculars. Fes servir el resultat que afirma que si els costats d’un paral·lelogram són Wu i Wv, aleshores les diagonals són Wu Wv i Wu Wv.

u v u v u v u u v v2 2 2

2( ) ( )W W W WW W W W WW

u v u v u v u u v v2 2 2

2( ) ( )W W W WW W W W WW

Les diagonals són iguals si es compleix que:

2 2 4 0 0u v u v u v u vW W W WW W W W

És a dir, si els vectors són perpendiculars.

042 Els vectors Wu , Wv i Wu Wv formen un triangle. Demostra que es compleix el teorema del cosinus aplicant-hi la definició de mòdul que proporciona el producte escalar Wu Wv 2 (Wu Wv ) (Wu Wv ).

Si anomenem a, b i c els costats del triangle:

a u v b u c vW W W W

Com que u v u v u v u u v v2 2 2

2( ) ( )W W W WW W W W W W

a b bc A c2 2 22 cos

Que és l’expressió que coneixem com el teorema del cosinus.

043 Si sabem que Wu 3, Wv 5 i que els dos vectors són perpendiculars, calcula’n el producte escalar: (3Wu 2Wv ) (2Wu Wv ).

( ) ( )3 2 2 6 2 6 3 0 2 5 42 2u v u v u u u v v vWW W W WWW W W W

044 Troba, en cada cas, el valor de p perquè els vectors tinguin mòdul 7. Sempre hi ha una única solució? Raona la resposta.a) Wu (2, 3, p) b) Wv ( 5, p, 6) c) Ww ( p, 1, 6)

a) 2 3 7 49 4 9 62 2 2 2( ) p p p

b) ( )5 6 7 49 25 36 122 2 2 2p p No hi ha soluciió.

c) p p p2 2 2 21 6 7 49 1 36 12 12( )

Podem obtenir dues solucions, una o cap.

301

5SOLUCIONARI

045 Quin valor ha de tenir t perquè els vectors Wu (3, t, 5) i Wv (2, 7, t) siguin perpendiculars? I perquè siguin paral·lels?

Perquè siguin perpendiculars, el producte escalar ha de ser 0.

( , , ) ( , , )3 5 2 7 6 2 0 3t t t t

Perquè siguin paral·lels, han de ser proporcionals:

3

2 7

5t

t No té solució.

No hi ha cap valor de t per al qual els vectors siguin paral·lels.

046 Escriu un vector de mòdul 1 que sigui ortogonal al vector de coordenades (1, 2, 1).

(Activitat real de Selectivitat)

Anomenem uW (u1, u2, u3) el vector que busquem.

uW ( , , ) ( , , ) ( , , ) ( , , )u u u u u u u1 2 3 1 2 3 11 2 1 1 2 1 2uu u2 3 0

Un vector que compleix aquesta condició és uW ( 2, 1, 0), però no és unitari.

Com que u 5W , un vector unitari que compleix aquesta condició és:

vW

1

52 1 0

2

5

1

50( , , ) , ,

047 Quant ha de valer m perquè els punts A(5, m, 7), B(3, 1, 4) i C(6, 5, 4) formin un triangle rectangle amb l’angle recte a B?

Els vectors BA m BC( , , ) ( , , )2 1 3 3 6 0iW W han de ser perpendiculars.

BA BC m m6 6 6 0 2W W

048 Calcula l’equació de la recta perpendicular al pla : 2 5 8x y z que passa pel punt P(3, 3, 5).

Vector normal a :

Punt:

n

P

( , , )

( , , )

2 5 1

3 3 5r

x y z:

3

2

3

5

5

1

W

049 Troba l’equació d’un pla perpendicular a la recta r x y x: 23

1 72

i que la talli al punt P (5, 0, 9).

El vector director de la recta és el vector normal del pla:

n x y z D( , , ) :3 1 2 3 2 0W

P D D( , , ) ( )5 0 9 3 5 0 2 9 0 33

El pla és : 3 2 33 0x y z .

050 Demostra que, en qualsevol triangle, els costats verifiquen que a b c.

Considerem com a costats del triangle els vectors uW, vW i uW vW.

Suposem que no és certa la hipòtesi a b > c, és a dir, que es compleix

que u v u vW W W W .

302

Producte escalar

Com que els dos membres són positius: u v u v2 2

W W W W

u v u v u v u u v v u u v v2 2 2 2 2

2 2( ) ( ) cosW W W W WW W W W W WW WW

u v u u v v2 2 2

2 W W WW W W

Si u v u vW W W W , aleshores:

u u v v u u v v u v u v2 2 2 2

2 2 2 2cos cosW W W W WW WW W WW W

Com que cos < 1, la desigualtat no és certa.

051 Determina el pla paral·lel a 5x 3y 2z 7 0 que passa pel punt P (1, 2, 1).

Si els plans són paral·lels, tenen el mateix vector normal.

n x y z D( , , ) :5 3 2 5 3 2 0W

P D D x y z( , , ) :1 2 1 5 1 3 2 2 1 0 1 5 3 2 11 0

052 Fes, de dues maneres diferents, el problema següent. Quina posició relativa

tenen la recta rx y z

x y z:

2 3 82 2 3

i el pla : x y z3 12?

I la recta r i el pla : 3 3 6x y z ?

Posició relativa de r i .

– Primera manera: estudiem les solucions del sistema d’equacions conjunt.

A

2 1 3

1 2 2

1 1 3

A*

2 1 3

1 2 2

1 1 3

8

3

12

1

1

rang (A) rang (A*) 3 → Sistema compatible determinat.

La recta i el pla es tallen en un punt.

– Segona manera: comparem el vector director i el normal.

Calculem el vector director de la recta.

r

x

y

z

:

19

3

8

3

14

3

7

3

→ vWr ( 8, 7, 3)

Com que nW vWr 0, la recta i el pla no són paral·lels ni la recta està continguda

en el pla; per tant, es tallen.

Posició relativa de r i .

– Primera manera:

A

2 1 3

1 2 2

3 3 1

A*

2 1 3 8

1 2 2 3

3 3 1 6

rang (A) 2 rang (A*) 3 → Sistema incompatible.

La recta i el pla són paral·lels.

303

5SOLUCIONARI

– Segona manera:

v

n

r ( , , )

( , , )

8 7 3

3 3 1( 8, 7, 3) (3,, 3, 1) 0

W

W

Els vectors són perpendiculars; per tant, la recta és paral·lela al pla

o hi està continguda.

P r( , , )1 0 2 i P ( , , )1 0 2 , la recta i el pla són paral·lels.

053 Expressa en forma implícita l’equació del pla perpendicular al vector Wn (3, 1, 10) i que passa pel punt P ( 3, 3, 2). El punt Q ( 4, 2, 5) pertany a aquest pla?

El pla ha de ser de la forma : 3 10 0x y z D

P D D( , , ) ( ) ( )3 3 2 3 3 1 3 10 2 0 14

:: 3 10 14 0x y z

El punt Q compleix que: 3 4 2 10 5 14 0( ) Q

054 Amb les rectes r x y z m: ( , , ) ( , , ) ( , , )1 2 0 4 1 i s x y z: 28 2

16

, troba m perquè:a) Les rectes siguin paral·leles. En aquest cas, troba el pla que les conté.b) Les rectes siguin perpendiculars. Es tallen? Si és així, determina’n

el punt de tall.

Escrivim un punt i un vector director de cada recta.

v m P

v P

r r

s s

( , , ) ( , , )

( , , ) ( , , )

4 1 1 2 0

8 2 6 2 0 1P Pr s ( , , )1 2 1

W

W#$

a)4

8

1

2 6

m → Les rectes són paral·leles si m 3.

Calculem el pla que les conté.

Dos vectors del pla són vWs i Pr WPs i un punt Pr

( 1, 2, 0), per tant:

:

x

y

z

1 8

2 2 2

6

b) v v v v m mr s r s ( , , ) ( , , )4 1 8 2 6 32 2 6 0

m17

3

W W W W

N’estudiem la posició relativa.

rang

1 2 1

4 117

3

8 2 6

3 → Les rectes no es tallen, es creuen.

m17

3

304

Producte escalar

055 Donats els plans : mx y z2 7 0 i ': 3 4 1 0x y z , troba m perquè els plans:a) Siguin paral·lels.b) Siguin perpendiculars.

a)m

3

1

4

2

1 No hi ha cap valor per a m que compleixi aquestes igualtats.

Per tant, i ' no poden ser paral·lels.

b) n n n n m m m' ' ( , , ) ( , , )1 2 3 4 1 3 4 2 0 2W W W W

056 Considera r : ( x, y, z ) (0, 3, 3) t (0, 1, 1). Demostra que:

a) r està continguda en el pla : 3 2 2 0x y z .b) r és paral·lela al pla ': 3 5 0x y z .

a) vWr (0, 1, 1) Pr (0, 3, 3) nW (3, 2, 2)

vWr nW 0 3 1 2 1 ( 2) 0 → vWr nW

r i paral·lels o r continguda en .

Pr (0, 3, 3) → 3 0 2 3 2 3 0 → r està continguda en .

b) vWr (0, 1, 1) Pr (0, 3, 3) nW ' (3, 1, 1)

vWr nW ' 0 3 1 1 1 ( 1) 0 → vWr nW '

r i ' paral·lels o r continguda en '.

Pr (0, 3, 3) ' → 3 0 1 3 1 3 5 0 → r és paral·lela a '.

057 Donada la recta rx y z

x y z:

4 7 06 2 2 8 0

, troba l’equació d’un pla

que la contingui i que sigui perpendicular a : 4 2 7 0x y z .

Escrivim les rectes r en forma paramètrica.

r

x

y

z

:

11

2

5

2

25

2

13

2P

11

2

25

20, , vW (5, 13, 2)

El pla ' està determinat per Pr, vWr i el vector normal de , nW (4, 2, 1).

':

x y z

x y z

11

2

25

20

5 13 2

4 2 1

9 3 42 87 0

': 3 14 29 0x y z

305

5SOLUCIONARI

058 Considera les rectes:

rx y z

x y z:

2 3 03 2 4 3 0

sx z

x y z:

2 5 12 02 3 5 0

Troba l’equació d’un pla que:a) Contingui r i sigui paral·lel a s.b) Contingui r i sigui perpendicular a s.

Escrivim les rectes r i s en forma paramètrica.

r

x

y

z y

v:

9

7

6

7

3

7

5

7

rr rP( , , ) , ,6 5 79

7

3

70W

s

x

y

z

v:

65

2

1

2

11

4

ss sP( , , ) , ,10 11 4 61

20W

a) Els vectors directors del pla són els de r i s, i qualsevol punt de r pertany

al pla.

:

x y z

x y z

9

7

3

7

6 5 7

10 11 4

57 94 116 33 0

b) Si un pla és perpendicular a s, totes les rectes contingudes en el pla

són perpendiculars a s i, per tant, r i s han de ser perpendiculars.

Com que vWr vWs ( , , ) ( , , )6 5 7 10 11 4 23 0, les rectes

no són perpendiculars i el pla que busquem no existeix.

059 Calcula l’equació d’una recta que talli perpendicularment

a r x y z:2

32

13

i que passi pel punt P(14, 3, 3).

Busquem un punt Q r tal que PQW sigui perpendicular al vector director

de la recta r.

Un punt genèric de r té la forma Q ( , , )2 3 2 1 3 .

PQW ( , , )2 14 2 3 2

PQW uW ( , , ) ( , , )2 14 2 3 2 2 2 3 17 34 0 2

Així, doncs, Q ( , , )4 1 7 i PQW ( , , )10 4 4 .

Per tant, la recta és sx y z

:14

10

3

4

3

4.

306

Producte escalar

060 Troba l’equació de la recta que talla r i s perpendicularment.

rxyz

sxy: :

111 2

1

6 42

zz 2

Determinem un punt P r i un punt Q s, de manera que el vector PQW sigui perpendicular a les dues rectes.

P r

Q s

( , , )

( , , )

1 11 2 1

6 4 2 2PPQ ( , , )5 4 13 2 3W

PQ

PQ

( , , )

( , , )

0 2 1 5 3 23 0

4 1 1 3 18

1

330 0

4

1

W

W

Per tant: P r

Q sPQ

( , , )

( , , )( , , )

1 3 5

2 1 31 4 8W

Així, doncs, la recta que busquem és: x y z1

1

3

4

5

8

061 Esbrina si hi ha algun pla que contingui la recta r i que sigui perpendicular a la recta s.

r x y z: 32

14 1

sxyz

:1 51 22

En cas afirmatiu, calcula l’equació del pla.

Si un pla és perpendicular a s, totes les rectes contingudes en el pla són

perpendiculars a aquesta recta i, per tant, r i s han de ser perpendiculars.

Com que ( , , ) ( , , )2 4 1 5 2 2 0 Els vectors són perpendiculars.

El pla que busquem té com a vector normal el vector director de s

i passa pel punt P(3, 1, 0) r.

: 5 2 2 0x y z D

D D( )5 3 2 1 2 0 0 17

L’equació del pla que busquem és : 5 2 2 17 0x y z .

062 Considera els punts A(3, 0, 0), B(0, 2, 0) i C(0, 0, 1).

a) Troba l’equació general del pla que els conté. b) Troba l’equació de la recta perpendicular a i que passa per l’origen

de coordenades. Troba també el punt d’intersecció de la recta amb el pla.

(Activitat real de Selectivitat)

a) El pla passa per A(3, 0, 0) i té per vectors

directors ABW ( 3, 2, 0) i BCW (0, 2, 1).

:

x y z

x y z

3

3 2 0

0 2 1

2 3 6 6 0

307

5SOLUCIONARI

b) La recta perpendicular a que passa per l’origen és:

r

x

y

z

:

2

3

6

Determinem el punt d’intersecció de la recta r i el pla .

2 2 3 3 6 6 6 06

49

El punt d’intersecció és P12

49

18

49

36

49, , .

063 Troba l’equació contínua de la recta que talla perpendicularment les rectes

r x y z1

12 1

20

: r x y z t2 0 0 1 1 1 1: ( , , ) ( , , ) ( , , )

(Activitat real de Selectivitat)

Trobem un punt P r i un punt Q s, de manera que el vector PQW sigui

perpendicular a les dues rectes.

P

QPQ

( , , )

( , , )( ,

1 2 2

11 2 ,, )1W

PQ

PQ

( , , )

( , , )

2 1 0 3 5 2 0

1 1 1 3 3 2 0

0

2

3

1 0 2

2

3

2

3

5

3

P

Q

( , , )

, ,

PQ1

3

2

3

1

3, ,

W

W

W

La recta que busquem és t: x y z

tx y z1

1

3

0

2

3

2

1

3

1

1 2

2

1:

064 Considera les rectes:

rx ay

y z:

11

sx y z

x y z:

2 08

Prova que, per a cap valor de a, r i s poden ser paral·leles, i esbrina l’únic valor de a pel qual es tallen. Per a aquest valor de a: a) Calcula el punt P intersecció de r i s i l’equació del pla que les conté.b) Determina l’equació de la recta t que està continguda en i que és perpendicular

a r en el punt P. Escriu l’equació de dues rectes més que siguin perpendiculars a r pel punt P.

(Activitat real de Selectivitat)

Escrivim les rectes en forma paramètrica.

r

x a

y

z

Pr: ( )

1

1

1 0 1, ,

vWr (a, 1, 1)

s

x

y

z

Qs:

16

3

1

3

8

3

2

3

166

3

8

30, ,

vWs (1, 2, 3)

308

Producte escalar

Estudiem si vWr i vWs són proporcionals.

a

1

1

2

1

3? → Les rectes no són paral·leles per a cap valor de a.

Les rectes r i s són secants si el rang de la matriu que formen Pr WQs, vWr i vWs és 2.

16

31

8

31

1 1

1 2 3

10 20a

a

r i s secants → rang (A ) 2 10 20 0 2a a

a) Resolem el sistema:

1 216

3

1

3

8

3

2

3

1

2

1 El punt d’intersecció és P (5, 2, 1).

El pla que conté les dues rectes és:

:

x y z

x y z

5 2 1

2 1 1

1 2 3

5 7 3 8 0

b) La recta t és la intersecció del pla i d’un altre pla perpendicular a la recta r

que passa per P.

El pla, ', perpendicular a r i que passa per P, té per vector

normal uW (2, 1, 1) i passa pel punt P (5, 2, 1) '.

': 2 0x y z D

P D D( , , )5 2 1 2 5 2 1 0 13'

El pla que busquem és x y z: 2 13' 00.

La recta t és la intersecció entre els dos plans.

tx y z

x y z:

2 13 0

5 7 3 8 0

Calculem dues rectes més perpendiculars a r i que passen per P.

Si r r v v u u ur r' ' ( , , ) ( , , )1 2 3 2 1 1 0W W

Per exemple: v

vr

r

'

''

( , , )

( , , )

0 1 1

1 1 1

WW

compleixen aquesta condició.

Així doncs, les rectes:

r

x

y

z

':

5

2

1

i r

x

y

z

'':

5

2

1

són perpendiculars a r pel punt P.

309

5SOLUCIONARI

065 Troba l’equació del pla que passa pel punt A(1, 0, 1), és perpendicular

al pla x y z2 1 0 és paral·lel a la recta x yz

2 00

.

(Activitat real de Selectivitat)

El pla que busquem té per vectors directors el vector normal al pla i el vector

director de la recta, i passa pel punt A.

La recta en forma paramètrica és r

x

y

z

vr: ( , , )

2

0

2 1 0W

: x y z2 1 0 Vector normal: nW (1, 1, 2)

L’equació del pla que busquem és:

' :

x y z

x y z

1 1

2 1 0

1 1 2

2 4 3 5 0

066 Considera A(1, 1, 1), B(2, 0, 1), C (5, 2, 1) i D(4, 3, 3).a) Justifica que els punts són els vèrtexs consecutius d’un paral·lelogram.b) Raona si aquest paral·lelogram és un rectangle.c) Determina l’equació general del pla que conté els quatre punts.

(Activitat real de Selectivitat)

a) ABW DCW (1, 1, 2) → ABW i DCW són paral·lels i de la mateixa mida.

BCW ADW (3, 2, 2) → BCW i ADW són paral·lels i de la mateixa mida.

Per tant, són els vèrtexs consecutius d’un paral·lelogram.

b) ABW DCW ( , , ) ( , , )1 1 2 3 2 2 0 No són vectors perpendiculars.

No és un rectangle, és un romboide.

c) Determinem el pla amb vectors directors ABW i BCW, i que passa per A.

:

x y z

x y z

1 1 1

1 1 2

3 2 2

2 8 5 1 0

067 Troba els punts R pertanyents a la recta: rx y z

x y z:

2 3 02 3 1 0

de tal manera

que els segments PQ i PR formen un angle recte, en què P(1, 0, 0) i Q(0, 1, 5).

(Activitat real de Selectivitat)

Escrivim la recta en forma paramètrica.

r

x

y

z t

vr: ( , , )

2

1 1 1 1W

310

Producte escalar

Si R r R ( , , )2 1

PQ PR( , , ) ( , , )

( , , ) (

1 1 5 3 1

1 1 5 3 , , )1 3 4 04

3

W W

Per tant, l’únic punt que compleix la condició és R10

3

7

3

4

3, , .

068 Considera els punts A(0, 3, 1) i B(0, 1, 5).a) Calcula els valors de x si saps que el triangle ABC de vèrtexs A, B i C ( x, 4, 3)

té un angle recte a C.b) Troba l’equació del pla que passa pels punts (0, 1, 5) i (3, 4, 3) i és paral·lel a la recta

definida per les equacions: x y z

x y0

2 3

(Activitat real de Selectivitat)

a) CA CB CA CB x x x x( , , ) ( , , )1 4 3 2 5 02 5W W W W

b) Escrivim la recta en forma paramètrica.

r

x

y

z

vr: ( ,

11

3

12

3

1 2 3, )W

P

QPQ

( , , )

( , , )( , , )

0 1 5

3 4 33 3 2W

Calculem l’equació del pla que passa pel punt P (0, 1, 5) i que té

per vectors directors vWr i PQW.

:

x y z

x y z

1 5

1 2 3

3 3 2

13 7 9 38 0

069 Troba l’equació contínua de la recta que talla perpendicularment les rectes:

rx z

x y z

r x y z

1

2

1 02 1 0

1 11

2

:

:

r1

90°

90°r2

(Activitat real de Selectivitat)

Trobem un punt P r1 i un punt Q r2, de manera que el vector PQW

sigui perpendicular a les dues rectes.

r

x

y

z

u1

1

3 1 1 1: ( , , )W rx y z

v2

1 1

1

21 1 2: ( , , )W

311

5SOLUCIONARI

Un punt genèric de la recta r1 és P ( , , )1 3

Un punt genèric de r2 és Q ( , , )1 2

PQW ( , , )1 3 2 1

PQ u

PQ v

3 4 3 0

4 6 2 0

3 4 3 0

4 6 2 0

5

3

W

WW

W

Per tant, els punts són P ( 4, 2, 5) i Q ( 3, 3, 5).

La recta que busquem té per vector director PQW (1, 1, 0) i passa per P ( 4, 2, 5).

s: x y z4

1

2

1

5

0

070 Considera les rectes:

r x y z s x y z: :12

22 4

23

11

21

a) Troba l’equació de la recta t que passa per l’origen i talla les dues rectes anteriors.

b) Troba la perpendicular comuna a les rectes r i s.

(Activitat real de Selectivitat)

a) La recta t és la intersecció dels plans i ', en què és el pla que conté

la recta r, i ', el pla que conté s i que passa per l’origen.

:

( , , )

( , , )

( , , )

O

u

P O

r

r

0 0 0

2 2 4

1 2 0

WW

:

x y z

x y z2 2 4

1 2 0

8 4 2 0

: 4 2 0x y z

':

( , , )

( , , )

( , , )

O

v

Q O

s

s

0 0 0

3 1 1

2 1 2

':

x y z

x y z3 1 1

2 1 2

8 5 0WW

Per tant, la recta t és:

tx y z

x y z:

4 2 0

8 5 0

b) Busquem un vector nW (a, b, c) perpendicular als vectors uWr i vWs .

( , , ) ( , , )

( , , ) ( , ,

a b c a b c

a b c

2 2 4 2 2 4 0

3 1 1)) 3 0

3

4

5

4a b c

a

b

c

Per exemple, un vector que compleix aquesta condició és nW ( 3, 5, 4).

La recta perpendicular comuna a r i a s és la intersecció dels plans i ', en què és el pla que conté r i que té per vector director nW, i ' és el pla

que conté s i que té per vector director nW.

312

Producte escalar

:

, ,

( , , )

( , , )

P

u

n

r

r

( )1 2 0

2 2 4

3 5 4

W

W :

:

x y z

x y z

x y

1 2

2 2 4

3 5 4

28 20 4 12 0

7 5 z 3 0

':

, ,

( , , )

( , , )

Q

v

n

s

s

( )2 1 2

3 1 1

3 5 4

':

x y z

x y z

2 1 2

3 1 1

3 5 4

15 18 23 0W

W

Per tant, la recta perpendicular comuna a la recta r i s és:

7 5 3 0

15 18 23 0

x y z

x y z

071 Calcula l’equació de la recta que passa pel punt P(2, 1, 1) i talla perpendicularment la recta:

x y z33

11 2

(Activitat real de Selectivitat)

Calculem el pla que és perpendicular a la recta i que passa per P(2, 1, 1).

Vector normal: v x y z D

P

r ( , , ) :

( ,

3 1 2 3 2 0

2 11 1 3 2 1 2 1 0 7, ) ( ) D D

W

El pla que busquem és : 3 2 7 0x y z .

Determinem el punt de tall de la recta i el pla.

x y z

x y z

x

y

3

3

1

1 2

3 2 7 0

39

14

15

14

1

7z

La recta passa per P (2, 1, 1) i Q39

14

15

14

1

7, , → PQW ( , , )11 1 16 .

La recta que busquem té com a vector director PQW i passa pel punt P (2, 1, 1).

sx y z

:2

11

1

1

1

16

072 Donats els punts A(1, 1, 1), B ( 1, 3, 1), C (1, 0, 0) i D (0, 2, 0); troba el punt P que pertany a la recta determinada per A i B de tal manera que el triangle CDP sigui rectangle amb hipotenusa CP.

(Activitat real de Selectivitat)

La recta que passa per A i B és: r

x

y

z

AB :

1 2

1 2

1

313

5SOLUCIONARI

Un punt genèric de rAB és P ( , , )1 2 1 2 1 .

DC DP ( , , ) ( , , )1 2 0 1 2 1 2 1 0 3 6 01

2

W W

Per tant, el punt que busquem és P(0, 2, 1).

073 Considera la recta r, definida per x y z14

12

i el pla

d’equació 2x y z 0. Determina i en cada un dels casos següents.

a) La recta r és perpendicular al pla .b) La recta r està continguda en el pla .

(Activitat real de Selectivitat)

rx y z

vr: ( , , )1

4

1

24 2W

: ( , , )2 0 2 1x y z nW

a) vWr ha de ser proporcional a nW 2

4

1

28

1

2

b) El punt Pr ( , , )1 0 1 2 1 1 0 1 0 2

vWr ( , , )4 2 és perpendicular a nW ( , , )2 1 2 : vWr nW 0 4 2 2 1 2 0 4( , , ) ( , , )

074 Decideix si el pla 6 4 1 0x y z pertany al feix de plans definit

per la recta 2 3 02 8 0

x y zy z

.

En cas afirmatiu, expressa-ho com a combinació lineal dels dos plans que defineixen la recta.

Estudiem si el pla conté la recta.

A

2 1 1

0 1 2

6 4 1

A*

2 1 1 3

0 1 2 8

6 4 1 1

rang (A) rang (A*) 2 → Sistema compatible indeterminat

El pla pertany al feix de plans definit per la recta. L’expressió del pla

com a combinació lineal dels dos plans que defineixen el feix de plans és:

6 4 1 2 3 2 8x y z x y z y z( ) ( )

6 2

4

1 2

1 3 8

3

1

Així, doncs: 6 4 1 3 2 3 2 8x y z x y z y z( ) ( )

314

Producte escalar

075 Determina un pla que contingui la recta:x y z

x y z2 5 0

2 3 8 0

i que sigui paral·lel al pla 6 22 1 0x z .

Escrivim les equacions paramètriques de la recta.

r

x

y

z

v:

16

3

11

3

8

3

13

3

rr rP( , , ) , ,11 8 316

3

8

30W

: 6 22 1 0x z Vector normal: nW (6, 0, 22)

Com que ' és paral·lel a ': 6 22 0x z D

Com que P r D Dr

16

3

8

30 6

16

322 0 0 3, , 22

Per tant, l’equació del pla que busquem és ': 6 22 32 0x z .

076 Troba un pla del feix de plans de la recta:

r x y z: 12

2 1

que sigui perpendicular a la recta:

sx ty tz t

:12

3

Expressem la recta en forma implícita:

rx y

y z

x y

y z:

1 2 4

2 1

2 5 0

3 0

El feix de plans és:

( )x y y z2 5 3 0 x y z( )1 2 5 3 0

Busquem un pla del feix que tingui el vector normal paral·lel al vector uWr (1, 2, 1).

1

1 2

2

1

1 No té solució.

Així doncs, el pla que ens demanen no existeix.

077 Calcula, utilitzant el feix de plans, l’equació del pla que conté

la recta rx y z

x y z:

3 2 4 85 2 5

i que passa pel punt P(1, 0, 3).

El feix de plans que conté la recta és:

3 2 4 8 5 2 5 0x y z x y z( )

I com que ha de passar pel punt P(1, 0, 3):

3 1 2 0 4 3 8 5 1 2 0 3 5 0( ) 7

3

315

5SOLUCIONARI

El pla que busquem és:

3 2 4 87

35 2 5 0

26

3

8

3

5

3

11

3

x y z x y z

x y z

( )

0 26 8 5 11 0x y z

078 Classifica en agut, obtús o recte l’angle que formen Wu i Wv segons el signe de Wu Wv .

Si és agut.u v u v u v0 0cosW W W WW W

Si és obtús.u v u v u v0 0cos W WW WW W

Si és recte.u v u v u v0 0cos W WW WW W

079 Per a quins valors de k els vectors Wa (k, 3, 5) i Wb (2, 4, 2) formen un angle obtús?

a b k2 12 10W W

Formaran un angle obtús quan: 2 22 0 11k k

080 Determina l’angle que forma el vector Wu (3, 2, 4) amb Wv (4, 0, 1), i troba un altre vector que formi el mateix angle amb Wv .

u v u v8 29 178

29 170 3603 68 88cos , , °W W W W

Perquè compleixi quew a b c w v u v( , , ) :W W WW W

cosu v

u v

w v

w v

a c

w v

4W

W W W

WW

W W W

W

És a dir, w uW W i 4 8a c . Per exemple, wW (3, 2, 4) compleix aquestes

condicions.

081 Troba l’angle que formen aquestes parelles de vectors.

a) Wu (4, 1, 3) i Wv (3, 0, 2)b) Wu (5, 4, 1) i Wv (2, 3, 2)c) Wu ( 4, 2, 5) i Wv (1, 3, 2)d) Wu (6, 8, 4) i Wv ( 9, 12, 6)

a) °cos18

26 130 9791 11 44 34, ' ''

b) °cos0

42 170 0

c) °cos8

45 140 3187 108 31 10, ' ''

d) °cos174

116 2611 180

26

3

8

3

5

3

11

3x y z 0 26 8 5 11 0x y z

316

Producte escalar

082 Determina la projecció ortogonal del vector Wu sobre el vector Wv.a) Wu (3, 5, 2) i Wv (1, 2, 0) b) Wu (4, 1, 2) i Wv (3, 2, 1)

a) Projv uu v

v

7

5

7 5

5

W WW

WW b) Projv u

u v

v

12

14

6 14

7

W W

WWW

083 Quin valor ha de tenir a perquè l’angle que formen Wu (1, 2, 2) i Wv ( 3, 1, a) sigui de 60°?

cos 602 1

3 10

1

27 16 86 0

2

2°a

aa a

L’única solució vàlida és a8 3 74

7.

084 Si Wu i Wv són vectors ortogonals i de mòdul 1, troba els possibles valors del paràmetre real a perquè els vectors Wu aWv i Wu aWv formin un angle de 60°.

(Activitat real de Selectivitat)

( ) ( )u a v u a v u u a v v u a v a22

22

21W WWW W WWW W W

( ) ( )u a v u a v u a u v a v a u a v a2

22

2 22 1 1W WW W WW WW WW

( ) ( )u a v u a v u a u v a v a u a v a2

22

2 22 1 1W WWW W WW WWW

cos 601

1 1

2

2 2°

( ) ( )u a v u a v

u a v u a v

a

a a

11

2

3

3a

W

W W

W W

W W

W

085 Decideix si el triangle amb vèrtexs A( 2, 4, 0), B(3, 3, 1) i C (6, 2, 4) és rectangle, acutangle o obtusangle.

AB ( , , )5 7 1 5 3W

AC ( , , )8 6 4 2 29W

BC ( , , )3 1 3 19W

El costat més gran és ACW i, a més:

AC AB BC2 2 2

116 19 75W W W

Així, doncs, el triangle és obtusangle.

086 Calcula l’angle que formen Wa i Wb, si saps que Wa 3, Wb 8 i que Wa Wb 6.

a b a a b b a b2 2 2

2 36 9 2 64W W W WW WW W

a b1

236 9 64

37

2( )W W

a b a b37

23 8 cos cos

37

48140 25 43° ' "W WW W

317

5SOLUCIONARI

087 Troba el valor de Wa Wb , si Wa 4 i Wb 7 i l’angle que formen Wa i Wb és de 67°.

( ) ( )a b a b a a b b2 2

2W W WWW W W W

a b a b2

16 2 4 7 67 49 43 11 6 56cos ° , ,W WW W

088 Si Wu 3 i Wv 4 i els dos vectors formen un angle de 135°, determina l’angle que formen els vectors 2Wu Wv i 3Wu 2Wv .

( ) ( )2 3 2 6 2 6 3 3 4 1352 2

2u v u v u u v v cos ° 2 4

6 2 22

2WW W WWW WW

( ) ( )2 2 4 4

2 4 3 4 3 4

2 2

2

u v u v u u v v

u v coss 135 4 4 24962° ,

WW WW W

W

W W

W

( ) ( )3 2 3 2 9 12 4

3 2 9 3 12

2 2

2

u v u v u u v v

u v 33 4 135 4 4 15 71062cos ° ,

W W W WW W

W

W W

W

cos( ) ( )

,

2 3 2

2 3 2

6 2 22

4 2496 1

u v u v

u v u v 55 71060 4566 62 49 55

,, ° ' "

W

W

W

W

W

W

W

W

089 Demostra que les diagonals d’un rombe són perpendiculars.

Els dos vectors, aW i bW, que generen un rombe tenen el mateix mòdul. Les diagonals

són aW bW i aW bW i el producte escalar és ( )( ) .a b a b a b2 2

0W WW W WW

Així, doncs, les diagonals són perpendiculars.

090 Determina els angles que descriuen les parelles de rectes següents:

a) rxyz

s x y z: :142 2

23 4

12

b) r x y z s x y z: :16

24

12

13

12

21

c) rxyz

sx y z

: :2 47 14

2 6xx z4 0

d) rx y z

x y zs x y z: :

2 3 03 2 4 5

12

42

21

a) u v

u v

u v

r ( , , ) ( , , )1 1 2 3 4 2

5

6 29

i

cos

s

00 379 67 43 31, ' "°W W

WW

WW

b) u v

u v

u v

r ( , , ) ( , , )6 4 2 3 2 1

28

56 1

i

cos

s

441 0°

W W

WW

W W

W

318

Producte escalar

c) ur ( , , )1 4 14W

s

x

y

z

vs: ( , , )

4

35

28 5 2W

uW vW 0 90°

d) vWs (2, 2, 1)

r

x

y

z

:

5 2

10 uWr ( 2, 2, 1)

cosu v

u v

7

6 30 9526 17 42 56, ° ' "

W W

WW

091 Calcula l’angle que formen aquestes parelles de rectes i plans:

a) : :x y z rxyz

2 3 81

2 23

b) : :x y z r x y z3 6 32

32

14

c) : :2 2 2 32 3 8

2 4x y z r

x y zx y z

a) °n

ur

( , , )

( , , )

1 2 3

1 2 190 arccoss

arccos

u n

u n

r

r

906

14 6° 90 49 1 40 9° ° °, ,

W

W W

W

W

W

b) °n

ur

( , , )

( , , )

1 3 1

2 2 490 arccoos

arccos

u n

u n

r

r

900

11 24° 90 90 40° ° °

W

W W

W

W

W

c) nW (2, 2, 2)

r

x

y

z

: 4 uWr (1, 1, 1)

90 90° °arccos arcu n

u n

r

r

ccos6

3 1290 0 90° ° °

W

W

W

W

319

5SOLUCIONARI

092 Troba l’angle que defineixen aquestes parelles de plans:a) : :2 3 9 2 2 2 19x y z x y z

b) : :x y z x y z5 3 1 3 5 7 9

c) : :4 12 28 132 3

2 21

x y zx t sy t sz t

a)n

nn n

( , , )

( , , )

2 1 3

2 2 20 90°

W

W

b)n

n

n n

n

( , , )

( , , )

1 5 3

3 5 7

cosnn

43

35 830 7978 37 4 45, ' "°

W

W

W

W

W

W

c) Escrivim el pla en forma implícita.

:

x y z

x y z

2 2 1

1 2 1

3 1 0

3 7 1 0

n

n

n n( , , )

( , , )

4 12 28

1 3 7cos

nn n

236

944 591 0°

WW

WW

W

W

093 Determina el vector o els vectors unitaris, Wv (a, b, c) (amb a 0, b 0, c 0),

que formen un angle de 6

radians amb el vector Wu (1, 1, 1) i un angle de 4

radians amb el vector Ww (2, 0, 2).

(Activitat real de Selectivitat)

Plantegem un sistema amb les condicions de l’enunciat.

v a b c

u v

u v

a

2 2 2 1

6cos

bb c

w v

w v

a c

3

3

2

4

2 2

8

2

2cos

W

W

W

W

W

W

W

W

W

Si eliminem els denominadors i suprimim les arrels, obtenim el sistema

següent:

a b c

a b c

a c

2 2 2 1

3

2

1

W W

320

Producte escalar

Aquest sistema té dues solucions:

a

b

c

2 2

4

1

2

2 2

4

a

b

c

2 2

4

1

2

2 2

4

Per tant, hi ha dos vectors que compleixen les condicions del problema:

vW1 2 2

4

1

2

2 2

4, , vW2

2 2

4

1

2

2 2

4, ,

094 Donades les rectes:

rx y

zs

xy z

: :2 0

12

5 0

a) Determina’n la posició relativa.b) En cas que es tallin, determina l’angle que formen i el punt de tall.

(Activitat real de Selectivitat)

a) Passem les dues rectes a forma paramètrica.

rx y

z

x

y

z

:2 0

1

2

1

u Pr r( , , ) ( , , )1 1 0 2 0 1W

sx

y z

x

y

z

:2

5 0

2

5 vv Qs s( , , ) ( , , )0 1 1 2 5 0W

Estudiem el rang de la matriu formada pels vectors PQW (4, 5, 1), uWr i vWs.

A

4 5 1

1 1 0

0 1 1

→ rang ( A ) 2

Com que els vectors uW i vW no són paral·lels, r i s són secants.

b) Angle que formen:

cosu v

u v

r s

r s

1

2 2

1

260°

W

W

W

W

Calculem el punt de tall.

2 2

5

1

4

1

El punt de tall entre les dues rectes és C (2, 4, 1).

321

5SOLUCIONARI

095 Calcula l’angle que forma el pla x y z 0 amb la recta

d’equacions x yy z

11

.

(Activitat real de Selectivitat)

rx y

y zr

x t

y t

z t

: :1

1

1

1

ur ( , , )1 1 1W

: x y z 0 nW (1, 1, 1)

90 90° °arccos arcu n

u n

r

r

ccos1

3 319 47, °

W

W

W

W

096 Considera i ’ els plans de l’espai R3, determinats de la manera següent:El pla passa pels punts (0, 2, 1), (3, 1, 1) i (1, 1, 5) i el pla ’ passa pels punts (3, 0, 2), (2, 1, 1) i (5, 4, 2). Calcula: a) Una equació paramètrica de la recta intersecció dels plans i ’.b) L’angle que formen els plans i ’.c) L’equació del pla que conté la recta r i que forma un angle de 90°

amb el pla .

(Activitat real de Selectivitat)

Pla .

P (0, 2, 1) Q (3, 1, 1) R (1, 1, 5) PQW (3, 3, 0) PRW (1, 3, 4)

:

x y z

x y z x y

2 1

3 3 0

1 3 4

12 12 6 30 0 2 2: zz 5 0

Pla '.P'(3, 0, 2) Q'(2, 1, 1) R'(5, 4, 2) P'Q' (3, 3, 0) P'R' (2, 4, 4)

' ':

x y z

y z y z

3 2

1 1 1

2 4 4

6 6 12 0 2 0:

a) rx y z

y zr

x

y

z

:2 2 5 0

2 03 2:

11 2

b) cosn n

n n

'

'

3

3 2

2

245°

W

W

W

W

c)

P r

u

n

r

( , , )

( , , )

( , , )

0 3 1

1 2 2

2 2 1

W

W

'':

x y z

x y z

3 1

1 2 2

2 2 1

6 3 6 3 0

'': x y2 22 1 0z

#$ #$

322

Producte escalar

097 Esbrina per a quin valor de m la recta rx y z m

x y z:

2 52 2

es talla

amb la recta s x y z: 12

13

45

.

Calcula el cosinus de l’angle que formen les dues rectes.

(Activitat real de Selectivitat)

Estudiem el sistema d’equacions format per les equacions de les dues rectes.

x y z m

x y z

x y

y z

2 5

2 2

1

2

1

3

1

3

4

5

x y z m

x y z

x y

2 5

2 2

3 2 5

5 3 17y z

A

1 2 5

2 1 1

3 2 0

0 5 3

A

m

*

1 2 5

2 1 1 2

3 2 0 5

0 5 3 17

1 2 5

2 1 1

3 2 0

0 3rang ( )A

1 2 5

2 1 1 2

3 2 0 5

0 5 3 17

12 327

m

m

Si rang ( )12 327 0327

123m m A*

Si rang ( )12 327 0327

124m m A*

Si m327

12 rang ( A ) rang ( A* ) 3. El sistema és compatible determinat.

Les rectes r i s es tallen en un punt.

Determinem els vectors directors de les rectes.

r

xm

ym

z

:

4

5

3

5

2 2

5

11

5

ur ( , , )3 11 5W

sx y z

:1

2

1

3

4

5 vWs (2, 3, 5)

L’angle que formen r i s és el que formen els seus vectors directors.

cos arccosu v

u v

14

155 380 1824 0 1824 7, , 99 49, °

W

W

W

W

323

5SOLUCIONARI

098 Considerem els plans:

1 6 0: x y 2 2 4 2 0: x y z

en què és un paràmetre real. Es demana:

a) Determina les equacions paramètriques de la recta intersecció dels dos plans 1 i 2 quan 4.

b) Calcula raonadament perquè els plans 1 i 2 es tallin i formin un angle de 45°.

(Activitat real de Selectivitat)

a) Si 4 2 4 4 2 0 2 2 1 02 2: :x y z x y z

rx y

x y zr

x

y: :6 0

2 2 1 0

13 2

7 2

z

b) °n

n

n1

2

1 1 0

2 445

1( , , )

( , , )cos

nn

n n

2

1 22

2

6

2 20

1

2

6 20 4

W

W

W

WW

W

099 Donats els plans 1 i 2 d’equacions:

1 22 3 0 2 6 0: :x y z x y zes demana:

a) Calcular l’angle que formen els plans 1 i 2.

b) Calcular l’equació paramètrica de la recta r, intersecció dels plans 1 i 2.

c) Comprovar que el pla d’equació : x y 1 0 és el pla bisector de 1 i 2,

és a dir, que forma un angle 2

amb cadascun dels plans 1 i 2, on

és l’angle obtingut a l’apartat a).

(Activitat real de Selectivitat)

a)n

n

n n

n

1

2

1 2 1

2 1 1

1 2( , , )

( , , )cos

11 2

6

6 6

1

260

W

W

W

W

W

W

b) rx y z

x y zr

x

y: :2 3 0

2 6 0

5

4

z

c) : x y 1 0 nW (1, 1, 0)

cosn n

n n

1

1

3

6 2

3

230°

W

W

W

W

cosn n

n n

2

2

3

6 2

3

230°

W

W

W

W

Per tant, el pla és bisector dels plans 1 i 2.

324

Producte escalar

100 Donats la recta rx

y z:

11 0

i el pla : x y 2 0 :

a) Determina’n la posició relativa.b) Si es tallen, determina l’angle que formen i el punt de tall.

(Activitat real de Selectivitat)

a) rx

y zr

x

y

z

: :1

1 0

1

1 uWr (0, 1, 1)

: x y 2 0 nW (1, 1, 0)

Com que uWr nW 0 → Els vectors no són perpendiculars i, per tant, el pla

i la recta són secants.

b)

°

u

n

u

r

r

( , , )

( , , )

0 1 1

1 1 0

90 arccosn

u nr

901

2 2° arccos 30°

W

W

W

W

W

W

Determinem el punt de tall resolent el sistema format per totes

les equacions.

x

y z

x y

x

y

z

1

1 0

2 0

1

3

2

Per tant, el punt de tall és P ( 1, 3, 2).

101 Donades les rectes:

ry xz x

s x y z: :2 12 1

32

32

21

troba una recta bisectriu de r i s (una recta bisectriu d’unes altres dues rectes passa pel seu punt d’intersecció, es troba en el mateix pla que elles i forma el mateix angle amb totes dues).

(Activitat real de Selectivitat)

Determinem el punt d’intersecció de les rectes.

y x

z x

x y

y z

2 1

2 1

3

2

3

2

3

2

2

1

x

y

z

1

1

1

El punt d’intersecció de r i s és P ( 1, 1, 1).

325

5SOLUCIONARI

Determinem els vectors directors de r i s.

ry x

z xr

x

y

z

: :2 1

2 12 1

2 1

uWr (1, 2, 2)

sx y z

:3

2

3

2

2

1 vWs (2, 2, 1)

Busquem un vector wW (a, b, c) que compleixi aquesta condició:

cos ( uWr, wW ) cos ( vWs, wW )

u w

u w

v w

v w

r

r

s

s

W W

W W

W W

W W

a b c

w

a b c

w

a c a c

2 2

3

2 2

3

0

W W

Els vectors wW que busquem són del tipus wW (a, b, a).

A més, el vector wW ha de pertànyer al pla determinat

per les rectes r i s; és a dir, cal que sigui linealment dependent

dels vectors directors de les rectes.

a b a

b a a b1 2 2

2 2 1

0 3 4 03

4

Si fem b 4, un vector pot ser wW (3, 4, 3).

L’equació de la bisectriu de r i s és:

t

x t

y t

z t

:

1 3

1 4

1 3

102 Calcula l’altura que parteix de B en el triangle de vèrtexs A(3, 1, 2), B(4, 2, 1) i C (5, 3, 4), fent servir projeccions de vectors.

A

B

CD

hp

AB AC

AB AC

( , , ) ( , , )1 3 3 2 4 6

19 2 14

W

W W

W

p AD ABAB AC

ACACProj

32

2 14

8 14

7

W WW W

WW

h AB AD2 2

0 8452,WW

326

Producte escalar

103 Considera la recta: r x y z: 52

35

711

i el pla : 2 3 8 0x y z :

a) Decideix-ne la posició relativa i, si es tallen, l’angle que formen.

b) Calcula la projecció ortogonal de la recta r sobre el pla .

a) uWr (2, 5, 11) nW (2, 1, 3)

Com que uWr nW 0 → Els vectors no són perpendiculars. La recta talla el pla.

90 9042

150 1466 42° ° °arccos arccos

u n

u n,

W

W

W

W

b) Calculem el pla ' que conté r i que talla perpendicularment .

P r

u

n

r

( , , )

( , , )

( , , )

5 3 7

2 5 11

2 1 3

':

x y z

x y z

5 3 7

2 5 11

2 1 3

4 16 8 12 0

W

W

': x y z4 2 3 0

La recta s, projecció ortogonal de la recta r sobre , és el tall

dels plans i '.

sx y z

x y zs

x

y: :2 3 8 0

4 2 3 0

5 2

22

z

104 Calcula el punt simètric de P (6, 10, 22) respecte del pla 2 3 7 10 0x y z .

Determinem la projecció ortogonal Q del punt P sobre el pla .

Recta perpendicular a que passa per P.

P

nr

x

y( , , )

( , , ):

6 10 22

2 3 7

6 2

10 3

zz 22 7W

Intersecció entre la recta i el pla.

x

y

z

x y z

6 2

10 3

22 7

2 3 7 10 0

x

y

zQ

0

1

1

3

0 1 1( , , )

Q és el punt mitjà entre P i el seu punt simètric P'(a, b, c).

( , , ) , ,0 1 16

2

10

2

22

2

6a b c

a

b 88

20c

El punt simètric del punt P respecte del pla és P ( 6, 8, 20).

327

5SOLUCIONARI

105 Troba la projecció ortogonal P' del punt P (7, 3, 3) sobre el pla : 3 5x y z . Comprova que la distància de P al pla és la mateixa que la distància a P'.

Utilitza P' per calcular el punt simètric de P respecte del pla.

Trobem la projecció ortogonal P' del punt P sobre el pla .

Recta perpendicular a que passa per P.

P

nr

x

y

z

( , , )

( , , ):

7 3 3

3 1 1

7 3

3

3W

Intersecció entre la recta i el pla.

x

y

z

x y z

x

y

7 3

3

3

3 5

1

1

1

2

1 1 1z

P'( , , )

Comprovem que les distàncies són iguals:

d P( , )( )3 7 3 3 5

9 1 1

22

112 11

d P P PP( , ) ( , , )' ' 6 2 2 44 2 11W

Calculem el punt Q (q1, q2, q3) simètric del punt P respecte del pla .

( , , ) , ,1 1 17

2

3

2

3

2

1 2 3

1q q q

q 5

1

1

2

3

q

q

El punt simètric del punt P respecte del pla és Q ( 5, 1, 1).

106 Troba les coordenades del punt simètric a P (2, 1, 17) respecte de la recta r, l’equació vectorial de la qual és: r : (x, y, z) ( 1, 4, 2) (2, 1, 4).

Calculem la projecció ortogonal Q del punt P sobre r.

Pla perpendicular a r que passa per P.

P

ux y z

r

( , , )

( , , ):

2 1 17

2 1 42 4 73 0

W

Intersecció entre el pla i la recta.

x

y

z

x y z

1 2

4

2 4

2 4 73 0

x

y

zQ

5

7

14

3

5 7 14( , , )

Determinem el punt simètric P'(a, b, c) respecte de la projecció Q.

( , , ) , ,5 7 142

2

1

2

17

2

8

13a b c

a

b

cc 11

El punt simètric de P respecte de la recta r és P'(8, 13, 11).

328

Producte escalar

107 Considera el punt P (2, 0, 1) i la recta:

rx y

z:

2 62

a) Troba l’equació del pla que conté P i r.b) Calcula el punt simètric de P respecte de la recta r.

(Activitat real de Selectivitat)

a) Escrivim la recta r en forma paramètrica.

rx y

zr

x

y

z

: :2 6

2

6 2

2

uWr ( 2, 1, 0) Qr (6, 0, 2)

El pla que busquem té com a vectors directors uWr i PQWr, i passa

pel punt P.

P

u

PQ

r

r

( , , )

( , , )

( , , )

2 0 1

2 1 0

4 0 1

::

x y z

x y z

2 1

2 1 0

4 0 1

2 4 2 0

WW

b) Calculem la projecció ortogonal Q del punt P sobre r.

Pla perpendicular a r que passa per P.

P

ux y

r

( , , )

( , , ):

2 0 1

2 1 02 4 0

W

Intersecció entre el pla i la recta.

x y

z

x y

x

y

z

2 6

2

2 4 0

14

5

8

5

2

Q14

5

8

52, ,

Calculem el punt P'(a, b, c) simètric del punt P respecte del pla .

14

5

8

52

2

2

0

2

1

2, , , ,

a b c

a

b

c

18

5

16

5

3

El punt simètric del punt P respecte del pla és P'18

5

16

53, , .

108 Per a cada valor de a els punts P (1, 2, 3) i A(0, 1, a) són simètrics respecte d’un pla.Troba, de manera raonada, l’equació d’aquest pla. En particular, troba el pla per a a 2.

(Activitat real de Selectivitat)

329

5SOLUCIONARI

Trobem el pla que passa pel punt mitjà M del segment PA

i que té com a vector normal el vector PAW.

Ma

PA a

1

2

3

2

3

2

1 1 3

, ,

( , , )

: ( )x y a za

313

20

2

W

Per a a x y z x y z29

20 2 2 2 9 01 1: :

109 Escriu les equacions implícites d’una recta amb la direcció del vector (1, 1, 0) i que passa per P', en què P’ és el simètric de P(0, 2, 0) respecte del pla : x y z3 5.

(Activitat real de Selectivitat)

Determinem la projecció ortogonal Q del punt P sobre el pla .

Recta perpendicular a que passa per P.

P

nr

x

y

z

( , , )

( , , ):

0 2 0

1 3 12 3

W

Intersecció entre la recta i el pla.

x

y

z

x y z

x

y2 3

3 5 0

1

1

zzQ

1

1

1 1 1( , , )

Calculem les coordenades de P'(a, b, c), punt simètric de P respecte de .

( , , ) , ,1 1 10

2

2

2

0

2

2

4a b c

a

b

c 22

El punt simètric del punt P respecte del pla és P'(2, 4, 2).

L’equació de la recta, en forma implícita, que passa pel punt P' i té per vector

director (1, 1, 0) és:

x y zs

x y

y z

2

1

4

1

2

0

2

1

4

1

4

1

2

0

: sx y

z:

6 0

2 0

110 Troba la distància del punt A(3, 1, 2) al punt B(5, 2, 4).

d A B AB( , ) ( ) ( ) ( )5 3 2 1 4 2 72 2 2W

111 El triangle amb vèrtexs A(2, 5, 1), B(3, 2, 4) i C ( 2, 3, 11)?

AB BC

AC

( , , )    ( , , )

( , ,

1 7 5 5 3 5 1 7 5 3

4 8 122 4 14)

WW

W

Té dos costats iguals i un de desigual; el triangle és isòsceles.

330

Producte escalar

112 Quin és el costat més petit en el triangle amb vèrtexs A(3, 1, 4), B(0, 2, 3) i C (5, 5, 1)?

AB BC

AC

( , , )    ( , , )

( , , )

3 1 1 11 5 7 4 90

2 6 5 65

WW

W

El costat més petit és AB.

113 Determina les distàncies que hi ha entre aquests punts: P (1, 0, 3), Q (4, 5, 1) i R (10, 15, 3). Què en pots dir, dels tres punts?

d P Q PQ( , ) ( , , )3 5 2 38W

d Q R QR( , ) ( , , )6 10 4 2 38W

d P R PR( , ) ( , , )9 15 6 3 38W

Com que d P Q d Q R d P R( , ) ( , ) ( , ) Els tres punts estan alineats.

114 Comprova que la recta x y z32

13

3 no talla el pla 3 x 2y 8.

rx y

z u Pr r: ( , , ) ( , , )3

2

1

33 2 3 1 3 1 3W

: 3 2 8x y nW (3, 2, 0).

uWr nW 0 → uWr i nW són perpendiculars.

La recta és paral·lela al pla o hi està continguda.

Com que P r Pr r( , , ) ( , , )3 1 3 3 1 3i La recta i el pla són paral·lels.

115 Troba la distància al pla : 8x 4y z 5 0 dels punts P (2, 4, 12), Q (0, 1, 1) i R (1, 3, 2). Què en pots dir, del punt Q? I què tenen en comú P i R?

d P( , )( )

8 2 4 4 12 5

8 4 1

7

92 2 2 d R( , )

( )

8 1 4 3 2 5

8 4 1

7

92 2 2

d Q( , )( )

( )

8 0 4 1 1 5

8 4 10

2 2 2

El punt Q pertany al pla i els punts P i R n’equidisten.

116 Calcula la distància entre les parelles de plans següents:

a) : :xyz

4 2 212 5

'xxyz

4 43

4 2 4

b) : :xyz

x1 32

3

5 4' y

z1 2

8 3 2

c) : 2x 4y z 7 0 ' : x 3y 5z 9 0

331

5SOLUCIONARI

a) Escrivim els plans en forma implícita.

:

x y z

x y z

4 1 2

2 1 1

2 1 5

6 8 4 24 0

':

x y z

x y z

4 3 4

0 1 2

4 1 4

6 8 4 16 0

Els plans són paral·lels. Si prenem P ( , , ) :4 3 4 '

d d P( , ) ( , )( )

'6 4 8 3 4 4 24

36 64 16

8

1116

4 29

29

b) Escrivim els plans en forma implícita.

:

x y z

x y z

1 3

3 2 0

1 1 1

2 3 1 0

':

x y z

x y z

5 1 8

0 1 3

4 2 2

8 12 4 4 0

Els plans són coincidents d ( , )' 0.

c) Els plans són secants d ( , )' 0.

117 Troba la distància del punt P ( 5, 2, 2) al pla d’equació : 2x y z 4 0. Troba la projecció ortogonal de P sobre el pla , que és un punt P’. Comprova que la distància de P a P’ és la mateixa que de P al pla .

d P( , )( ) ( )

( )

2 5 2 2 4

2 1 13 6

2 2 2

Trobem la projecció ortogonal P' del punt P sobre el pla .

Recta perpendicular a que passa per P.

P

nr

x

y( , , )

( , , ):

5 2 2

2 1 1

5 2

2

zz 2W

Intersecció entre la recta i el pla.

x

y

z

x y z

x5 2

2

2

2 4 0

1

1

1

3

1 1 1y

zP'( , , )

Trobem la distància de P a P':

d P P PP d P P d P( , ) ( , , ) ( , ) ( , )' ' '6 3 3 54 3 6W

332

Producte escalar

118 Troba dos punts, un de la recta rx pyz p

:325

i un altre de la recta

s x y z: 62

113

22

que es trobin a la mínima distància.

Determinem un punt P r i un punt Q s amb la condició que el vector PQW

sigui perpendicular a les dues rectes.

P p p

Q q q qP

( , , )

( , , )

3 2 5

6 2 11 3 2 2QQ p q q p q( , , )2 9 3 13 2 3W

PQ u p q q p q pr ( , , ) ( , , )2 9 3 13 2 3 1 0 1 12 2 0 6pW W

PQ v p q q p q qs ( , , ) ( , , )2 9 3 13 2 3 2 3 2 17 551 0

3q

W W

Per tant, els punts que busquem són P(3, 2, 1) i Q(0, 2, 4).

119 a) Calcula les equacions implícites de la recta r1 que passa pels punts A(1, 2, 3) i B (2, 2, 3).

b) Calcula l’equació general del pla que passa pels punts A, B i C (2, 2, 4).c) Quants plans diferents es poden formar amb els punts A, B, C i D (1, 2, 4)?

Justifica la resposta.d) Prova que els punts A, B, C i D anteriors formen un quadrat i calcula’n l’àrea.(Activitat real de Selectivitat)

a)A

ABr

x

y

z

( , , )

( , , ):

1 2 3

1 0 0

1

2

3

ry

z:

2

3W

b)

A

AB

AC

( , , )

( , , )

( , , )

1 2 3

1 0 0

1 0 1

::

x y z

y

1 2 3

1 0 0

1 0 1

2 0W

W

c) : y DD

2 0 2 2 01 2 4( , , )

Amb els punts A, B, C i D només es pot formar un pla.

d)

AB BC

CD DA

( , , ) ( , , )

( , , ) ( , , )

1 0 0 0 0 1

1 0 0 0 0 1

W

W

W

W

Aquests vectors són de mòdul 1 i, a més, paral·lels dos a dos; per tant,

generen un quadrat d’àrea 1.

120 D’una recta r se sap que està continguda en el pla d’equació x y 0, que A(0, 0, 0) pertany a r, i que el vector que uneix A i B (1, 0, 1) és perpendicular a r. Determina la recta r i calcula la distància entre r i el pla paral·lel a que passa per B.(Activitat real de Selectivitat)

El vector director de la recta r, uWr (a, b, c), és perpendicular al vector normal

del pla , nW (1, 1, 0)

uWr nW (a, b, c) (1, 1, 0) a b a b0

El vector director de la recta és de la forma uWr (a, a, c).

333

5SOLUCIONARI

El vector director de la recta r, uWr (a, a, c), és perpendicular al vector ABW (1, 0, 1).

uWr ABW (a, a, c) (1, 0, 1) a c a c0

El vector director de la recta és de la forma uWr (a, a, a). Per exemple, per a a 1,

el vector uWr (1, 1, 1) és un vector director de r.

Per tant, l’equació de la recta r és:

Punt:

Vector director:

A

u r

( , , )

( , , )

0 0 0

1 1 1r

x y z:

1 1 1W

El pla ' és paral·lel al pla ': x y D 0

B D D( , , )1 0 1 1 0 0 1'': x y 1 0

Com que r és paral·lela a ', la distància de la recta al pla és la mateixa

que la distància del punt A(0, 0, 0) r al pla '.

d r d A( , ) ( , )' '0 0 1

1 1

1

2

2

2

121 a) Calcula l’equació de la recta que passa per l’origen i és perpendicular al pla : x y z 3.

b) Troba el punt de tall de la recta amb el pla .

c) Troba el punt de la recta xyz

31 2

la distància del qual al punt P (1, 0, 2)

sigui 5 .

(Activitat real de Selectivitat)

a)Punt:

Vector director:

A

n

( , , )

( , , )

0 0 0

1 1 1r

x

y

z

:W

b)

x

y

z

x y z

x

y

z

3

1

1

1

1

1 1 1P ( , , )

c) Un punt genèric de la recta r és Q ( , , )3 1 2 .

d P Q PQ( , ) ( ) ( ) ( ) ( )5 1 3 2 1 5

6

2 2 2 2

2 112 11 5 1

W

Per tant, el punt que busquem és Q ( , , )1 2 3 .

122 La trajectòria d’un projectil ve donada per la recta: rxyz

:231 2

.

a) Estudia si el projectil impacta amb la superfície determinada pel pla 3 x y z 0.b) Calcula el punt d’impacte i la distància recorreguda pel projectil des del punt

inicial P (2, 3, 1) fins al punt d’impacte.

(Activitat real de Selectivitat)

334

Producte escalar

a)

x

y

z

x y z

2

3

1 2

3 0

xx

y

z

0

5

5

2

Aquest sistema té solució; per tant, el projectil impacta amb la superfície

determinada pel pla.

b) El punt d’impacte és Q (0, 5, 5).

d (P, Q ) PQ ( , , )2 2 4 24 2 6W

123 Troba els punts de la recta r : x 1 y 2 z que equidisten dels plans 1: 4 x 3 z 1 0 i 2: 3 x 4 y 1 0.(Activitat real de Selectivitat)

Un punt genèric de la recta r és P ( , , )1 2 .

d P d P( , ) ( , )1 2

4 4 3 1

16 9

3 3 8 4 1

9 116

3 7 63

2

3 7 63

8

Els punts són Q Q1 2

5

2

1

2

3

2

11

8

13

8

3

8, , , ,i .

124 Calcula la distància entre els plans x y z 5 i x y z 1 0.(Activitat real de Selectivitat)

Els plans són paral·lels. La distància entre els dos plans és la mateixa

que la distància entre un punt del primer pla al segon pla.

A x y z( , , ) :3 2 0 5

d d A( , ) ( , )' '3 2 0 1

1 1 1

4

3

4 3

3

125 Considera P i Q els punts de l’espai amb coordenades P (0, 0, 0) i Q (0, 1, 2). Troba la condició que ha de complir un punt de coordenades A(x, y, z) perquè la distància des de A fins a P sigui igual que la distància des de A fins a Q.El conjunt de tots els punts que satisfan aquesta condició forma un pla? Raona la resposta. (Activitat real de Selectivitat)

PA x y z

QA x y zPA QA

( , , )

( , , )1 2WW

W

W

x y z x y z y z2 2 2 2 2 21 2 2 4 5( ) ( )

La condició que han de complir les coordenades del punt A(x, y, z) és que 2y 4z 5.

El conjunt de tots els punts que compleixen aquesta condició

és el pla : 2 y 4 z 5.

335

5SOLUCIONARI

126 Un helicòpter situat al punt P (1, 2, 1) vol aterrar al pla : x y 3z 0.a) Calcula l’equació en forma contínua

de la recta de la trajectòria que el dugui al punt més proper del pla .

b) Calcula aquest punt.c) Calcula la distància que ha de recórrer.

(Activitat real de Selectivitat)

a) La trajectòria és la recta perpendicular al pla que passa per P.

Punt:

Vector director:

P

n

( , , )

( , , )

1 2 1

1 1 3r

x

y

z

:

1

2

1 3W

b)

x

y

z

x y z

x

1

2

1 3

3 0

5

11

16

11

7

11

6

11

y

z

Q5

11

16

11

7

11, ,

c) d P Q PQ( , ) , ,6

11

6

11

18

11

6 11

11

W

127 Troba de manera raonada les equacions dels dos plans paral·lels al pla d’equació : 12 x 3 y 4z 7 que disten 6 unitats de .

(Activitat real de Selectivitat)

1 paral·lel al pla 1 112 3 4 0: x y z D

2 paral·lel al pla 2 212 3 4 0: x y z D

Com que A (0, 1, 1) :

d d AD

( , ) ( , )( )

(1 1

2 2

12 0 3 1 4 1

12 3 4 ))26

DD

D7 78

71

85

Els plans són: 1 212 3 4 71 0 12 3 4 85 0: :x y z x y zi

128 Troba l’equació general del pla que talla els eixos de coordenades en els punts (1, 0, 0), (0, 2, 0) i (0, 0, 3).

Troba els punts de la recta x y z que estan a distància 17

d’aquest pla.

(Activitat real de Selectivitat)

336

Producte escalar

Si iA B C( , , ), ( , , ) ( , , ):1 0 0 0 2 0 0 0 3

A

AB

AC

( , , )

( , , )

( , , )

1 0 0

1 2 0

1 0 3

::

x y z

x y z

1

1 2 0

1 0 3

6 3 2 6 0W

W

Un punt genèric de la recta r : x y z és de la forma P ( , , ).

d P( , )6 3 2 6

36 9 4

1

711 6 1

1 6

111

7

11

1 6

11

5

11

Els punts que busquem són: Q1

7

11

7

11

7

11

5

11

5

11

5

11, , , ,i Q2 .

129 a) Troba l’equació del pla perpendicular a la recta x y zx y z

3 02 1 0

que passa pel punt (5, 0, 10).

b) Troba la distància d’aquest pla al punt (2, 1, 0).

(Activitat real de Selectivitat)

a) El vector director de la recta és el vector normal del pla que busquem.

rx y z

x y zr

x

y

z

: :3 0

2 1 0

7 3

4 2

3 1 2u r ( , , )W

Punt:

Vector normal:

P

ur

( , , )

( , , )

5 0 10

3 1 2: 3 2 5 0x y z

W

b) d ( )( , , )( )

2 1 03 2 1 2 0 5

3 1 2

10

14

5 1

2 2 2

44

7

130 Troba la distància del pla : 4 x 10 y 2 z 1 al pla :xyz

2 3.

(Activitat real de Selectivitat)

En determinem les posicions relatives.

A

u

v

( , , )

( , , )

( , , )

:

0 0 0

2 1 1

3 1 1

xx y z

x y z n2 1 1

3 1 1

2 5 0 2 5 1( , , )W

W

: ( , , )4 10 2 1 4 10 2x y z nW

nW i nW són proporcionals → i són paral·lels.

La distància entre els dos plans és igual a la distància entre un punt A(0, 0, 0)

i el pla .

d d A( , ) ( , )4 0 10 0 2 0 1

16 100 4

1

120

300

60

337

5SOLUCIONARI

131 Construeix un triangle equilàter de manera que dos dels vèrtexs siguin P (1, 2, 3) i Q ( 1, 4, 3) i el tercer vèrtex R estigui en el pla

: x y z 2. Quina àrea té?

(Activitat real de Selectivitat)

R a b c a b c( , , ) 2

d P Q d P R d Q R( , ) ( , ) ( , )

( ) ( ) ( ) ( )

( ) (

2 2 0 1 2 3

1

2 2 2 2 2 2

2

a b c

a b 4 32 2) ( )c

8 1 2 3

8 1 4 3

2 2 2

2 2

( ) ( ) ( )

( ) ( ) (

a b c

a b c ))2

Tenim el sistema d’equacions:

a b c

a b c

a b

2

1 2 3 8

1 4

2 2 2

2 2

( ) ( ) ( )

( ) ( ) (( )

, ,

,c

a b c

a3 8

1 2 1

5

32

1 1 1

2 bb c2 2

4

3

7

3,

Obtenim dues solucions: R R1 21 2 15

3

4

3

7

3( , , ) , ,i

Calculem l’àrea:

Base PQ 8 2 2W

Com que els angles d’un triangle equilàter valen 60º:

Altura 8 60 83

26sin °

Àrea2 2 6

22 3

132 Considera les rectes:

r x y z sx yx z

: :1

11

22

3 5 03 8 0

a) Troba l’equació del pla que conté r i és paral·lel a s.b) Calcula la distància entre el pla i la recta s.

(Activitat real de Selectivitat)

a) sx y

x zs

x

y

z

:3 5 0

3 8 0

8 3

1: u Qs s( , , ) ( , , )3 1 1 8 1 0W

rx y z

v Pr r: ( , , ) ( , , )1

1

1

2

21 1 2 0 1 2W

RP

Q

338

Producte escalar

El pla té com a vectors directors uWs i vWr i passa

pel punt Pr (0, 1, 2).

P

v

u

r

r

s

( , , )

( , , )

( , , )

0 1 2

1 1 2

3 1 1

::

x y z

x y z

1 2

1 1 2

3 1 1

3 5 4 13 0W

W

b) La distància de la recta s al pla és la distància d’un punt de la recta s

al pla.

Prenem Q ss ( , , )8 1 0 .

d s d Qs( , ) ( , )3 8 5 1 4 0 13

9 25 16

32

50

116 2

5

133 Considera els punts A( , 2, ), B (2, , 0) i C ( , 0, 2).a) Hi ha algun valor de per al qual els punts A, B i C estiguin alineats?b) Comprova que si A, B i C no estan alineats el triangle que formen

és isòsceles.c) Calcula l’equació del pla que conté el triangle ABC per al valor

de 0 i troba la distància d’aquest pla a l’origen de coordenades.

(Activitat real de Selectivitat)

a) Perquè A, B i C estiguin alineats, ABW i ACW han de ser proporcionals.

AB

AC

( , , )

( , , )

2 2

0 2 2

2

0

2

2 2

W

W

→ No hi ha solució.

Els tres punts no estan alineats.

b) AB AB( , , ) ( ) ( ) ( )2 2 2 22 2 2

33 8

0 2 2 0 2 2 8

2

2

2 2 2AC AC

BC

( , , ) ( )

( , , 2 2 2 3 82 2 2 2) ( ) ( )BC

W W

W W

W W

Per tant, AB B RWW . El triangle sempre és isòsceles.

c) El pla passa per A(0, 2, 0) i té per vectors directors ABW ( , , )2 2 0

i ACW ( , , )0 2 2 :

A

AB

AC

( , , )

( , , )

( , , )

0 2 0

2 2 0

0 2 2

:

:

x y z

x y z

x y z

2

2 2 0

0 2 2

4 4 4 8 0

2 0

W

W

d O( , )2

1 1 1

2

3

2 3

3

339

5SOLUCIONARI

134 La recta x y z13

22

talla els tres plans coordenats en tres punts.

Determina les coordenades d’aquests punts, les distàncies que hi ha entre cada parell de punts i indica quin és el que es troba enmig dels altres dos. (Activitat real de Selectivitat)

Tall amb l’eix X.

xy z y

z

y

z0 0

1

3

2

2

1 0

2 0

1

2P ( , , )0 1 2

Tall amb l’eix Y.

y xz x

z

x

z

01

3

2

2

1

3

6 3 2

1

3

4

33

1

30

4

3Q , ,

Tall amb l’eix Z.

z xy x

y

x

y0

1

3

2

2

1

1 3

1

2R ( , , )1 2 0

La distància entre cada parell de punts són els mòduls dels vectors

que determinen.

PQ PQ1

31

2

3

14

3, , WW

PR PR( , , )1 3 2 14WW

QR QR2

32

4

3

2 14

3, , WW

La distància més llarga és la que hi ha des del punt P fins al punt R;

per tant, el punt Q és el que es troba enmig dels altres dos.

135 Considera els punts A(0, 1, 0) i B (1, 0, 1). a) Escriu l’equació que han de verificar els punts X ( x, y, z ) que equidisten

de A i B.b) Determina l’equació que verifiquen els punts X ( x, y, z ) la distància dels quals a A

és igual a la distància de A a B.c) Escriu les equacions paramètriques de la recta formada pels punts C ( x, y, z )

del pla x y z 3, de tal manera que el triangle ABC és rectangle amb l’angle recte al vèrtex A.

(Activitat real de Selectivitat)

a) d A X d B X( , ) ( , )

x y z x y z x y z2 2 2 2 2 21 1 1 2 2 2 1 0( ) ( ) ( )

L’equació que verifiquen els punts X és un pla.

b) d A X d A B( , ) ( , )

x y z x y z2 2 2 2 2 21 3 1 3( ) ( )

Es tracta de l’equació de l’esfera de centre el punt A(0, 1, 0) i radi 3 .

340

Producte escalar

c) C x y z C: ( , , )3 3

El triangle ABC és rectangle amb angle recte en A:

ABW ( , , )1 1 1 i ACW ( , , )1 3 són perpendiculars.

ABW ACW 0 1 3 0 2

Així doncs, els punts que busquem són de la forma C ( , , )2 1 . És a dir,

pertanyen a la recta:

r

x

y

z

: 2

1

136 Demostra que les rectes:

Lx ty tz t

Lx yx

1 2

13 33

3 4 0: :

zz 4 0

són paral·leles. Troba l’equació del pla paral·lel al determinat per aquestes rectes

i que en disti 6 .

(Activitat real de Selectivitat)

L

x t

y t

z t

u P1 1

1

3 3

3

1 3 1: ( , , ) 11 1 3 3( , , )W

Lx y

x zL

x

y2 2

3 4 0

4 0

4

8 3: :

z

v Q2 21 3 1 4 8 0( , , ) ( , , )W

Les dues rectes tenen el mateix vector director; per tant, són paral·leles.

El pla determinat per L1 i L2 té com a vectors directors uW1 i P1 WQ2, i passa

pel punt P1 1 3 3( , , ).

P

u

P Q

1

1

1 2

1 3 3

1 3 1

5 11 3

( , , )

( , , )

( , , )

WW

: :

x y z

x y z x y

1 3 3

1 3 1

5 11 3

2 2 4 8 0 2 zz 4 0

Un pla paral·lel a ha de ser de la forma ': x y z D2 0; per tant:

d d PD D

( , ) ( , )' '1

1 3 6

1 1 4

4

66

4 66 4 2

6 4 10D

D

D

Els plans que busquem són 1 22 2 0 2 10 0: :x y z x y zi .

341

5SOLUCIONARI

137 El pla és el que passa pels punts P1( 3, 0, 0), P2 (1, 1, 1) i P3 ( 1, 0, 1).Troba els dos punts de la recta:

r x y z:1

10

21

que estan a distància 1 del pla .

(Activitat real de Selectivitat)

El pla està determinat pel punt P1 3 0 0( , , ) i pels vectors

directors P1 WP3 ( , , )2 0 1 .

P

P P

P P

1

1 2

1 3

3 0 0

4 1 1

2 0 1

( , , )

( , , )

( , , )

:

x y z

x y z

3

4 1 1

2 0 1

2 2 3 0W

W

rx y z

r

x

y

z

: :1

1

0

2

11

2

Un punt genèric de r és A( , , )1 2 .

d A( , )( )2 1 2 2 3

1 4 4

1

31

1 33 1 4

3 1 2

Els punts que busquem són A1 4 1 6 2 1 0( , , ) ( , , )i A2 .

PREPARA LA SELECTIVITAT(Activitats de Selectivitat)

1 Et demanem:

a) Defineix el mòdul d’un vector. Propietats.

b) Determina els valors de a i b (a 0) perquè els vectors Wu (a, b, b), Wv (b, a, b) i Ww (b, b, a) siguin unitaris i ortogonals dos a dos.

a) El mòdul d’un vector és l’arrel quadrada positiva del producte escalar del vector

per si mateix.

Propietats: 1 0

2

3

.

. ,

.

u

k u k u k

u v u v

R

W

W

W W W W

W

b) u v w a b a b1 2 1 2 12 2 2 2W W W

uW vW ab ba bb ab b2 02

uW wW bb ab ba ab b2 02

342

Producte escalar

uW wW ab bb ba ab b2 02

Resolem el sistema:

a b

ab b

a b

b a b

2 2

2

2 22 1

2 0

2 1

2 0( )

b a

b a

a b

a b

0 1

2

1

3

2

3

1

3

2

3

,

,

,

Com que a > 0, tenim dues solucions:

a b

a b

1 0

1

3

2

3

,

,

2 Determina una constant a perquè el pla d’equació : ax y z 2 formi un angle

de 3

radians amb el pla z 0.

: ax y z 2 nW ( a, 1, 1)

': z 0 nW ' ( 0, 0, 1)

cos3

1 1 0 0 1

1 1

1

22 2 2 2

( , , ) ( , , )a

a a

1

2

1

22 4 2

2

2

aa a

3 Troba els punts de la recta x y z21

11 2

que es troben a distància 1

del pla : 2 x 2y z 5 0.

: 2 2 5 0x y z nW ( 2, 2, 1)

rx y z

r

x

y

z

: :2

1

1

1 2

2

1

2

uWr (1, 1, 2)

Un punt genèric de r és A( , , )2 1 2 .

d A( , )( ) ( ) ( )2 2 2 1 2 5

4 4 11

2 3

31 2 3 3

3 3

23

3 3

20

Els punts que busquem són A A1 21 2 6 2 1 0( , , ) ( , , )i .

343

5SOLUCIONARI

4 Troba les equacions dels plans paral·lels a : 2 x y 2 z 3 situats a 6 unitats de distància d’aquest.

' paral·lel a ': 2 2 0x y z D

Si prenem P ( , , )1 1 0 , tenim que:

d d PD

( , ) ( , )( )

( )' '

2 1 1 2 0

2 1 26

2 2 2

3

36 3 18

18 3 15

18 3 21

DD

D

D

Per tant, els plans són 1 2 2 15 0: x y z i 2 2 2 21 0: x y z .

5 Es demana:

a) Trobar un punt de la recta rx ty tz

:1 2

1 equidistant dels punts P ( 1, 2, 1)

i Q (0, 3, 1).

b) Calcular l’equació implícita d’un pla de manera que el simètric del punt P respecte del pla sigui el punt Q.

Un punt genèric de la recta r és R (1 2t, t, 1).

PRW ( , , )2 2 2 2t t

QRW ( , , )1 2 3 2t t

PR QR t t t t( ) ( ) ( ) ( ) ( )2 2 2 2 2 1 32 2 2 2 2 ( )

( ) ( ) ( ) ( )

2

2 2 2 2 1 3 1

2

2 2 2 2t t t t t

WW

Així doncs, un punt de la recta r equidistant dels punts P i Q és R (3, 1, 1).

6 Donats els punts P (1, 1, 3) i Q (0, 1, 0), es demana:

a) Trobar tots els punts R de tal manera que la distància entre P i R sigui igual a la distància entre Q i R. Descriu aquest conjunt de punts.

b) Trobar tots els punts S continguts a la recta que passa per P i Q que verifiquen dist (P, S ) 2 dist (Q, S ), en què «dist» significa distància.

a) Busquem els punts R (a, b, c) que verifiquin que d ( P, R ) d ( Q, R ).

PR QR a b c a b c( ) ( ) ( ) ( )1 1 3 12 2 2 2 2 2WW

( ) ( )a c a c a c

a

b

c

1 3 3 5

5 32 2 2 2

Els punts que compleixen aquesta condició són de la forma R ( , , )5 3

i configuren el pla d’equació :

x

y

z

5 3

.

344

Producte escalar

b) Trobem la recta que passa per P i Q.

Punt:

Vector director:

P

PQ

( , , )

( , , )

1 1 3

1 0 3r

x

y

z

:

1

1

3 3W

Un punt genèric de la recta és S ( , , )1 1 3 3 .

PS PS( , , )0 3 10 2W W

QS QS( , , )1 0 3 10 20 102W W

d P S d Q S( , ) ( , )2 10 2 10 20 10

3 8 4

2 2

2 00

2

2

3

Els punts que busquem són S S1 21 1 31

31 1( , , ) , ,i .

7 Troba les equacions de la recta r que passa per P (2, 1, 1), està continguda

en el pla : x 2 y 3 z 1, i és perpendicular a la recta sx zy z

:2 3

4.

Escrivim la recta s en forma paramètrica:

x

y

z

3 2

4 uWs (2, 1, 1)

Determinem el pla ' perpendicular al pla que passi pel punt P.

':

x y z

x y z

2 1 1

2 1 1

1 2 3

5 3 6 0

La recta r ha de ser rx y z

x y z:

2 3 1

2 8

8 Calcula la distància del punt P (1, 1, 3) a la recta r.

rxyz

:111 2

Calculem el pla que passa per P i és perpendicular a r.

Punt:

Vector normal:

P

u r

( , , )

( , , )

1 1 3

1 1 2: x y z2 8 0

W

345

5SOLUCIONARI

Determinem el punt de tall Q de la recta r i el pla .

x

y

z

x y z

x1

1

1 2

2 8 0

2

yy

zQ

0

3

1

2 0 3( , , )

La distància del punt P a la recta r és la distància de P a Q.

d P Q( , ) ( ) ( ) ( )2 1 0 1 3 3 22 2 2

9 Determina els punts de la recta r d’equacions rx

y z:0

1 32

que equidisten

del pla d’equació : x z 1 i del pla ’ d’equació ’ : y z 3.

s

x

yz s

x

y

z

: :

0

13

2

0

1 2

Un punt genèric de la recta s és P ( , , )0 1 2 .

d P d P( , ) ( , )'

1 2 1

1 1

1 2 3

1 12 2 2 2

( )

( )

2

2

4

22 4

4

3

4

3

Els punts que busquem són P P1 204

3

5

30

4

3

11

3, , , ,i .

L I T E R AT U R A Y M AT E M ÁT I C A S

Les cendres de l’ÀngelaEl senyor O’Neill és el mestre del quart curs de l’escola. L’anomenem «Puntet» perquè és petit com un punt. Ens fa classe a l’única aula on hi ha tarima, perquè així pot ser més alt que nosaltres, pot amenaçar-nos amb la seva palmeta de freixe i pelar la poma a la vista de tots. El primer dia del curs, al setembre, escriu a la pissarra tres paraules que seguiran allà la resta del curs: Euclides, geometria, idiota. Diu que si agafa algun nen tocant aquestes paraules, aquell nen passarà la resta de la seva vida amb una sola mà. Diu que qualsevol que no entengui els teoremes d’Euclides és idiota. «Bé, repetiu: qualsevol que no entengui els teoremes d’Euclides és idiota.» Naturalment, tots sabem el que és un idiota, ja que els mestres ens diuen constantment que ho som.

En Brendan Quigley aixeca la mà.

–Senyor, què és un teorema i què és un Euclides?

Esperem que en «Puntet» reprengui en Brendan com ho fan tots els mestres quan se’ls fa una pregunta, però ell mira en Brendan amb un petit somriure.

–Mireu, aquí tenim un nen que no té una sola pregunta, sinó dues. Com et dius?

–Brendan Quigley, senyor.

–Aquest és un nen que arribarà lluny. On arribarà, nens?

–Lluny, senyor.

–I tant que sí. El nen que vol saber alguna cosa de la gràcia, de l’ele-gància i de la bellesa d’Euclides no pot fer res més que pujar a la vida. Què farà aquest nen a la vida, nens?

–Pujar, senyor.

–Sense Euclides, nens, les matemàtiques serien una cosa mesquina i insegura. Sense Euclides no seríem capaços d’anar d’aquí cap allà. Sen-se Euclides, la bicicleta no tindria rodes. Sense Euclides, sant Josep no podria haver estat fuster, ja que la fusteria és geometria i la geometria és fusteria. Sense Euclides, aquesta mateixa escola no es podria haver construït.

FRANK MCCOURT

Números realesProductes vectorial i mixt6

347

6SOLUCIONARI

Les cendres de l’Àngela Frank McCourt

Aquesta novel·la narra la vida a Irlanda abans i durant la Segona Guerra Mundial. No conté més

referències a les matemàtiques que les que surten a la magistral descripció de la primera classe

que imparteix el professor «Puntet».

–Fotut Euclides –murmura darrere meu Paddy Clohessy. En «Puntet» li diu amb veu tallant:

–Tu, noi: com et dius?

–Clohessy, senyor.

–Ah, el noi que vola amb una ala. I de nom?

–Paddy.

–Paddy, i què més?

–Paddy, senyor.

–I, què li deies a en McCourt, Paddy?

–Li deia que ens hauríem de posar de genolls i donar gràcies a Déu que hagi existit Euclides.

–Ja ho crec, Clohessy. Veig la mentida podrida als teus llavis. Què veig, nois?

–La mentida, senyor.

–I com està la mentida, nois?

–Podrida, senyor.

–On, nois, on?

–Als seus llavis, senyor.

–Euclides era grec, nois. Què és un grec, Clohessy?

–Una mena d’estranger, senyor.

–Clohessy, ets retardat mental. I doncs, Brendan, sens dubte tu deus saber què és un grec, oi?

–Sí, senyor. Euclides era grec.

En «Puntet» li dirigeix un somriure. Diu a Clohessy que ha d’imitar el model de Quigley, que sap el que és un grec. Traça dues línies, l’una al costat de l’altra, i ens diu que són línies paral·leles, i que el que tenen de màgic i de misteriós és que no es troben mai, encara que es prolonguin fins a l’infinit, encara que es prolonguin fins a les espatlles de Déu, i això, nois, és molt lluny, tot i que ara hi ha un jueu alemany que està posant tot el món de potes enlaire amb les seves idees sobre les línies paral·leles.

Escoltem en «Puntet» i ens preguntem què té a veure tot això amb l’estat del món, amb el fet que els alemanys ho envaeixin tot i bombardegin tot allò que s’aguanta dret. No l’hi podem preguntar nosaltres, però podem fer que en Brendan Quigley li pregunti. És ben clar que en Brendan és la nineta dels ulls del mestre, i això significa que li pot fer les preguntes que vulgui.

La geometria que s’estudia en aquests temes es diu euclidiana, perquè s’ajusta a les idees que va exposar Euclides en un dels llibres més importants de tota la història: els Elements. Però també existeixen geometries no euclidianes. Investiga les diferències que hi ha entre unes i altres, i escriu un treball breu amb les dades que n’hagis obtingut.

Rep el nom de geometria no euclidiana la branca de la geometria que conté postulats

i propietats que difereixen en algun punt dels establerts per Euclides.

348

Productes vectorial i mixt

ABANS DE COMENÇAR... RECORDA

001 Calcula els determinants següents:

a) b) c)5 0 22 1 46 0 1

10 4 12 0 2

2 33 3 01 0

x y x y z

4

a)

5 0 2

2 1 4

6 0 1

17

b)

x 1 y

0 4 1

2 0 2

8x 8y 2

c)

x 2 y z 3

3 3 0

1 0 4

12x 12y 3z 33

002 Troba un punt i un vector director de la recta següent:

rx y z

:1

11

11

2

Un punt és P(1, 1, 1) i un vector director és Wu

(1, 1, 2).

003 Calcula un punt i un vector normal d’aquest pla:

: 2 3 5 0x y z

Un punt és P( 4, 1, 0) i un vector normal és Wn

(2, 3, 1).

004 Dibuixa el lloc geomètric dels punts que equidisten de dues rectes:

a) Que es tallen. b) Que són paral·leles.

a) b)

005 Troba el lloc geomètric dels punts del pla la suma de coordenades cartesianes dels quals és 10.

El lloc geomètric és el pla d’equació: x y z 10 0 .

r

s

r

s

349

6SOLUCIONARI

ACTIVITATS

001 Calcula Wu Wv, si saps que Wu ( 1, 1, 0), Wv 2 i que l’angle que formen és 30°

Wu Wv Wu Wv sin ( )1 1 0 21

222 2 2

002 Si Wu ( 1, 1, 0), Wv (0, 1, 1), calcula l’angle que formen i troba l’àrea del paral·lelogram que formen.

Calculem els mòduls:

Wu ( )1 1 0 22 2 2

Wv 0 1 1 22 2 2( )

I, mitjançant el producte escalar, calculem l’angle:

cos| | | |

( ) ( )u v

u v

1 0 1 1 0 1

2 2

1

21220°

W W

WW

Finalment, trobem l’àrea del paral·lelogram:

Àrea Wu Wv Wu Wv sin sin2 2 120 23

23°

003 Si Wu Wv 2Wt i Wu Ww Wt , calcula:

a) Wu ( Ww Wv ) c) 3Wu ( 2 Ww Wv )b) Wu (2 Ww Wv ) d) (2 Ww Wv ) ( Wu )

a) Wu (Ww Wv ) Wu Ww Wu Wv Wt 2Wt 3Wt

b) Wu (2Ww Wv ) Wu ( 2Ww ) Wu Wv 2(Wu Ww ) Wu Wv 2Wt 2Wt W0

c) 3Wu ( 2Ww Wv ) 6(Wu Ww ) 3(Wu Wv ) 6Wt 6Wt 12Wt

d) (2Ww Wv ) ( Wu ) 2(Ww Wu ) (Wv Wu ) 2Wt 2Wt W0

004 Si Wu (0, 1, 0), Wv ( 1, 2, 0) i Ww ( 4, 1, 1), troba:

a) Wu ( Ww Wv ) c) 3 Wu ( 2W Ww Wv )b) Wu (2 Ww Wv ) d) (2W Ww Wv ) ( Wu )

a) Wu (Ww Wv )

(0, 1, 0) ( 3, 3, 1)

i j k

0 1 0

3 3 1

W W W

Wi 3Wk ( 1, 0, 3)

b) Wu (2Ww Wv ) (0, 1, 0) ( 9, 0, 2)

i j k

0 1 0

9 0 2

W W W

2Wi 9Wk (2, 0, 9)

c) 3Wu ( 2Ww Wv )

(0, 3, 0) (7, 4, 2)

i j k

0 3 0

7 4 2

W W W

6Wi 21Wk (6, 0, 21)

d) (2Ww Wv ) ( Wu ) ( 9, 0, 2) (0, 1, 0)

i j k

9 0 2

0 1 0

W W W

2Wi 9Wk ( 2, 0, 9)

350

Productes vectorial i mixt

005 Troba el vector normal al pla que passa per:

a) A(1, 1, 1) B(3, 1, 0) C( 1, 0, 1)

b) D(0, 0, 0) E(2, 2, 2) F(0, 1, 2)

a) El vector normal és:

WAB WAC

i j k

2 0 1

2 1 0

W W WWi 2Wj 2Wk ( 1, 2, 2)

b) El vector normal és:

WDE WDF i j k

2 2 2

0 1 2

W W W

6Wi 4Wj 2Wk ( 6, 4, 2)

006 Troba una base ortogonal que contingui Wu (2, 1, 0).

Agafem un vector no proporcional a Wu , per exemple (0, 0, 1), i trobem el segon

vector de la base, que anomenem Wv .

Wv (2, 1, 0) (0, 0, 1)

i j k

2 1 0

0 0 1

W W W

Wi 2Wj ( 1, 2, 0)

Trobem el tercer vector de la base, que anomenem Ww .

Ww Wu Wv

(2, 1, 0) ( 1, 2, 0)

i j k

2 1 0

1 2 0

5Wk (0, 0, 5)

Una base @ {(2, 1, 0), ( 1, 2, 0), (0, 0, 5)} és ortoganal.

007 Troba una base de vectors ortogonals en l’espai. Quantes bases hi ha? Raona la resposta.

Qualsevol conjunt de tres vectors perpendiculars entre ells formen una base de

l’espai. Com que hi ha infinits conjunts de tres vectors perpendiculars entre ells, hi

ha infinites bases.

Un exemple de base ortogonal és la base canònica: @ {(1, 0, 0), (0, 1, 0), (0, 0, 1)}

008 Calcula el vector director de la recta determinada per cada parella de plans.

a) 1: y z 1 0 2: 3x z 6 0

b) 1: 2x y 3z 0 2: 3x y 5z 0

a)

n1 (0, 1, 1)

n2 (3, 0, 1)n1 n2

i j k

0 1 1

3 0 1

i 3 j 3k

W

WWW

W

W

WW W W

El vector director de la recta és Wv

( 1, 3, 3).

b)

n1 (2, 1, 3)

n2 (3, 1, 5)n1 n2

i j k

2 1 3

3 1 5

8i 19 j k

W

WWW

W

W

WW W W

El vector director de la recta és Wv

( 8, 19, 1).

351

6SOLUCIONARI

009 Donada la recta:

rx y z

:1

11

12

,

troba’n l’equació implícita, i comprova que el producte vectorial dels vectors normals dels plans és proporcional a Wv (1, 1, 2).

Trobem l’equació implícita de la recta r.

xy 1

1

xz 1

2

x y 1 0

2x z 1 0

Els vectors normals dels plans són:

Wn1 (1, 1, 0)

Wn2 (2, 0, 1)

Wn1 Wn2

i j k

1 1 0

2 0 1

i j 2k ( 1, 1, 2)

W W WW W W

El vector Wn1 Wn2 ( 1, 1, 2) és proporcional a Wv (1, 1, 2).

010 Troba l’àrea tancada als triangles que tenen com a vèrtexs:

a) A(0, 0, 0) B( 1, 2, 1) C ( 1, 1, 1)

b) A(3, 0, 0) B(0, 2, 0) C (0, 0, 1)

a) Dos costats del triangle són:

WAB ( 1, 2, 1)

WAC ( 1, 1, 1)

WAB WAC

i j k

1 2 1

1 1 1

i 2 j 3k ( 1, 2, 3)

W W WW W W

WAB WAC ( 1)2 ( 2)2 32 14

Àrea

AB AC

2

14

2

W W

b) Dos costats del triangle són:

WAB ( 3, 2, 0)

WAC ( 3, 0, 1)

WAB WAC

i j k

3 2 0

3 0 1

2i 3 j 6k (2, 3, 6)

WW

WW

WW

WAB WAC 22 32 62 49 7

Àrea

AB AC

2

7

2

W W

352

Productes vectorial i mixt

011 Calcula l’àrea del triangle que té com a vèrtexs:A(2, 1, 3) B(3, 0, 1) C( 1, 2, 0)

Comprova que el resultat és independent dels vectors que triïs per trobar l’àrea.

WAB (1, 1, 2) WAC ( 3, 3, 3)

WAB WAC

i j k

1 1 2

3 3 3

3i 9 j 6k ( 3, 9, 6)

WW

WW

WW

WAB WAC ( 3)2 92 ( 6)2 126 3 14

Àrea

AB AC

2

3 14

2

W W

Comprovem que el resultat és independent dels vectors triats.

L’àrea del triangle de costats WBA ( 1, 1, 2) i WBC ( 4, 2, 1) és:

WBA WBC i j k

1 1 2

4 2 1

3i 9 j 6k

WW

WW

WW

WBA WBC 126 3 14

Àrea

BA BC

2

3 14

2

W W

L’àrea del triangle de costats WCA (3, 3, 3) i WCB (4, 2, 1) és:

WCA WCB

i j k

3 3 3

4 2 1

3i 9 j 6k

WW

WW

WW

WCA WCB 126 3 14

Àrea

CA CB

2

3 14

2

W W

L’àrea del triangle no varia.

012 Troba la distància del punt P (1, 0, 2) a les rectes:

a) r x yy z

: 03 0

b) sx y z

:1

23

52

a) Calculem un vector director Wvr i un punt A de la recta.

vr

i j k

1 1 0

0 1 3

3i 3 j k ( 3, 3, 1)

W W

W

WW W W

A(0, 0, 0) r WAP (1, 0, 2)

Wvr WAP

i j k

3 3 1

1 0 2

6i 7 j 3k ( 6, 7, 3)

W W WW W W

353

6SOLUCIONARI

Wvr 9 9 1 19 Wvr WAP 36 49 9 94

La distància del punt P a la recta r és:

d(P , r )vr AP

vr

94

19

1.786

19

W

W

W

b) Calculem un vector director Wvs i un punt A de la recta.

Wvs (1, 3, 2) A(0, 2, 5) s WAP (1, 2, 3)

vs AP

i j k

1 3 2

1 2 3

13i 5 j k ( 13, 5, 1)WW W

W

WW W W

Wvs 1 9 4 14 Wvs WAP 169 25 1 195

La distància del punt P a la recta s és:

d(P , s)vs AP

vs

195

1513

W

W

W

013 Calcula la distància de l’origen de coordenades a les rectes següents:

a) rxyz

:1 320

b) sx y z

:2

11

14

2

a) Determinem un vector director Wvr i un punt A de la recta.

Wvr (3, 1, 0)

A(1, 2, 0) s WAO

( 1, 2, 0)

vr AO

i j k

3 1 0

1 2 0

7k (0, 0, 7)WW W

W

WW

Wvr 9 1 10 Wvr WAP 49 7

La distància de l’origen de coordenades O a la recta r és:

d(O , r )vr AO

vr

7

10

7 10

10

W

W

W

b) Calculem un vector director Wvs i un punt A de la recta.

Wvs ( 1, 1, 2) A(2, 1, 4) s WAO ( 2, 1, 4)

vs AO

i j k

1 1 2

2 1 4

2i k ( 2, 0, 1)WW W

W

WW W

Wvs 1 1 4 6 Wvs WAO 4 1 5

La distància de l’origen de coordenades O a la recta s és:

d(O , s)vs AO

vs

5

6

30

6

W

W

W

354

Productes vectorial i mixt

014 Calcula el producte mixt [ Wu, Wv, Ww ]; els vectors són Wu (0, 1, 0), Wv (1, 1, 1) i Ww (0, 0, 1).

v w

i j k

1 1 1

0 0 1

i j ( 1, 1, 0)

W W

W W

WW W

(Wu, Wv , Ww ) Wu (Wv Ww ) (0, 1, 0) ( 1, 1, 0) 1

015 Calcula el volum del paral·lelepípede definit pels vectors Wu ( 2, 0, 0), Wv (3, 2, 1) i Ww ( 2, 0, 4).

v w

i j k

3 2 1

2 0 4

8i 14 j 4k (8, 14, 4)

W W

W W

WW W W

[Wu, Wv , Ww ] Wu (Wv Ww ) ( 2, 0, 0) (8, 14, 4) 16

Volum [Wu, Wv , Ww ] 16

016 Calcula el producte mixt dels vectors següents:Wu (3, 8, 0) Wv (0, 1, 3) Ww ( 5, 4, 0)

[Wu, Wv , Ww ]

3 8 0

0 1 3

5 4 0

156

017 Comprova que aquest producte mixt és zero:

[ Wu, Wv , Wu Wv ]

Considerem: Wu (Wu1, Wu2, Wu3) Wv (Wv 1, Wv 2, Wv 3)

[Wu, Wv , Wu Wv ]

u1 u2 u3

v1 v2 v3

u1 v1 u2 v2 u3 v3

0

La tercera fila és combinació de la primera i la segona.

018 Troba el volum del paral·lelepípede determinat pels vectors:Wu ( 1, 1, 1) Wv (1, 1, 1) Ww (0, 0, 2)

Com que Wu i Wv són proporcionals, els tres vectors, Wu, Wv i Ww , no generen

un paral·lelepípede.

019 Obtingues el volum del tetraedre els vèrtexs del qual tenen les coordenades següents:

A(0, 0, 0) B(0, 0, 1) C(0, 2, 0) D (3, 0, 0)

WAB (0, 0, 1) WAC (0, 2, 0) WAD (3, 0, 0)

[WAB, WAC, WAD]

0 0 1

0 2 0

3 0 0

6

Volum

[ AB AC AD]

6

6

61

W W W

355

6SOLUCIONARI

020 Determina la distància entre les següents rectes que es creuen:

r yx z

: 12

sx y z

:1

21

34

2

Trobem un punt i un vector de cada recta.

vr

i j k

0 1 0

1 0 1

i k (1, 0, 1)

W W

W

WW W Pr( 2, 1, 0) r

Wvs ( 2, 3, 2) Ps ( 1, 1, 4) s

El vector determinat pels punts Pr i Ps és: WPrPs (1, 2, 4)

vr vs

i j k

1 0 1

2 3 2

3i 4 j 3k (3, 4, 3)

W W

W W

WW W W

[WPrPs, Wvr , Wvs] WPrPs (Wvr Wvs) (1, 2, 4) (3, 4, 3) 7

Per tant, la distància entre les rectes és:

d(r , s)[Pr Ps , vr , vs ]

vr vs

7

9 16 9

238

34

W W

W

W

W

021 Estudia la posició relativa d’aquestes rectes, i calcula la distància que hi ha entre elles:

r x yx y z

: 3 20 s x y z

x y z: 3

2

Trobem un punt i un vector de cada recta.

vr

i j k

3 1 0

1 1 1

i 3 j 4k (1, 3, 4)

W W

W

WWW W

Pr (0, 2, 2) r

vs

i j k

1 1 1

1 1 1

2 j 2k (0, 2, 2)

W W

W

WW W

Ps

5

2,

1

2, 0 s

Determinem la posició relativa de les dues rectes.

El vector determinat pels punts Pr i Ps és: WPrPs

5

2,

3

2, 2

vr vs

i j k

1 3 4

0 2 2

14i 2 j 2k ( 14, 2, 2) 0

[PrPs , vr , vs ] PrPs (vr vs )5

2,

3

2, 2 ( 14, 2, 2) 42 0

W

W W

WWW

W W

WW

WW

W W

És a dir, les rectes es creuen.

Així, doncs, la distància entre les rectes és:

d(r , s)[PrPs , vr , vs ]

vr vs

42

( 14)2 22 22

42

204

7 51

17

W

W

W

W

W

356

Productes vectorial i mixt

022 Calcula el lloc geomètric dels punts de l’espai que equidisten de dos punts fixos A(2, 0, 1) i B(4, 2, 0).

Anomenem P(x, y, z) els punts de l’espai que equidisten dels punts A i B.

d( A, P ) ( x 2)2 ( y 0)2 ( z 1)2

d(B , P ) ( x 4)2 ( y 2)2 ( z 0)2

Si igualem les dues distàncies, tenim que:

( x 2)2 ( y 0)2 ( z 1)2 ( x 4)2 ( y 2)2 ( z 0)2

4 4 2 15 0x y z

El lloc geomètric dels punts que equidisten de A i de B és el pla

d’equació:

: 4 x 4 y 2z 15 0

023 Troba el lloc geomètric dels punts que equidisten de tres punts fixos A(1, 0, 0), B (0, 1, 0) i C (0, 0, 1).

Anomenem P(x, y, z) els punts que equidisten dels punts A, B i C.

d( A, P ) ( x 1)2 x 1

d(B , P ) ( y 1)2 y 1

d(C , P ) ( z 1)2 z 1

Igualem les distàncies:

x 1 y 1 z 1

El lloc geomètric dels punts que equidisten dels punts A, B i C és la recta

d’equació:

r :x 1

1

y 1

1

z 1

1

024 Determina el lloc geomètric dels punts de l’espai que disten 1 unitat dels plans:

a) 1: x 0 b) 2: x y 2z 0

a) Anomenem P(x, y, z) els punts de l’espai que disten 1 unitat del pla 1.

d(P , 1)x

11 x 1

x 1

x 1

El lloc geomètric està format per dos plans:

1: x 1 2 : x 1

b) Anomenem P(x, y, z) els punts de l’espai que disten 1 unitat del pla 2.

d(P , 2 )x y 2z

1 1 41 x y 2z 6

x y 2z 6

x y 2z 6

El lloc geomètric està format per dos plans:

1: x y 2z 6 0 2 : x y 2z 6 0

357

6SOLUCIONARI

025 Determina el lloc geomètric dels punts que equidisten d’aquests plans:

1: x 0 2: x y 2 z 0

Anomenem P(x, y, z) els punts que equidisten dels plans 1 i 2.

d(P , 1)x

1x

d(P , 2 )x y 2z

6

Igualem les distàncies:

d(P , 1) d(P , 2 ) xx y 2z

6

xx y 2z

6

xx y 2z

6

El lloc geomètric està format pels plans:

1: ( 6 1)x y 2z 0 2 : ( 6 1)x y 2z 0

026 Troba l’equació general de l’esfera que té com a centre el punt C (7, 1, 0) i que passa pel punt de coordenades P ( 3, 4, 2).

Calculem el radi de l’esfera.

d(P , C ) 102 ( 5)2 22 129

L’equació de l’esfera és:

( x 7)2 ( y 1)2 ( z 0)2 129

Si desenvolupem obtenim l’equació general.

x 2 y 2 z 2 14 x 2y 79 0

027 Estudia si aquesta equació correspon a una esfera. En cas afirmatiu, calcula’n el centre i el radi.

x y z y2 2 2 2 2 0

Si aquesta equació correspongués a una esfera s’hauria de complir que:

2a 0

2b 2

2c 0

a2 b2 c2 r 2 2

a 0

b 1

c 0

1 r 2 2 r 2 1

Com que r 2 1 no té solució, aquesta equació no correspon a una esfera.

028 Discuteix la posició relativa de la recta s: ( x, y, z ) (4, 2, 1) (1, 2, 1) i l’esfera de centre C (2, 0, 3) i radi r 3.

Trobem la distància de la recta al centre de l’esfera:

Wvs (1, 2, 1) A(4, 2, 1) s WAC

( 2, 2, 2)

vs AC

i j k

1 2 1

2 2 2

6i 6k (6, 0, 6)W W W WW W W

358

Productes vectorial i mixt

vs AC 36 36 72W W

d(C , s)vs AC

vs

72

12 22 ( 1)212 3

W

W

W

La distància de la recta al centre és més gran que el radi; per tant, la recta

és exterior a l’esfera.

029 Troba la posició relativa del pla : 3x 2y 3z 1 i l’esfera de centre C (2, 0, 3) i radi r 3.

Trobem la distància del centre de l’esfera al pla.

d(C , )3 2 2 0 3 3 1

(3, 2, 3)

14

223

La distància del centre al pla és més petita que el radi; per tant, el pla és secant

a l’esfera.

030 Troba el pla tangent a l’esfera de centre C (2, 0, 3) i radi r 3 en el punt P (2, 0, 0).

El vector normal del pla és: WCP

(0, 0, 3)

L’equació del pla tangent a l’esfera en el punt P és: : 0 ( x 2) 0 ( y 0) ( 3) ( z 0) 0 : 3z 0

031 Calcula la recta normal a l’esfera de centre C(2, 0, 3) i radi r 3 en el punt P(2, 0, 0).

P(2, 0, 0)

CP (0, 0, 3)W s

x

y

z

:

2

0

3

032 Comprova, amb un exemple, la relació que hi ha entre:

Una recta perpendicular al pla tangent a una esfera en un punt i la recta normal a l’esfera en aquest mateix punt.

Una recta perpendicular al pla tangent a una esfera en un punt és la recta normal

a l’esfera en aquest punt.

Per exemple, el pla tangent a l’esfera de centre C(2, 0, 1) i radi r 2

en el punt P(0, 0, 1) és 2x 0.

La recta tangent a aquest pla a P(0, 0, 1) és:

s:

x 2

y 0

z 1

La recta normal a l’esfera a P(0, 0, 1) és:

s:

x 2

y 0

z 1

Les dues expressions coincideixen.

359

6SOLUCIONARI

033 Si Wu ( 1, 4, 2), Wv ( 3, 1, 6) i Ww (3, 3, 1), calcula:

a) Wu Wv b) Wu (Wv 2Ww ) c) 2Wu Wv Ww

a)

u v

i j k

1 4 2

3 1 6

22i 11k (22, 0, 11)W W W WW W W

b)

u ( v 2w )

i j k

1 4 2

9 5 8

42i 10 j 41k (42, 10, 41)W W W W W WW W W

c) 2u v w 2u

i j k

3 1 6

3 3 1

2( 1, 4, 2) ( 19, 15, 12) ( 21, 23, 8)W W W W

W W W

034 Si Wu (3, 2, 0) i Wv (3, 6, 2), troba Wu Wv i Wv Wu , i explica’n el resultat.

u v

i j k

3 2 0

3 6 2

4i 6 j 12kW W W W WW W W

v u

i j k

3 6 2

3 2 0

4i 6 j 12kW W W W WW W W

Wu Wv i Wv Wu tenen el mateix mòdul, la mateixa direcció i sentits oposats.

035 Troba dos vectors que tinguin com a mòdul 5 i que siguin perpendiculars als vectors Wu (2, 0, 1) i Wv (6, 3, 2).

Calculem un vector perpendicular a Wu i Wv .

u v

i j k

2 0 1

6 3 2

3i 10 j 6k ( 3, 10, 6)W W W W WW W W

Wu Wv 9 100 36 145

Els vectors perpendiculars a Wu i Wv de mòdul 5 són:

Ww1 5

145( 3, 10, 6)

3 145

29,

10 145

29,

6 145

29

Ww2 5

145( 3, 10, 6)

3 145

29,

10 145

29,

6 145

29

036 Si Wu (3, 2, 0) i Wv (3, 6, 2), calcula Wu (Wu Wv ). Extreu-ne una conclusió general.

u v

i j k

3 2 0

3 6 2

4i 6 j 12kW W W W WW W W

Wu (Wu Wv )

(3, 2, 0) ( 4, 6, 12) 0

Wu (Wu Wv ) sempre val 0 perquè Wu Wv és un vector perpendicular a Wu .

037 Justifica mitjançant determinants que Wu Wu 0.

El determinant d’una matriu amb dues files iguals sempre és nul.

360

Productes vectorial i mixt

038 Demostra, utilitzant les propietats dels determinants, que el producte vectorial distribueix la suma.

Wu (Wv Ww ) Wu Wv Wu Ww

Considerem els vectors Wu (u1, u2 , u3 ), Wv (v1, v2 , v3 ) i Ww (w1, w2 , w3 ).

i j k

u1 u2 u3

v1 w1 v2 w2 v3 w3

i j k

u1 u2 u3

v1 v2 v3

i j k

u1 u2 u3

w1 w2 w3

W W WW W WW W W

Wu (Wu Ww ) Wu Wv Wu Ww

039 Determina, fent servir el producte vectorial, l’angle que formen els vectors (4, 1, 3) i (3, 0, 2).

Wu

(4, 1, 3) Wv

(3, 0, 2)

u v

i j k

4 1 3

3 0 2

2i j 3k ( 2, 1, 3)

W

W WW

W W WW

Wu Wv Wu Wv sin

→ sinu v

u v

( 2, 1, 3)

26 13

14

26 130,2035

11° 44' 34"168° 15' 26"

W

W

W

W

Si agafem l’angle entre dos vectors com el més petit que formen les seves

direccions quan es tallen, la solució és 11° 44' 34".

040 Troba [Wu, Wv , Ww ] en els casos següents.

a) Wu (0, 4, 2) Wv ( 2, 7, 1) Ww (5, 2, 1)b) Wu (0, 4, 2) Wv (5, 2, 1) Ww ( 2, 7, 1)c) Wu (9, 4, 1) Wv ( 1, 5, 11) Ww (12, 6, 0)

a) Wu , Wv , Ww

0 4 2

2 7 1

5 2 1

34 c) Wu , Wv , Ww

9 4 11

1 5 11

12 6 10

0

b) Wu , Wv , Ww

0 4 2

5 2 1

2 7 1

34

041 Demostra, utilitzant propietats dels determinants, que per a k, Wu, Wv i Ww es verifica que:

[kWu, Wv, Ww ] [Wu, kWv, Ww ] k [Wu, Wv, Ww ]

Considerem els vectors Wu

(u1, u2 , u3 ), Wv

(v1, v2 , v3 ) i Ww

(w1, w2 , w3 ).

Si utilitzem les propietats dels determinants:

ku1 ku2 ku3

v1 v2 v3

w1 w2 w3

u1 u2 u3

kv1 kv2 kv3

w1 w2 w3

k

u1 u2 u3

v1 v2 v3

w1 w2 w3

Per tant: [kWu , Wv , Ww ] [Wu , kWv , Ww ] k[Wu , Wv , Ww ]

361

6SOLUCIONARI

042 Donat el vector Wa (3, 5, 2), tria un altre vector Wb i comprova que [Wa, Wb, 2Wa Wb ] 0. Explica per què passa això.

Per a qualsevol vector Wb que triem, 2Wa Wb és combinació lineal

dels vectors Wa i Wb . I, com que el producte mixt de vectors linealment dependents

és zero: [Wa , Wb , 2Wa Wb ] 0

043 Explica per què per a qualssevol Wa, Wb i Wc :

[Wa 2Wb Wc, 3Wa 5Wb, 2Wa 7Wb Wc ] 0

El producte mixt dels tres vectors definits com a combinació lineal

de Wa , Wb i Wc és:

1 2 1

3 5 0

2 7 1

0

Això succeeix perquè els tres vectors són coplanaris i, per tant, el volum

del paral·lelepípede que formen és zero.

044 Comprova amb Wu (3, 4, 5) i Wv ( 2, 3, 1) si es verifica la igualtat: Wu Wv Wu Wv (Wu Wv )2

u v

i j k

3 4 5

2 3 1

19i 13 j k ( 19, 13, 1)

W

W WW

W W WW

Wu Wv 3 59

Wu 9 16 25 50 5 2

Wv 4 9 1 14

(Wu Wv )2 ( 6 12 5)2 169

Wu Wv (Wu Wv )2 5 2 14 169 10 7 169

Com que 3 59 10 7 169 → No es verifica la igualtat.

045 En general, el producte vectorial no té la propietat associativa: (Wu Wv ) Ww Wu (Wv Ww )

Comprova-ho per a Wu Wj , Wv Wk i Ww Wk.

( u v ) w ( j k ) k i k j

u ( v w ) j ( k k ) j 0 0

W

W

WW

W W

W W

W WW WW WW

W W W No es compleix la propietat associativa.

046 Demostra que (Wu Wv ) (Wu Wv ) 2(Wu Wv ), siguin quins siguin els vectors Wu i Wv.

( u v ) ( u v ) u u u v v u v vW W WW W WW WW W W W

Sabem que:

u u v v 0

v u u v

W

W

W

W

WW

W

W

W ( u v ) ( u v ) 2( u v )W W WW W W

362

Productes vectorial i mixt

047 Troba un vector Wa que tingui mòdul 3, i tal que si Wb (3, 3, 0) es verifiqui que Wa Wb (6, 6, 3).

Considerem el vector Wa

(a1, a2 , a3 ) .

a b

i j k

a1 a2 a3

3 3 0

3a3i 3a3 j 3(a 1 a2 )k (6, 6, 3)

W WW

WW W W W

Igualem coordenades:

3a3 6

3(a1 a2 ) 3

a3 2

a2 1 a1

Wa (a1, 1 a1, 2)

Wa 2a12 2a1 5

Com que Wa

3 2a12 2a1 5 9

a1 2

a1 1

Per tant, hi ha dos vectors que compleixen la condició inicial: Wa1 ( 2, 1, 2)

i Wa2 (1, 2, 2)

048 Calcula el valor de a perquè el producte vectorial dels vectors (a, a, 2) i (2, a, 1) sigui proporcional al vector (1, 1, 0).

(Activitat de Selectivitat)

Wu (a, a, 2) Wv (2, a, 1)

u w

i j k

a a 2

2 a 1

3ai (4 a) j (a2 2a)k ( 3a, 4 a, a2 2a)

W W

W

W

W W W W

Com que ha de ser proporcional al vector (1, 1, 0), aleshores:

3a

1

4 a

1

a2 2a

0

3a 4 a

a2 2a 0a 2

049 Si tenim Wu (3, 2, 5) i Wv ( 1, 1, 1), quins vectors Ww compleixen que WWu Ww Wv ? Escriu-ne tres exemples.

Considerem el vector Ww (w1, w2 , w3 ).

u w

i j k

3 2 5

w1 w2 w3

( 5w2 2w3 )i (5w1 3w3 ) j (2w1 3w2 )k ( 1, 1, 2)

W W

W

W

W W W W

Igualem coordenades:

5w2 2w3 1

5w1 3w3 1

2w1 3w2 1

w1

1 3

5

w2

1 2

5w3

Obtenim tres exemples donant valors a :

Per a 2 → Ww1 ( 1, 1, 2)

Per a 3 ⎯→ Ww2 (2, 1, 3)

Per a 8 ⎯→ Ww3 (5, 3, 8)

363

6SOLUCIONARI

050 Troba un vector Wu, de manera que per als vectors Wa (3, 5, 1), Wb (1, 0, 4) i Wc ( 3, 1, 2) es verifiqui que 2Wa 3Wu Wb Wc.

b c

i j k

1 0 4

3 1 2

4i 10 j k ( 4, 10, 1)

W WW

W

W W W W

2a 3u b c 3u 2a b c u1

3(2a b c )W WW W W W WW WW W W

→ Wu

1

3[2 (3, 5, 1) ( 4, 10, 1)]

10

3, 0, 1

051 Com han de ser dos vectors perquè compleixin que Wu Wv Wu Wv ?

Wu Wv Wu Wv sin Wu Wv → sin 1

Els vectors han de ser perpendiculars.

052 Troba el vector director de la recta: r x y zx y z

: 12 6 18 13 08 4 12 5 0

vr

i j k

12 6 18

8 4 12

(0, 0, 0)

W W W

W Els plans no defineixen una recta.

Com que els vectors normals són proporcionals, els plans són paral·lels

o coincidents, és a dir, no es tallen en una única recta.

053 Determina l’equació d’una recta perpendicular a r i s que passi pel punt P(3, 2, 0), si:

r xy z

:2

22

3 s

xyz

:2

35 2

Wvr (1, 2, 3) Wvs

(0, 1, 2)

vr vs

i j k

1 2 3

0 1 2

i 2 j k (1, 2, 1)

W W W

W WW W W

La recta que busquem és t:

x 3

y 2 2

z

054 Decideix si les rectes són paral·leles:

rxyz

:1 3

22

s x y zx

: 2 3 02yy z2 5 0

Wvr (3, 2, 1)

vs

i j k

2 1 1

1 2 2

4i 3 j 5k (4, 3, 5)

W W W

W W W W

3

4

2

3

1

5 Els vectors no són proporcionals;

per tant, les rectes no són paral·leles.

364

Productes vectorial i mixt

055 Determina la distància del punt P(1, 2, 3) a la recta:

rx y z

:1

21

1 2

Agafem un vector director Wvr i un punt A de la recta.

Wvr (2, 1, 2)

A(1, 1, 0) r WAP

(0, 3, 3)

vr AP

i j k

2 1 2

0 3 3

3i 6 j 6k (3, 6, 6)

W W W

W W W WW

Wvr 4 1 4 3 Wvr WAP 9 36 36 9

La distància del punt P a la recta r és:

d(P , r )vr AP

vr

9

33

W

W

W

056 Troba la distància del punt P( 2, 2, 9) a la recta:

r x yx y z

: 3 16 2 1

Troba també el punt de r que es troba a aquesta distància de P. Comprova que les dues distàncies coincideixin.

Calculem la distància del punt P a la recta.

Agafem un vector director Wvr i un punt A de la recta.

vr

i j k

1 3 0

1 6 2

6i 2 j 3k (6, 2, 3)

W W W

W W W W

A(1, 0, 0) r WAP

( 3, 2, 9)

vr AP

i j k

6 2 3

3 2 9

12i 45 j 6k (12, 45, 6)

W W W

W W W WW

Wvr 36 4 9 7 Wvr WAP 144 2.025 36 2.205 21 5

La distància del punt P a la recta r és:

d(P , r )vr AP

vr

21 5

73 5

W

W

W

Determinem el punt que es troba a aquesta distància P.

Trobem el pla perpendicular a la recta r que passa per P.

6x 2y 3z D 0 6 ( 2) 2 2 3 ( 9) D 0 D 43

: 6x 2y 3z 43 0

Calculem el punt Q intersecció de la recta r amb el pla .

x 3y 1

x 6y 2z 1

6x 2y 3z 43 0

x 5

y 2

z 3

Q( 5, 2, 3)

d(P, Q) WPQ

( 3, 0, 6) 45 3 5 d(P , r )

365

6SOLUCIONARI

057 Troba un punt a la recta rx ty tz t

:2 4

1 52

que es troba a 3 unitats de distància

del punt P(8, 11, 3).

Comprova que aquest és el punt de la recta més proper a P.

Un punt de la recta r és de la forma R(2 4t , 1 5t , 2t ).

d(P , R) PR (4t 6, 5t 12, 2t 3) 45t 2 180t 189W

d(P , R) 3 45t 2 180t 189 3 t 2 → El punt és R(10, 9, 4).

Calculem la distància de P a la recta.

PR vr

i j k

2 2 1

4 5 2

9i 18k ( 9, 0, 18)WW W W

W W W

Wvr 16 25 4 3 5

WPR Wvr 81 324 9 5

d(P , r )PR vr

vr

9 5

3 53

W W

W

Com que la distància del punt a la recta també és 3, el punt R és el més

proper a P.

058 Calcula el punt del pla :xyz

1 31 41 2 2

que és més proper

al punt P (17, 4, 9). A quina distància es troba?

Hem de trobar la projecció del punt P sobre el pla.

Calculem el vector normal Wn del pla.

n

i j k

3 4 2

0 1 2

6i 6 j 3k (6, 6, 3) n (2, 2, 1)

W W W

W W W W W

La recta perpendicular al pla que passa per P és:

r :

x 17 2t

y 4 2t

z 9 t

Calculem la intersecció d’aquesta recta amb el pla.

17 2t 1 3

4 2t 1 4

9 t 1 2 2

2

1

t 6

El punt d’intersecció és P'(5, 8, 3).

La distància del punt P al pla coincideix amb la distància entre P i P'.

d(P , P' ) PP' ( 12, 12, 6) 18W

366

Productes vectorial i mixt

059 Troba la distància entre les rectes:

r x y z: 5 2 2

sxyz

:1

44

Determinem un punt i un vector de cada recta.

Wvr (1, 1, 1) Pr(5, 2, 2) Wvs (0, 1, 1) Ps( 1, 4, 4)

Calculem un vector determinat per un punt de cada recta.

WPrPs ( 6, 2, 2)

Calculem el producte mixt dels tres vectors.

vr vs

i j k

1 1 1

0 1 1

2i j k (2, 1, 1)

W W W

W W W W W Wvr Wvs 6

[ WPrPs, Wvr, Wvs] WPrPs (Wvr Wvs) ( 6, 2, 2) (2, 1, 1) 12

Així, doncs, la distància entre les rectes és:

d(r , s)[PrPs , vr , vs ]

vr vs

12

62 6

W

W

W

W

W

060 Comprova que la distància entre les rectes següents és 3, i troba un punt de cada una de les rectes que es trobi a aquesta distància.

rxyz

sx y z

: :4 2

16 4

13

41

144

Calculem la distància entre les dues rectes.

Determinem un punt i un vector de cada recta.

Wvr (2, 0, 4) Pr(4, 1, 6) Wvs (3, 1, 4) Ps(1, 4, 1)

Calculem un vector determinat per un punt de cada recta.

WPrPs ( 3, 5, 5)

Calculem el producte mixt dels tres vectors.

vr vs

i j k

2 0 4

3 1 4

4i 4 j 2k (4, 4, 2)

W W W

W W W W W

Wvr Wvs 16 16 4 6

[ WPrPs, Wvr, Wvs] WPrPs (Wvr Wvs) ( 3, 5, 5) (4, 4, 2) 18

La distància entre les rectes és:

d(r , s)[PrPs , vr , vs ]

vr vs

18

63

W

W

W

W

W

Calculem els punts que es troben a aquesta distància.

Considerem R i S dos punts genèrics de les rectes r i s, respectivament.

R(4 2 , 1, 6 4 ) r

S(1 3 , 4 , 1 4 ) s

WRS ( 2 3 3, 5, 4 4 5)

367

6SOLUCIONARI

El vector WRS ha de ser perpendicular als vectors directors de les rectes.

RS vr 20 22 26 0

RS vs 22 26 34 0

2

3

W

W

W

W

Els punts que busquem són: R(8, 1, 14) i S(10, 1, 13).

La distància entre aquests dos punts és: d(R, S) RS (2, 2, 1) 3W

061 Calcula la distància entre les rectes r i s:

r passa pel punt P ( 1, 2, 4) i és paral·lela a la recta x y x3

32

16

5.

sx y z

x y z:

2 42 1

Les equacions paramètriques de les rectes r i s són:

r :

x 1 3

y 2

z 4 5

s :

x 5 3

y

z 6 5

Determinem un punt i un vector de cada recta:

Wvr (3, 1, 5) Pr( 1, 2, 4) Wvs ( 3, 1, 5) Ps(5, 0, 6)

Com que Wvr i Wvs són proporcionals, les rectes són paral·leles o coincidents.

La distància entre les rectes és la distància de Pr ( 1, 2, 4)

a la recta s.

WPrPs (6, 2, 10)

Pr Ps vr

i j k

6 2 10

3 1 5

0W

W W WW

d(r , s) d(Pr , s)Pr Ps vr

vr

0W

W

W

Les rectes són coincidents.

062 Expressa en forma paramètrica les equacions de la recta que resulta de tallar els plans : 2 3 5 10x y z i ' : x y z2 4.

a) Resolent primer el sistema que formen.b) Fes-ho també calculant un vector director.

a) 2x 3y 5z 10

x y 2z 4

x 2

y 2

z

b) vr

i j k

2 3 5

1 1 2

i j k (1, 1, 1)

W W W

W W W W

Calculem un punt de la recta.

2x 3y 5z 10

x y 2z 4

Si z 2

2x 3y 0

x y 0

x 0

y 0

Un punt de la recta és P(0, 0, 2)

L’equació paramètrica de la recta és r :

x

y

z 2

368

Productes vectorial i mixt

063 Troba l’equació del pla que conté la recta r i és paral·lel a la recta s.

rx y z

:1

22

23

1 s x y z

x y z: 2 8 0

5 2 2 6 0

Busquem el pla que passa per Pr (1, 2, 3) i que té com a vectors directors

Wvr (2, 2, 1) i Wvs.

vs

i j k

2 1 1

5 2 2

4i j 9k ( 4, 1, 9)

W W W

W W W W

:

x 1 y 2 z 3

2 2 1

4 1 9

17x 22y 10z 57 0

064 Determina l’equació d’una recta que passa pel punt ( 1, 4, 2) i que és paral·lela a la recta intersecció dels plans:

: 2 3 2 0x y z ' : x y z2 2 5 0

Busquem la recta que passa per P( 1, 4, 2) i que té com a vector director el vector

director de la recta donada.

vs

i j k

2 3 1

1 2 2

4i 5 j 7k (4, 5, 7)

W W W

W W W W

L’equació de la recta que busquem és:

s :

x 1

4

y 4

5

z 2

7

065 Decideix si les dues rectes es tallen i, si és així, determina’n el punt de tall. Troba també l’equació d’una recta que passi pel punt A(3, 1, 0) i que sigui perpendicular a les dues rectes:

rx y z

:1

61

42

sxyz

:5 24

2

Determinem la posició relativa de les rectes.

Wur ( 1, 1, 2) Pr(0, 6, 4) Wvs (2, 1, 0) Qs(5, 4, 2)

WPrQs (5, 2, 2)

rang

5 2 2

1 1 2

2 1 0

2

Com que els vectors directors no són proporcionals, les rectes es tallen.

Determinem el punt de tall de les rectes.

x

1

y 6

1

z 4

2

x 5 2

y 4

z 2

x 1

y 7

z 2

→ El punt de tall és C( 1, 7, 2).

369

6SOLUCIONARI

Determinem la recta que passa per A(3, 1, 0) i que és perpendicular a les rectes r i s.

ur vs

i j k

1 1 2

2 1 0

2i 4 j k (2, 4, 1)

W W W

W W W W W

La recta és t:

x 3

2

y 1

4

z

1

066 Una recta passa pel punt (1, 1, 0) i és paral·lela als plans x y 1, x z 1. Troba’n les equacions.

(Activitat de Selectivitat)

Busquem una recta que passa per P(1, 1, 0) i que té com a vector director

el producte vectorial dels vectors normals dels plans.

i j k

1 1 0

1 0 1

i j k (1, 1, 1)

W W WW W W

La recta és r :

x 1

1

y 1

1

z

1

067 Troba una recta paral·lela al pla : 2x 5y z 2 0 que sigui a 2 30de distància.

Agafem un punt del pla, per exemple: P(2, 1, 3)

Si al punt P sumem un vector de mòdul 2 30 que sigui perpendicular al pla,

obtindrem un punt que es troba a aquesta distància del pla.

El vector perpendicular al pla amb mòdul 2 30 ha de ser un vector proporcional

al vector normal del pla.

Com que Wn (2, 5, 1), el vector que busquem ha de ser de la forma:

Wn (2, 5, 1) (2 , 5 , )

I ha de complir que:

(2 , 5 , ) 30 2 2 30 2

És a dir, un vector perpendicular al pla amb mòdul 2 30 és Wu

(4, 10, 2).

Q Wu

(2, 1, 3) (4, 10, 2) (6, 9, 5)

El punt Q(6, 9, 5) és a 2 30 de distància del pla.

La recta que busquem passa pel punt Q i té com a vector director un vector

perpendicular al vector normal.

Si Wvr (a, b, c), s’ha de complir que:

Wvr Wn 0 (a, b, c ) (2, 5, 1) 2a 5b c 0

Si a 1 i b 0 c 2

El vector Wvr (1, 0, 2) és perpendicular a Wn .

La recta que busquem és:

r:

x 6

y 9

z 5 2

370

Productes vectorial i mixt

068 Calcula la distància de P ( 3, 4, 0) a les rectes:

a) rx

yz

:4

22

23

c) t x y zx y z

: 2 3 1 02 4 12 0

b) sxyz

:3 219 5

3

a) Wur (2, 1, 3) Qr( 4, 2, 2) WPQr ( 1, 2, 2)

PQr ur

i j k

1 2 2

2 1 3

4i j 3k (4, 1, 3)W W W WW W W

W

d(P, r )PQr ur

ur

26

14

13

7

91

7

W

W

W

b) Wus (2, 5, 1) Qs(3, 19, 3) WPQs (6, 15, 3)

PQs us

i j k

6 15 3

2 5 1

0W

W W WW → El punt P pertany a la recta s.

c)

ut

i j k

1 2 3

2 4 1

14i 5 j 8k (14, 5, 8)W W W WW W W

Qt(1, 3, 2) WPQt (4, 1, 2)

PQt ut

i j k

4 1 2

14 5 8

18i 4 j 34k (18, 4, 34)W W W WW W W

W

d(P , t )PQt ut

ut

1.496

285

2 106.590

285

W

W

W

069 Troba la distància entre la recta r t t t: ( , , )13 7 4 2 i el pla : 4x 2y z 7 0. Troba dos punts del pla que es trobin a aquesta distància

de r. Obtingues l’equació de la projecció ortogonal de r sobre .

vr (1, 1, 2)

n (4, 2, 1)

W

W Són perpendiculars.

La recta és paral·lela al pla o hi està continguda.

Calculem la distància d’un punt de la recta, P(13, 7, 4) r , al pla :

d(r , ) d(P , )4 13 2 7 4 7

42 22 ( 1)2

63

213 21

Per determinar dos punts del pla que es trobin a 3 21 , de distància, calculem

els punts P' i Q' del pla que siguin projecció ortogonal de dos punts P i Q

de la recta r.

371

6SOLUCIONARI

Projecció ortogonal P' del punt P(13, 7, 4) r sobre el pla.

La recta perpendicular al pla que passa per P és r1:

x 13 4p

y 7 2p

z 4 pTrobem el punt P' intersecció entre r1 i .

x 13 4p

y 7 2p

z 4 p

4 x 2y z 7 0

x 1

y 1

z 1

p 3

P'(1, 1, 1)

Projecció ortogonal Q' del punt Q(14, 6, 2) r sobre el pla.

La recta perpendicular al pla que passa per Q és

r2:

x 14 4p

y 6 2p

z 2 pTrobem el punt Q' intersecció entre r2 i .

x 14 4p

y 6 2p

z 2 p

4 x 2y z 7 0

x 2

y 0

z 1

p 3

Q'(2, 0, 1)

La projecció ortogonal de r sobre passa pels punts P' i Q'.

r': x 1

y 1

z 1 2

070 Calcula la distància entre les dues rectes.

r x yx y z

: 3 2 1 03 2 6 9 0

s

x ty tz t

:1 4

1 62 4

Justifica el que passa.

Determinem els vectors directors de les rectes.

vr

i j k

3 2 0

3 2 6

12i 18 j 12k ( 12, 18, 12)W W W W

Wvs (4, 6, 4)

vr vs

i j k

13

21

4 6 4

0

W

W W

W W

→ Les rectes són paral·leles o coincidents.

L’equació implícita de la recta s és: 3x 2y z 5 0

x 2y z 1 0

rang

3 2 0

3 2 6

3 2 0

1 0 1

rang

3 2 0 1

3 2 6 9

3 2 0 5

1 0 1 1

2

Les rectes són coincidents; per tant, d(r , s) 0 .

372

Productes vectorial i mixt

071 Troba la distància entre les rectes r : ( , , )3 2 5 i s, que passa pels punts A( 1, 2, 4) i B(3, 10, 0).

Calculem un punt i un vector director de cada recta.

Wur ( 1, 2, 1) P(3, 0, 5) A( 1, 2, 4) WAB (4, 8, 4)

Com que els vectors són proporcionals, les rectes són paral·leles o coincidents.

WAP (4, 2, 1) AP u r

i j k

4 2 1

1 2 1

5 j 10k (0, 5, 10)W W WW W W

W

Wur 1 4 1 6 Wur WAP 25 100 5 5

Com que les rectes són paral·leles:

d(s, r ) d( A, r )ur AP

ur

5 5

6

5 30

6

W

W

W

072 Calcula la distància de la recta rxyz

:2 3

1 24

a l’eix Z.

Wur (3, 2, 1) Pr(2, 1, 4) O(0, 0, 0) Wvz (0, 0, 1)

[ WPr O, Wur, Wvz]

2 1 4

3 2 1

0 0 1

7

ur v z

i j k

3 2 1

0 0 1

2i 3 j (2, 3, 0)WW W WW W W

Wur Wvz 13

d(r , s)[PrO , ur , v z ]

ur v z

7

13

7 13

13

W W

W W

W

073 Determina l’equació d’un pla paral·lel al pla OXY que es trobi a 5 unitats de distància.

L’equació del pla OXY és z 0. Hi ha dos plans paral·lels que són a 5 unitats

de distància d’aquest pla; són els plans z 5 i z 5.

074 Troba l’equació d’un pla paral·lel a l’eix Y que es trobi a 4 unitats de distància.

Hi ha infinits plans en aquesta situació. Per exemple: z 4 i z 4.

075 Determina les equacions d’una recta paral·lela a l’eix Y que és a 6 unitats de distància de O (0, 0, 0).

L’eix Y té per equació:

x 0

z 0. Una recta que és a 6 unitats de l’origen

és la recta que passa per (6, 0, 0). La recta paral·lela a l’eix Y que passa

pel punt (6, 0, 0) és:

x 6

z 0

373

6SOLUCIONARI

076 Troba el valor del paràmetre a perquè les rectes:

r x y a z: 3 sxyz

:3

63

es trobin a una distància de 2 unitats.

Determinem un punt i un vector director de cada recta.

Wur ( 1, 1, 1) P(0, a, 3) Wvs (1, 1, 1) Q( 3, 6, 3)

WPQ

( 3, 6 a, 6)

[Wur, Wvs, WPQ]

1 1 1

1 1 1

3 6 a 6

6 2a

ur vs

i j k

1 1 1

1 1 1

2i 2 j (2, 2, 0)WW W WW W W

Wur Wvs 8

d(r , s)[ ur , vs , PQ ]

ur vs

6 2a

8

3 a

22 3 a 2

a 1

a 5

W W

W W

W

077 Considera el triangle que té com a vèrtexs els punts A(1, 1, 2), B(1, 0, 1) i C (1, 3, 2)a) Raona si el triangle és rectangle.b) Calcula la recta r que passa per B i és perpendicular al costat AC.c) Calcula la recta s que passa pels punts A i C.d) Si D és el punt de tall de les rectes r i s, calcula el mòdul del vector BDW.e) Calcula la longitud del costat AC.f ) Calcula el producte vectorial dels vectors ACW i ABW i comprova que el mòdul

és igual a h b, en què h és el mòdul del vector BDW i b és la longitud del costat AC (calculats en apartats anterior).

(Activitat de Selectivitat)

a) Analitzem els productes escalars WAC WAB, WAC WBC i WAC WBC.

WAC (0, 4, 0) WAB (0, 1, 3) WBC (0, 3, 3)

WAC WAB 4 0

WAC WBC 12 0

WAB WBC 6 0

El triangle no té costats perpendiculars; no és un triangle rectangle.

b) Determinem el pla perpendicular al costat AC i que passi pel punt B.

WAC

(0, 4, 0) →

: 4 y D 0

B(1, 0, 1) → D 0

Així, doncs, el pla és : y 0.

La recta que passa per A i C és:

s:

x 1

y 1 4

z 2

374

Productes vectorial i mixt

Calculem el punt de tall del pla i la recta.

x 1

y 1 4

z 2

y 0

→ El punt de tall és D(1, 0, 2).

La recta que busquem és la recta que passa per B i D.

WBD

(0, 0, 3)

r:

x 1

y 0

z 1 3

c) WAC

(0, 4, 0) →

s:

x 1

y 1 4

z 2

d) WBD

(0, 0, 3) 3

e) WAC

(0, 4,0) 4

f )

AC AB

i j k

0 4 0

0 1 3

12i (12, 0, 0)WW W W

W W WAC WAB 12

WBD WAC 3 4 12

Per tant: WAC WAB WBD WAC

078 Donades les rectes:r x y z

x y: 0

2 7 s x

y: 2

5

Troba un punt en cada una d’elles de tal manera que el vector que els uneixi sigui perpendicular a totes dues.

(Activitat de Selectivitat)

Pr(7, 0, 7)

vr

i j k

1 1 1

1 2 0

2i j k (2, 1, 1)W W W WW W W

Ps(2, 5, 0)

vs

i j k

1 0 0

0 1 0

k (0, 0, 1)W WW W W

Considerem A(7 2 , , 7 ) un punt genèric de la recta r, i

B(2, 5, )

un punt genèric de la recta s. Hem de trobar i de manera

que el vector WAB sigui perpendicular a les dues rectes.

WAB ( 2 5, 5, 7)

AB ( 2, 1, 1) 6 12 0

AB (0, 0, 1) 7 0

1

6

W

W

Els punts són: A(5, 1, 6) i

B(2, 5, 6)

375

6SOLUCIONARI

079 La recta x yy z

11

talla en P i Q respectivament els plans y 0 i x 0.

a) Determina els punts (si n’hi ha) de l’eix Z que equidistin de P i de Q. Naturalment, aquests possibles punts depenen del valor de .

b) Determina perquè, a més, els punts de l’eix Z formin amb P i Q un triangle equilàter.

(Activitat de Selectivitat)

Trobem els punts P i Q.

x y 1

y z 1

y 0

P(1, 0, 1)

x y 1

y z 1

x 0

Q(0, 1, 1 )

a) Un punt genèric de l’eix Z és de la forma A(0, 0, ). Busquem els punts A

que compleixen: d(A, P) d(A, Q)

WAP

(1, 0, 1 ) 1 (1 )2

WAQ

(0, 1, 1 ) 1 (1 )2

d( A, P ) d( A, Q ) 1 (1 )2 1 (1 )2

0

2

2

Si 0 pot agafar qualsevol valor

Tots els punts de l’eix Z equidisten de P i Q.

Si

02

2

L’únic punt de l’eix Z que equidista de P i Q és

A 0, 0,2

2.

b) Els punts A, P i Q formaran un triangle equilàter si existeix un per al qual:

d( A, P ) d( A, Q ) d(P , Q )

Si

0 A 0, 0,2

2

AP 1, 0,2

12

4

AQ 1, 0,2

12

4

PQ ( 1, 1, ) 1 1 2

12

41 1 2 3 2 4

W

W

W

A, P i Q no formen un triangle equilàter.

Si

0 P(1, 0, 1), Q(0, 1, 1) i A(0, 0, )

AP (1, 0, 1 ) 1 (1 )2

AQ (0, 1, 1 ) 1 (1 )2

PQ ( 1, 1, 0) 2

1 (1 )2 20

2

W

W

W

Hi ha dos punts de l’eix Z que formen un triangle equilàter amb P i Q:

A(0, 0, 0) i A'(0, 0, 2).

No té solucions reals.

376

Productes vectorial i mixt

080 a) Demostra que les rectes:

l1: x ty tz t2

l2: x y zx y

02 1 0

es creuen en l’espai.b) Troba la distància entre aquestes rectes.

(Activitat de Selectivitat)

a) Expressem la recta l2 en forma paramètrica.

l2 :

x 1 p

y 1 2p

z p

Determinem un punt i un vector director de cada recta.

Wu1 (1, 1, 1) P(0, 0, 2) Wv2

( 1, 2, 1)

Q( 1, 1, 0) WPQ

( 1, 1, 2)

[Wu1, Wv2, WPQ]

1 1 1

1 2 1

1 1 2

1

u1 v2

i j k

1 1 1

1 2 1

3i 2 j k ( 3, 2, 1)WW W W WW W W

Com que [Wu1, Wv2, WPQ] 0 i Wu1 Wv2 0 → Les rectes es creuen.

b) Wu1 Wv2 9 4 1 14

d(r , s)[u1, v2 , PQ ]

u1 v2

1

14

14

14

WPQW

W

W

W

081 a) Calcula l’equació de la recta que passa per l’origen i que és perpendicular al pla : x y z 3. Troba el punt de tall de la recta amb el pla .

b) Troba el punt de la recta rxyz

: 31 2

la distància del qual al punt P (1, 0, 2) sigui 5.

(Activitat de Selectivitat)

a) Busquem una recta que passa per O(0, 0, 0) i que té com a vector director

el vector normal del pla Wn (1, 1, 1).

s:

x

y

z

Trobem el punt de tall de la recta amb el pla .

x

y

z

x y z 3

1 Q(1, 1, 1)

377

6SOLUCIONARI

b) Un punt genèric de la recta r és A( , 3 , 1 2 ).

WPA

( 1, 3 , 1 2 ) 5 ( 1)2 (3 )2 ( 1 2 )2 5

1

El punt que compleix la condició és A(1, 2, 3).

082 Determina les coordenades d’un punt que disti 2 unitats de la recta:x y z1

1 11

1(Activitat de Selectivitat)

A(1, 0,1) r

Wn (0, 1, 1) és un vector perpendicular al vector director Wv (1, 1, 1) de la recta.

Determinem un vector perpendicular a la recta que tingui mòdul 2.

Com que Wn 2 →

2 n 2W

L’extrem P del vector WOP WOA 2 Wn és un punt que es troba a 2 unitats

de distància de la recta.

P (1, 0, 1) 2 (0, 1, 1) 1, 2 , 1 2

Comprovem que és cert:

PA u 2

i j k

0 2 2

1 1 1

2 2 i 2 j 2 k 2 2 , 2 , 2W

W

W

W

W

W

WW

WPA Wu 2 3

d(P , r )PA u

u

2 3

(1, 1, 1)2

W

W

W

083 Troba l’equació general del pla que talla els eixos coordenats als punts (2, 0, 0), (0, 1, 0) i (0, 0, 2). Determina els punts de la recta x y z que estan a distància 5 d’aquest pla.

Busquem un pla que passa per A(2, 0, 0) i que té per vectors directors WAB

( 2, 1, 0) i WAC ( 2, 0, 2).

:

x 2 y z

2 1 0

2 0 2

2x 4 y 2z 4 0 : x 2y z 2 0

Determinem els punts de la recta que es trobin a 5 unitats del pla.

Un punt genèric de la recta és P(t , t , t ).

d(P , )t 2t t 2

1 4 15

4t 2 5 6

4t 2 5 6

t5 6 2

4

t5 6 2

4

Els punts són

P1

5 6 2

4,

5 6 2

4,

5 6 2

4

i P2

5 6 2

4,

5 6 2

4,

5 6 2

4.

378

Productes vectorial i mixt

084 Donat el pla 1 3 6: x y z , calcula perquè la recta r que passa pel punt P (1, 1, 2) i que és perpendicular a aquest pla ( 1) sigui paral·lela al pla 2 3: x y . Calcula la distància de la recta r a l’origen.

(Activitat de Selectivitat)

Busquem la recta que passa per P(1, 1, 2) i que té com a vector director el vector

normal al pla 1, Wn

(3, , 1).

r:

x 1 3

y 1

z 2

Si r és paral·lela a 2 , el vector director de r és perpendicular al vector normal de 2 .

(3, , 1) (1, 1, 0) 0 3 0 3

Calculem la distància de r a l’origen de coordenades.

OP u

i j k

1 1 2

3 3 1

5i 5 j ( 5, 5, 0)WW

WW

WW

W WOP Wu 50

d(O , r )OP u

u

50

(3, 3, 1)

50

19

5 38

19

W

W

W

085 Donades les rectes:

r x ayay z

: 21

s x zy z

: 13

es demana:a) Discutir la posició relativa de les dues rectes r, s segons els valors del paràmetre a.b) Si a 1, calcular la distància mínima entre les dues rectes r, s.

(Activitat de Selectivitat)

a) Estudiem el rang de la matriu de coeficients i de la matriu ampliada del sistema

que formen els quatre plans.

x ay 2

ay z 1

x z 1

y z 3

A

1 a 0

0 a 1

1 0 1

0 1 1

A*

1 a 0 2

0 a 1 1

1 0 1 1

0 1 1 3

1 a 0 2

0 a 1 1

1 0 1 1

0 1 1 3

4a

0 a 1

1 0 1

0 1 1

a 1

Si a 0 → rang (A*) 4 i rang (A) 3 → Les rectes es creuen.

Si a 0 → rang (A) rang (A*) 3 → Les rectes es tallen en un punt.

b) Escrivim les rectes en forma paramètrica.

r :

x 3

y 1

z

s :

x 1

y 3

z

Calculem un punt i un vector director de cada recta.

379

6SOLUCIONARI

Wur ( 1, 1, 1) P(3, 1, 0) Wvs (1, 1, 1) Q(1, 3, 0) WPQ ( 2, 2, 0)

[Wur, Wvs, WPQ]

1 1 1

1 1 1

2 2 0

4

ur vs

i j k

1 1 1

1 1 1

2 j 2k (0, 2, 2)WW

WW

WW

W

Wur Wvs 8 2 2

d(r , s)[ ur , vs , PQ ]

ur vs

4

2 22

WW W

W W

086 Les trajectòries de dos avions vénen donades per les rectes:

rxyz

rxyz

1 2

111 2

1

2: :

a) Estudia si les trajectòries es tallen, es creuen o son coincidents.b) Calcula la distància mínima entre totes dues trajectòries.

(Activitat de Selectivitat)

a) Determinem un punt i un vector director de cada recta.

Wu1 (1, 1, 2) P(1, 1, 1) Wv2 ( 1, 1, 0) Q(1, 0, 2) WPQ (0, 1, 1)

[Wu1, Wv2, WPQ]

1 1 2

1 1 0

0 1 1

2

u1 v2

i j k

1 1 2

1 1 0

2i 2 j ( 2, 2, 0)WW

WW

WW

W

Com que [Wu1, Wv2, WPQ] 0 i Wu1 Wv2 0 → r1 i r2 es creuen.

b)

d(r1, r2 )[ u1, v2 , PQ ]

u1 v2

2

2 2

2

2

WW W

W W

087 Donat un cub (hexaedre regular) de costat 1 dm, es considera una de les diagonals i la diagonal d’una de les cares de manera que aquestes no tinguin cap vèrtex en comú.

1 dm

Z

X

Y

Troba la distància entre aquestes diagonals.

Indicació: dibuixa el cub amb un vèrtex a l’origen de coordenades i els vèrtexs contigus sobre els eixos de coordenades.(Activitat de Selectivitat)

380

Productes vectorial i mixt

D1 diagonal que passa pels punts A(0, 0, 0) i B(1, 1, 1).

D2 diagonal que passa pels punts C(1, 0, 0) i D(0, 1, 0).

Calculem un punt i un vector director de cada diagonal..

WAB (1, 1, 1) A(0, 0, 0) WCD ( 1, 1, 0) C(1, 0, 0) WAC (1, 0, 0)

[ WAB , WCD, WAC ]

1 1 1

1 1 0

1 0 0

1

AB CD

i j k

1 1 1

1 1 0

i j 2k ( 1, 1, 2)W WW

WW

WW

W

Wu Wv 6

d(D1, D2 )[ u, v , AC ]

u v

1

6

6

6

WW

W

W

W

088 Un asteroide que segueix aproximadament la trajectòria donada per la recta:

r xy

z: 12

2 1

s’està apropant a un planeta situat al punt P (1, 1, 2).

a) Calcula la distància més propera a la qual es trobarà del planeta.

b) Calcula el punt de la trajectòria de l’asteroide on s’arribarà a aquesta distància mínima.

c) Si inicialment l’asteroide es troba en el punt Q 1 012

, , , calcula la distància

que haurà de recórrer per arribar a aquest punt.

(Activitat de Selectivitat)

a) Escrivim l’equació de la recta en forma contínua.

r: x 1

1

y

2

z1

2

1

2

Q 1, 0,1

2

u (2, 4, 1)

WPQ 2, 1,5

2

Calculem la distància de la recta al punt.

PQ u

i j k

2 15

22 4 1

9i 3 j 6k (9, 3, 6)W

W

WW

W

W

W

W

WPQ Wu 3 14

d(P , r )PQ u

u

3 14

(2, 4, 1)

3 14

216

W W

W

W

381

6SOLUCIONARI

b) Un punt genèric de la recta r és

R 1 2 , 4 ,1

2.

Trobem el valor de perquè el vector WPR

2 2, 4 1,5

2 sigui

perpendicular al vector director de la recta.

WPR Wu

2 2, 4 1,5

2(2, 4, 1) 21

21

20

1

2

El punt que compleix la condició és R(0, 2, 0).

c) WQR

1, 2,1

2

21

2

089 Donats el punt Q (3, 1, 4) i la recta r d’equacions paramètriques:

rxyz

:2 32

1 4es demana:

a) Trobar la distància del punt Q a la recta r.

b) Justificar que la recta s que passa per Q i que té (1, 1, 1) com a vector direccional no talla r.

c) Calcular la distància entre les rectes r i s.(Activitat de Selectivitat)

a) Un punt de la recta és A( 2, 0, 1) i un vector director és Wur (3, 2, 4) .

WQA

( 5, 1, 3)

QA ur

i j k

5 1 3

3 2 4

2i 11j 7k ( 2, 11, 7)WW

WW

WW

WW

WQA Wur 174

d(Q , r )QA ur

ur

174

(3, 2, 4)

174

296

W W

W

b)

s:

x 3

y 1

z 4

Wvs (1, 1, 1)

[Wur , Wvs, WQA]

3 2 4

1 1 1

5 1 3

6 0→ Les rectes es creuen.

c) ur vs

i j k

3 2 4

1 1 1

2i j k (2, 1, 1)

WWWW

WW

WW Wur Wvs 6

d(s, r )[ur , vs , QA]

ur vs

6

66

W

W

W

W

W

382

Productes vectorial i mixt

090 Considera la recta r x yy z

: 2 72 4

i el punt P (1, 2, 3).

a) Calcula les equacions paramètriques del pla que és perpendicular a la recta r i que conté el punt P.

b) Considera la recta:

sxyz

:123 2

Quina és la posició relativa entre la recta s i el pla ?

c) Calcula quines són les coordenades del punt Q de la recta s que està més proper a la recta r. Justifica la resposta.

(Activitat de Selectivitat)

a) Busquem un pla que passa pel punt P(1, 2, 2) i que té com a vector normal

el vector director de la recta.

r : x 2y 7

y 2z 4

x 1 4

y 4 2

z

Wur (4, 2, 1)

: 4 x 2y z D 0

4 1 2 2 3 D 0 D 3

: 4 x 2y z 3 0

:

x

y

z 3 4 2

b) Vector director de la recta s: Wvs (0, 1, 2)

Vector normal de : Wn

(4, 2, 1)

Wvs Wn 0 4 1 ( 2) 2 1 0 La recta és paral·lela al pla o hi està

continguda.

Com que el punt P(1, 2, 3) r i

P(1, 2, 3) , la recta i el pla són coincidents.

c) Un punt genèric de s és Q(1, 2 , 3 2 ).

Un punt genèric de la recta r és R( 1 4 , 4 2 , ).

WRQ (2 4 , 2 2, 2 3)

WRQ ha de ser perpendicular a les dues rectes:

RQ ur 15 21 0

RQ vs 5 4 0

WW

WW

5

7

4

5

→ El punt és

Q 1,6

5,

7

5.

091 Troba la distància del punt P (2, 1, 1) a la recta:

r

x

y

z

:

1323

(Activitat de Selectivitat)

383

6SOLUCIONARI

Wur (0, 1, 1)

A1

3,

2

3, 0 r WAP

5

3,

1

3, 1

ur AP

i j k

0 1 1

5

3

1

31

2

3i

5

3j

5

3k

2

3,

5

3,

5

3

WW W W W

W W W

Wur WAP 6

d(P , r )ur AP

ur

6

23

WW

W

092 Amb les dades de l’activitat anterior, troba el punt Q de r més proper a P. Comprova que la distància entre aquests punts és la mateixa que has calculat abans.

El punt Q és la intersecció de la recta r i el pla que passa per P

i que és perpendicular a la recta.

El pla que passa per P i que és perpendicular a r té com a vector normal

el vector director de r.

ur (0, 1, 1) : y z D 0

P(2, 1, 1) 1 1 D 0 D 2: y z 2 0

W

Calculem la intersecció entre la recta i el pla.

x1

3

y2

3

z

y z 2 0

2

3 →

Q1

3,

4

3,

2

3

Comprovem que la distància entre els dos punts coincideix amb la distància

del punt a la recta..

d(P , Q ) PQ5

3,

1

3,

1

33 d(P , r )W

093 Calcula la distància entre les rectes d’equacions:

r xy z

:1

34

7 s x

y z: 2

23

34

(Activitat de Selectivitat)

Trobem un punt i un vector de cada recta.

Wur (1, 3, 7) Pr(0, 1, 4) Wvs (1, 3, 4) Qs (2, 2, 3) WPrQs (2, 1, 1)

ur vs

i j k

1 3 7

1 3 4

9i 3 j ( 9, 3, 0)W W W WW W W

[PrQs , ur , vs ] PrQs ( ur vs ) (2, 1, 1) ( 9, 3, 0) 15W WW WW W

d(r , s)[ PrQs , ur , vs ]

ur vs

15

81 9

10

2

W W

W

W

W

384

Productes vectorial i mixt

094 Troba l’àrea del paral·lelogram definit pels vectors Wu (4, 3, 2) i Wv (2, 3, 3).

u v

i j k

4 3 2

2 3 3

3i 8 j 6k ( 3, 8, 6)W

WWW

WW

WW

Àrea del paral·lelogram 9 64 36 109

095 Donats els vectors Wu (1, 2, 0) i Wv (0, 1, 2), calcula:

a) El producte vectorial de Wu i Wv.b) Un vector unitari ortogonal a Wu i Wv.c) L’àrea del paral·lelogram que té com a costats els vectors Wu i Wv.(Activitat de Selectivitat)

a)

u v

i j k

1 2 0

0 1 2

4i 2 j k (4, 2, 1)W

WWW

WW

WW

b)

u v 21

El vector unitari i ortogonal als dos vectors donats és:

1

21( u v )

4 21

21,

2 21

21,

21

21

c) Àrea del paral·lelogram Wu Wv 21

096 Troba l’àrea del triangle de vèrtexs A(1, 2, 3), B(4, 3, 4) i C (5, 9, 9).

WAB

(3, 5, 1) WAC (4, 7, 6)

AB AC

i j k

3 5 1

4 7 6

37i 14 j 41k ( 37, 14, 41)

WW

WW

WWW W

WAB WAC 3.246

Àrea del triangle

1

2 WAB WAC

3 246

2

.

097 Els vectors Wu ( 4, 3, 2) i Wv (1, 2, 3) són dos dels costats d’un triangle.a) Calcula el perímetre d’aquest triangle.b) Troba’n l’àrea.

a) Perímetre Wu Wv Wu Wv

29 14 ( 4 1)2 (3 2)2 (2 3)2 14,18

b)

u v

i j k

4 3 2

1 2 3

5i 14 j 11k (5, 14, 11)W WW

W WW

WW

Àrea del triangle 1

2 Wu Wv

342

2

3 38

2

385

6SOLUCIONARI

098 Tres vèrtexs consecutius d’un paral·lelogram són A (3, 1, 0), B (4, 5, 2) i C (4, 7, 2). Troba’n el quart vèrtex i l’àrea.

AB

C

WAB

(1, 4, 2)

El punt D compleix que:

WOD WOC WAB (4, 7, 2) (1, 4, 2) (3, 3, 4) → D (3, 3, 4)

Calculem l’àrea.

WAD

(0, 2, 4)

AB AD

i j k

1 4 2

0 2 4

20i 4 j 2k ( 20, 4, 2)W WW

WW

WW

W

Àrea del paral·lelogram WAB WAD

( 20, 4, 2) 2 105

099 El centre del paral·lelogram ABCD és M (2, 2, 6). Determina els vèrtexs C i D, si saps que A (0, 1, 3) i B (1, 4, 5). Calcula’n també l’àrea.

AB

CD

M

El punt C compleix que: WOC WOA 2 WAM (0, 1, 3) 2(2, 1, 3) (4, 3, 9) →

C(4, 3, 9)

El punt D compleix que: WOD WOB 2 WBM (1, 4, 5) 2(1, 2, 1) (3, 0, 7) →

→ D(3, 0, 7)

AB AD

i j k

1 3 2

3 1 4

14i 2 j 10k (14, 2, 10)W WW

WW

WW

W

Àrea del paral·lelogram WAB WAD

(14, 2, 10) 10 3

100 Comprova que els punts A (2, 3, 0), B (4, 5, 2), C (7, 6, 5) i D (6, 1, 4) són coplanaris. Troba l’àrea del polígon ABCD.

Els punts són coplanaris si els vectors WAB (2, 2, 2), WAC (5, 3, 5) i WAD (4, 2, 2)

són linealment dependents.

2 2 2

5 3 5

4 2 4

0 → Els punts són coplanaris.

Calculem l’àrea del triangle descrit pels vectors WAB i WAC.

AB AC

i j k

2 2 2

5 3 5

4i 4k (4, 0, 4)W WW

WW

WW

Àrea T1

1

2 WAB WAC

1

24 0 4 2 2( , , )

Calculem l’àrea del triangle descrit pels vectors WAC i WAD.

AC AD

i j k

5 3 5

4 2 4

22i 22k (22, 0, 22)W WW

WW

WW

Àrea T2

1

2 WAC WAD

1

222 0 22 22 2( , , )

Àrea del quadrilàter 2 2 22 2 24 2

386

Productes vectorial i mixt

101 Calcula l’àrea d’un rombe de 2 cm de costat, si saps que té un angle de 60°.

Tenim dos vectors de mòdul 2 cm que formen un angle de 60°.

Àrea del rombe Wu Wv Wu Wv sin 2 2 sin 60° 2 3 cm2

102 Considera el triangle determinat pels vectors Wu (1, 0, 3) i Wv (2, 1, 4):

a) Troba’n l’àrea.b) Decideix si és acutangle, rectangle o obtusangle.

a) u v

i j k

1 0 3

2 1 4

3i 10 j k (3, 10, 1)W

WWW

WW

WW

Àrea del triangle 1

2 Wu Wv ( , , )3 10 1

110

2

b) Calculem els mòduls dels vectors que formen el triangle.

Wu 10

Wv 21

Wu Wv ( 1)2 ( 1)2 ( 7)2 51

L’angle format pels vectors Wu i Wv ha de ser l’angle més gran del triangle perquè

el costat Wu Wv és el de mida més gran.

cos u v

u v

10

10 21133° 38' 7"

W

W

W

W

El triangle és obtusangle.

103 Calcula l’àrea d’un triangle rectangle que té un catet sobre la recta:

rx y

z:3

31

42

i dos dels vèrtexs són P (5, 3, 4) i Q (6, 3, 3).

P

Q

r

Comprova que es verifica el teorema de Pitàgores.

Calculem el punt de tall de la recta r amb el pla que passa pel punt P

i és perpendicular a r.

El pla que passa pel punt P i és perpendicular a r té com a vector normal el vector

director de la recta r.

vr (3, 4, 1) : 3x 4 y z D 0

P(5, 3, 4) 3 5 4 ( 3) 4 D 0 D 7

W

El pla és : 3 4 7 0x y z

387

6SOLUCIONARI

Calculem la intersecció entre aquest pla i la recta r.

x 3

3

y 1

4

y 1

4z 2

3x 4 y z 7 0

x 3

y 1

z 2

El tercer vèrtex del triangle rectangle és A(3, 1, 2).

Calculem l’àrea del triangle.

AP AQ

i j k

2 2 2

3 4 1

10i 4 j 14k ( 10, 4, 14)W WW

WW

W WW

WAP WAQ 2 78

Àrea del triangle 1

2 WAP WAQ 78

Comprovem que es verifica el teorema de Pitàgores.

d(P , Q ) PQ 38W

d( A, Q ) AQ 26W

d( A, P ) AP 12W

[d(P , Q )]2 [d( A, Q )]2 [d( A, P )]2

382

262

122

38 26 12

104 Calcula perquè els punts A(1, 1, 1), B (3, 0, 2), C (5, 2, 2) i D (2, 1, ) siguin coplanaris. Calcula l’àrea del polígon ABCD.

(Activitat de Selectivitat)

Els punts són coplanaris si els vectors WCA

( 4, 3, 1), WCB ( , , )2 2 0 ,

i WCD

( 3, 3, 2) són linealment dependents.

4 3 1

2 2 0

3 3 2

4 2

Els punts són coplanaris si 4 2 0 2

Si D(2, 1, 2) , els vectors WCB ( 2, 2, 0) i WCD ( 3, 3, 0) són proporcionals;

per tant, els punts C, B i D estan alineats, i estan situats en aquest ordre.

La figura ABCD és un triangle de vèrtexs A, C i D.

CA CD

i j k

4 3 1

3 3 0

3i 3 j 3k (3, 3, 3)W WW

WW

W WW

Àrea del triangle

CA CD

2

(3, 3, 3)

2

3 3

2

W W

388

Productes vectorial i mixt

105 Considera el pla d’equació x y z5 3 i les rectes r i s amb les equacions:

r xy z

sx

y z: :32

24

31

22

Determina:a) Els punts d’intersecció del pla amb cada una de les dues rectes.b) L’àrea i el perímetre del triangle format pels dos punts anteriors i l’origen

de coordenades.(Activitat de Selectivitat)

a)

x 3y 2

2

x 3z 4

3

x 5y z 3 0

P(3, 2, 4)

x 1

2y

y z 2

x 5y z 3 0

Q( 1, 0, 2)

b) WOP

( 3, 2, 4) WOQ

(1, 0, 2) WPQ

( 4, 2, 6)

OP OQ

i j k

3 2 4

1 0 2

4i 2 j 2k ( 4, 2, 2)W WW

WW

W WW

WOP WOQ 2 6

Àrea del triangle OP OQ

2

2 6

26

W W

Perímetre del triangle

OP PQ OQ 29 2 14 5 15,1W W W

106 Determina el valor de a perquè els punts A (1, 0, 1), B (1, 2, 1) i C (1, 0, a) siguin els tres

vèrtexs d’un triangle d’àrea 72

.

WAB

(0, 2, 0) WAC

(0, 0, a 1)

AB AC

i j k

0 2 0

0 0 a 1

(2a 2)i (2a 2, 0, 0)W WW W

WW

WAB WAC

2a 2

Àrea del triangle

AB AC

2

2a 2

2

7

2

2a 2 7

2a 2 7

a9

2

a5

2

W W

107 Donades les rectes r x yx z

r x yy z1 2

2 22 1

: :i , es demana:

a) Determinar les coordenades del punt P en què es tallen i les equacions del pla que les conté.

b) Calcular l’equació de la recta s que passa pel punt Q (2, 0, 1) i que talla perpendicularment r1.

c) Obtenir les coordenades del punt R, intersecció de r1 i s, i l’àrea del triangle de vèrtexs P, Q i R.

(Activitat de Selectivitat)

389

6SOLUCIONARI

a) Calculem el punt d’intersecció.

x 2y 2

x 2z

x y

y z 1

P( 2, 2, 1)

El pla que conté aquestes dues rectes és un pla que passa

pel punt P ( 2, 2, 1) i que té com a vectors directors els vectors directors

de les rectes

r1:

x 2

y 1

z

v1 ( 2, 1, 1)W

r2:

x 1

y 1

z

v2 ( 1, 1, 1)W

:

x 2 y 2 z 1

2 1 1

1 1 1

y z 1 0

b) Trobem el pla perpendicular a r1 que passa per Q. Aquest pla té com a vector

normal el vector director de r1.

v1 ( 2, 1, 1) : 2x y z D 0

Q(2, 0, 1) 2 2 0 1 D 0 D 3: 2x y z 3 0

W

Calculem el punt de tall de r1 i .

2( 2 ) 1 3 02

3T

4

3,

1

3,

2

3

La recta que busquem, s, passa pels punts Q i T.

WQT

2

3,

1

3,

5

3 →

s :

x 2 2t

y t

z 1 5t

c) La intersecció entre r1 i s és el punt

T4

3,

1

3,

2

3.

WPQ

(4, 2, 0)

WPT

10

3,

5

3,

5

3

PQ PT

i j k

4 2 0

10

3

5

3

5

3

10

3i

20

3j

10

3,

20

3, 0W W

W W

W W

W

WPQ WPT 10 5

3

Àrea del triangle

1

2 WPQ WPT

5 5

3

390

Productes vectorial i mixt

108 En l’espai es consideren:

La recta r intersecció de dos plans d’equacions implícites:x y z 5 2 x y 2 z 2

I la recta s que passa pels punts: P (3, 10, 5) Q (5, 12, 6)Es demana:a) Calcular les equacions paramètriques de la recta r i de la recta s.b) Calcular el punt H, intersecció de r i s i l’angle que determinen r i s.c) Calcular els punts M i N de la recta r per als quals l’àrea de cada un dels triangles

de vèrtexs PQM i PQN és de 3 unitats.(Activitat de Selectivitat)

a)

r :

x 3

y 8

z

WPQ

(2, 2, 1) →

s :

x 3 2

y 10 2

z 5

b) Calculem la intersecció de les dues rectes.

3 3 2

8 10 2

5

4

1H(1, 8, 4)

L’angle que determinen r i s és el que determinen els seus vectors directors.

cosu v

u v

(1, 0, 1) (2, 2, 1)

2 3

1

2

2

245°

W

W

W

W

c) Un punt genèric de la recta r és M( 3 , 8, ).

WPM ( 6 , 2, 5 ) WPQ (2, 2, 1)

PM PQ

i j k

6 2 5

2 2 1

W WW W W

( ) ( ) ( )

( , , )

2 8 4 2 8

2 8 4 2 8

i j kW W W

WPM WPQ 3( 4)

Àrea del triangle PM PQ

2

3 4

23

4 2

4 2

6

2

W W

Els punts que busquem són M(3, 8, 6) i N( 1, 8, 2)..

109 Donats els punts: A(1, 2, 3) B (0, 2, 0) C (1, 0, 1)a) Prova que no estan alineats i escriu l’equació general del pla determinat

per aquests tres punts.b) Determina les equacions paramètriques de la recta que és l’altura del triangle ABC

corresponent al vèrtex C.c) Calcula l’àrea de ABC.d) Calcula un punt D del pla que has calculat

a l’apartat a) de tal manera que el triangle ABD compleixi les dues condicions següents: ABD és un triangle rectangle amb l’angle recte al vèrtex A. Àrea (ABD) Àrea (ABC )

(Activitat de Selectivitat)

AB

C D

391

6SOLUCIONARI

a) Els vectors WAB

( 1, 0, 3) i WAC ( , , )0 2 2 no són proporcionals;

per tant, els tres punts no estan alineats.

El pla que els conté passa pel punt A i té com a vectors

directors WAB i WAC .

:

x 1 y 2 z 3

1 0 3

0 2 2

6x 2y 2z 4 0 : 3x y z 2 0

b) L’altura que busquem és la recta perpendicular al segment AB que passa

pel punt C.

Calculem el pla que passa per C i que és perpendicular al segment AB.

Aquest pla té com a vector normal el vector WAB .

AB ( 1, 0, 3) : x 3z D 0

C(1, 0, 1) 1 1 0 3 1 D 0 D 4: x 3z 4 0

W

Trobem el punt de tall del pla amb la recta que passa per A i B.

rAB :

x 1 t

y 2

z 3 3t

1 t 3(3 3t ) 4 0 t3

5

El punt de tall és

C'

2

5, 2,

6

5

L’altura que busquem és la recta que passa per C i C'.

WCC'

3

5, 2,

1

5s :

x 1 3

y 10

z 1

c) AB AC

i j k

1 0 3

0 2 2

6i 2 j 2k ( 6, 2, 2)W WW

WW

WW

W

WAB WAC 2 11

Àrea del triangle

1

2 WAB WAC 11

d) L’altura dels dos triangles és WCC'

3

5, 2,

1

5

110

5.

WCC' és un vector perpendicular al costat AB. El punt D pertany a una recta

que passa per A i que té com a vector director WCC'.

m:

x 1 3

y 2 10

z 3

Per tant, el punt D ha de ser de la forma D(1 3 , 2 10 , 3 ).

Calculem l’àrea del triangle ABD.

WAB

( 1, 0, 3) WAD ( 3 , 10 , )

AB AD

i j k

1 0 3

3 10

30 i 10 j 10 k (30 , 10 , 10 )W WW

WW

WW

W

392

Productes vectorial i mixt

WAB WAD 10 11

Àrea del triangle

1

2 WAB WAD 5 11

Com que Àrea (ABD) Àrea (ABC):

Àrea

Àrea

( ABC ) 11

( ABD) 5 1111 5 11

1

5

1

5

1

5

Per tant, hi ha dos punts que compleixen les condicions:

D1

2

5, 4,

16

5i D2

8

5, 0,

14

5

110 Es donen els punts A (2, 1, 1) i B (1, 0, 1), i la recta:

r x yz

: 52

2Es demana que calculis raonadament:a) El punt C de r que equidista de A i B.b) L’àrea del triangle ABC.(Activitat de Selectivitat)

a) Un punt genèric de la recta r és C(5 , , 2 2 ).

WAC

( 3, 1, 2 3) WBC

( 4, , 2 1)

WAC WBC 6 2 16 19 6 2 12 171

2

Per tant, el punt és

C9

2,

1

2, 1 .

b)

AB AC

i j k

1 1 2

5

2

3

22

i 7 j 4k ( 1, 7, 4)W W

W

W

W

W

W

W

WAB WAC 66

Àrea del triangle

1

2 WAB WAC

66

2

111 Dels plans paral·lels al pla x y z 8, troba el que determina amb els eixos coordenats un triangle d’àrea 8 3 .

(Activitat de Selectivitat)

Els plans paral·lels a x y z 8 són de la forma x y z d .

Calculem els punts de tall d’un pla paral·lel al donat amb els eixos

coordenats.

Tall amb l’eix X:

x y z d

y 0

z 0

A(d , 0, 0)

393

6SOLUCIONARI

Tall amb l’eix Z:

x y z d

x 0

y 0

C(0, 0, d )

Tall amb l’eix Y:

x y z d

x 0

z 0

B(0, d , 0)

Calculem l’àrea del triangle ABC.

WAB

( d , d , 0) WAC ( d , 0, d )

AB AC

i j k

d d 0

d 0 d

d2 i d2 j d2k (d2 , d2 , d2 )W WW

WW

WW

W WAB WAC d2 3

Àrea del triangle 1

2 WAB WAC

d d

d

2 3

28 3

4

4

Els plans que busquem són: x y z 4 i x y z 4

112 Donats el pla : 3 2 10 0x y z i el punt P (1, 2, 3), es demana:

a) Trobar l’equació de la recta r perpendicular al pla que passa pel punt P.b) Trobar el punt Q intersecció de i r.c) Trobar el punt R intersecció de amb l’eix Y.d) Trobar l’àrea del triangle PQR.(Activitat de Selectivitat)

a) La recta que busquem passa pel punt P i té com a vector director el vector

normal del pla.

r :

x 1 3

y 2 2

z 3

b)

x 1 3

y 2 2

z 3

3x 2y z 10 0

1 Q( 2, 0, 4)

c) Eix Y:

x 0

y 0

Calculem la intersecció de amb l’eix Y.

3x 2y z 10 0

x 0

z 0

R(0, 5, 0)

d) WPQ

( 3, 2, 1) WPR

( 1, 7, 3)

PQ PR

i j k

3 2 1

1 7 3

13i 10 j 19k (13, 10, 19)W WW

WW

WW

W WPQ WPR 3 70

Àrea del triangle

1

2 WPQ WPR

3 70

2

394

Productes vectorial i mixt

113 Calcula el volum del paral·lelepípede descrit pels vectors Wu (3, 0, 4), Wv (2, 3, 5) i Ww (8, 5, 2).

[Wu , Wv , Ww ]

3 0 4

2 3 5

8 5 2

43

Volum del paral·lelepípede [Wu , Wv , Ww ]| 43

114 Els tres vectors següents descriuen un tetraedre, Wu (1, 2, 9), Wv (3, 6, 2) i Ww (3, 2, 2). Troba’n el volum.

[Wu , Wv , Ww ]

1 2 9

3 6 2

3 2 2

232

Volum del tetraedre

1

6 [Wu , Wv , Ww ]

232

6

116

3

115 Calcula el volum del paral·lelepípede que defineixen els vectors Wu ( 3, 2, 1), Wv (4, 5, 3) i Ww (7, 3, 4). Què hi observes? Dóna-hi una explicació.

[Wu , Wv , Ww ]

3 2 1

4 5 3

7 3 4

0

Volum del paral·lelepípede [Wu , Wv , Ww ]| 0

Els tres vectors són linealment dependents; per tant, els tres punts són coplanaris

i no descriuen cap paral·lelepípede.

116 Troba el valor de m perquè els vectors Wu (1, m 2, 1), Wv (2, 5, 0) i Ww (1, m, 1) determinin un paral·lelepípede de volum 6.

[Wu , Wv , Ww ]

1 m 2 1

2 5 0

1 m 1

14 4m

Volum [Wu , Wv , Ww ]

14 4m 614 4m 6

14 4m 6

m 2

m 5

117 Els punts A (0, 0, 1), B (1, 0, 2), C (0, 1, 2) i D (4, 1, 5) són els vèrtexs d’un tetraedre. Calcula’n volum.

WAB

(1, 0, 1) WAC

(0, 1, 1) WAD

(4, 1, 6)

[ WAB , WAC , WAD]

1 0 1

0 1 1

4 1 6

11

Volum del tetraedre

1

6 [ WAB , WAC , WAD]

11

6

118 El pla : 3x y 2 z 18 0 defineix amb els plans coordenats un tetraedre amb el vèrtex a l’origen de coordenades. Calcula’n el volum.

395

6SOLUCIONARI

Calculem els punts de tall del pla amb els eixos.

Tall amb l’eix X:

3x y 2z 18 0

y 0

z 0

A(6, 0, 0)

Tall amb l’eix Y:

3x y 2z 18 0

x 0

z 0

B(0, 18, 0)

Tall amb l’eix Z:

3x y 2z 18 0

x 0

y 0

C(0, 0, 9)

Calculem el volum del tetraedre.

[ WOA, WOB, WOC ]

6 0 0

0 18 0

0 0 9

972

Volum del tetraedre 1

6 [ WOA, WOB, WOC ]

972

6162

119 Determina el volum del tetraedre de vèrtexs A (1, 2, 0), B (0, 2, 1), C ( 1, 0, 6) i D (4, 1, 1).

Amb aquest volum, decideix si els punts són coplanaris o no.

WAB

( 1, 0, 1) WAC

( 2, 2, 6) WAD

(3, 1, 1)

[ WAB , WAC , WAD]

1 0 1

2 2 6

3 1 1

0

Volum del tetraedre

1

6 [ WAB , WAC , WAD] 0

Els quatre punts no determinen cap volum. Els punts són coplanaris.

120 Troba l’àrea del triangle que es forma en tallar el pla : 2 3 5 30 0x y z amb els tres eixos coordenats. Calcula també el volum que apareix quan unim aquests tres punts amb l’origen de coordenades, i fes-ho de dues maneres diferents.

Calculem els punts de tall del pla amb els eixos de coordenades

Tall amb l’eix X:

2x 3y 5z 30 0

y 0

z 0

P(15, 0, 0)

Tall amb l’eix Y:

2x 3y 5z 30 0

x 0

z 0

Q(0, 10, 0)

Tall amb l’eix Z:

2x 3y 5z 30 0

x 0

y 0

R(0, 0, 6)

396

Productes vectorial i mixt

Calculem l’àrea del triangle PQR.

WPQ

( 15, 10, 0) WPR

( 15, 0, 6)

PQ PR

i j k

15 10 0

15 0 6

60i 90 j 150k ( 60, 90, 150)W WW

WW

WW

W

WPQ WPR 30 38

Àrea del triangle

1

2 WPQ WPR 15 38

Calculem l’àrea del tetraedre OPQR.

Primera manera.

WOP

(15, 0, 0) WOQ

(0, 10, 0) WOR (0, 0, 6)

[ WOP, WOQ, WOR ]

15 0 0

0 10 0

0 0 6

900

Volum

1

6 [ WOP, WOQ, WOR ]

900

6150

Segona manera.

Podem calcular el volum del tetraedre mitjançant la fórmula del volum d’una

piràmide.

Agafem com a base el triangle inicial, i l’altura és la distància de l’origen

al pla determinat per aquest triangle.

d(O , )2 0 3 0 5 0 30

4 9 25

30

38

Volum

1

3Àrea de la base Altura

1

315 38

30

38150

121 Tenim els punts A(2, 3, 0) i B ( 2, 1, 4).Determina:a) L’equació del pla , mediatriu del segment AB.b) El volum del tetraedre format per i els tres plans coordenats.c) L’equació de la recta perpendicular al pla que passa per l’origen.Nota: el pla mediatriu d’un segment és perpendicular al segment i passa pel seu punt mitjà..

(Activitat de Selectivitat)

a) El pla passa pel punt mitjà M, del segment AB, i té com a vector normal

el vector director de la recta que passa per A i B.

M2 2

2,

3 1

2,

0 4

2M(0, 2, 2)

AB ( 4, 2, 4) : 4 x 2y 4z D 0

M(0, 2, 2) 4 0 2 2 4 2 D 0 D 4

W

El pla és : 2 2 2 0x y z

397

6SOLUCIONARI

b) Calculem els punts de tall del pla amb els eixos de coordenades.

Tall amb l’eix X:

2x y 2z 2 0

y 0

z 0

P( 1, 0, 0)

Tall amb l’eix Y:

2x y 2z 2 0

x 0

z 0

Q(0, 2, 0)

Tall amb l’eix Z:

2x y 2z 2 0

x 0

y 0

R(0, 0, 1)

Determinem el volum del tetraedre OPQR.

[ WOP, WOQ, WOR ]

1 0 0

0 2 0

0 0 1

2

Volum del tetraedre

1

6 [ WOP, WOQ, WOR ]

2

6

1

3

c) La recta que busquem passa per l’origen i té com a vector director el vector

normal del pla .

s :

x 2t

y t

z 2t

122 Tenim els punts P (8, 13, 8) i Q ( 4, 11, 8). Es considera el pla , perpendicular al segment PQ pel seu punt mitjà.

a) Troba l’equació del pla .b) Calcula la projecció ortogonal del punt O (0, 0, 0) sobre el pla .c) Troba el volum del tetraedre determinat pels punts en què el pla talla els eixos

de coordenades i l’origen de coordenades.(Activitat de Selectivitat)

a) El pla passa pel punt mitjà M, del segment PQ, i té per vector normal el vector

director de la recta que passa per P i Q.

M8 4

2,

13 11

2,

8 8

2M(2, 1, 0)

PQ ( 12 24, 16) : 12x 24 y 16z D 0

M(2, 1, 0) 12 2 24 1 16 0 D 0 D 48

W

El pla és : 3 6 4 12 0x y z

b) La projecció ortogonal del punt O sobre el pla és el punt de tall de

amb la recta perpendicular a que passa per O.

La recta perpendicular a que passa per O té com a vector director el vector

normal de .

r :

x 3

y 6

z 4

398

Productes vectorial i mixt

Calculem la intersecció entre i r.

x 3

y 6

z 4

3x 6y 4z 12 0

12

61

La projecció ortogonal és

R36

61,

72

61,

48

61

c) Calculem els punts de tall del pla amb els eixos de coordenades.

Tall amb l’eix X:

3x 6y 4z 12 0

y 0

z 0

A(4, 0, 0)

Tall amb l’eix Y:

3x 6y 4z 12 0

x 0

z 0

B(0, 2, 0)

Tall amb l’eix Z:

3x 6y 4z 12 0

x 0

y 0

C(0, 0, 3)

[ WOA, WOB, WOC ]

4 0 0

0 2 0

0 0 3

24

Volum del tetraedre 1

6 [ WOA, WOB, WOC ]

24

64

123 Tenim el prisma triangular (triangles iguals i paral·lels) de la figura, amb A (1, 1, 0), B (1, 0, 1), C (0, 1, 1) i A' (1, 1, ).Calcula:a) L’equació del pla que passa pels punts A, B i C.b) El valor de perquè el pla ', que conté els

punts A', B' i C', disti una unitat del pla .c) Per a 1, l’equació del pla ' i el volum del prisma.(Activitat de Selectivitat)

a) El pla passa pel punt A i té per vectors directors WAB i WAC.

WAB

(0, 1, 1) WAC

( 1, 2, 1)

:

x 1 y 1 z

0 1 1

1 2 1

x y z 0

b)

' és paral·lel al pla ' : x y z D 0

A'(1, 1, ) ' 1 1 1 ( 1) 1 D 0 D

El pla és ' : x y z 0

Com que i ' són paral·lels, agafem A(1, 1, 0) .

d( , ' ) d( A, ' )1 1 0

1 1 11

3

3

A'B'

C'

AB

C

'

399

6SOLUCIONARI

Hi ha dos punts que compleixen la condició:

A' 1, 1, 3 i A' 1, 1, 3

c) 1 ' : x y z 1 0

[WAB, WAC, WAA' ]

0 1 1

1 2 1

0 0 1

1

Volum del prisma

1

2 [WAB, WAC, WAA' ]

1

2

124 Troba les equacions de les rectes obtingudes en tallar cada un dels plans, 1 3: x y z , 2 0: x z i 3 0: y z amb el pla 4 0: z .

Aquests quatre plans limiten un tetraedre del qual s’obtindrà l’àrea de la cara situada al pla 4 i l’altura sobre aquesta cara, i explica el mètode utilitzat.

(Activitat de Selectivitat)

1 4 r1: x y z 3

z 0r1:

x 3 a

y a

z 0

2 4 r2 : x z 0

z 0r2 :

x 0

y b

z 0

3 4 r3 : y z 0

z 0r3 :

x c

y 0

z 0

La base del tetraedre és un triangle els vèrtexs del qual són els punts d’intersecció

d’aquestes rectes.

A r1 r2

x y z 3

z 0

x z 0

z 0

A(0, 3, 0)

B r1 r3

x y z 3

z 0

y z 0

z 0

B(3, 0, 0)

C r2 r3

x z 0

z 0

y z 0

z 0

C(0, 0, 0)

L’àrea de la base del tetraedre és l’àrea del triangle ABC.

AB AC

i j k

3 3 0

0 3 0

9k (0, 0, 9)W WW W W

W WAB WAC 9

4

400

Productes vectorial i mixt

Àrea de la base

1

2 WAB WAC

9

2

El vèrtex del tetraedre és el punt D, que és la intersecció dels tres primers plans.

D 1 2 3

x y z 3

x z 0

y z 0

D(1, 1, 1)

Calculem el volum del tetraedre.

WAB (3, 3, 0) WAC

(0, 3, 0) WAD

(1, 2, 1)

[WAB, WAC, WAD ]

3 3 0

0 3 0

1 2 1

9

Volum del tetraedre

1

6 [WAB, WAC, WAD ]

9

6

3

2

Apliquem la fórmula del volum d’una piràmide per calcular-ne l’altura.

Volum

Àrea de la base Altura

3 Altura

3 Volum

Àrea de la base

33

2

9

2

1

125 Considerem un paral·lelepípede de bases ABCD i EFGH, de manera que tenim:

A (3, 1, 0) B (0, 0, 1) C ( 1, 3, 1) E (6, 3, 0)Troba l’àrea d’una de les bases, el volum del paral·lelepípede i la distància entre les bases.

Calculem l’àrea del paral·lelogram ABCD.

WBA (3, 1, 1) WBC

( 1, 3, 0)

BA BC

i j k

3 1 1

1 3 0

3i j 10k (3, 1, 10)W WW W W

W W W WBA WBC 110

Àrea de la base ABCD WBA WBC 110

Calculem D, el quart vèrtex de la base ABCD.

WAB WDC

( 3, 1, 1) ( 1 d1, 3 d2 , 1 d3 ) D(2, 4, 0)

Calculem el volum del paral·lelepípede generat per WAB, WAD i WAE .

WAB ( 3, 1, 1) WAD

( 1, 3, 0) WAE

(3, 2, 0)

[WAB, WAD, WAE ]

3 1 1

1 3 0

3 2 0

11

Volum del paral·lelepípede [WAB, WAD, WAE ] 11

La distància entre les bases és l’altura del paral·lelepípede.

Volum Àrea de la base Altura AlturaVolum

Àrea de la base

11

110

110

10